MBE KAPLAN--TORTS Flashcards

1
Q

skip

A

skip

How well did you know this?
1
Not at all
2
3
4
5
Perfectly
2
Q

skip

A

skip

How well did you know this?
1
Not at all
2
3
4
5
Perfectly
3
Q

skip

A

skip

How well did you know this?
1
Not at all
2
3
4
5
Perfectly
4
Q
  1. A woman owned a beautiful Siamese cat. The cat had a value of $500. The woman allowed the cat to roam loose in the neighborhood. The cat frequently entered the backyard of a neighbor, who lived in the house next to the woman. The neighbor hated cats. One day, the neighbor looked out his kitchen window and saw the cat chewing on his prized rosebushes.
    The neighbor telephoned the woman and told her to retrieve her cat or he would kill it. The woman immediately ran over to the neighbor’s property to get the cat. In the interim, the neighbor loaded his rifle and went outside. The woman saw the neighbor with the gun and said, “Please don’t shoot my cat.” The neighbor responded, “I’m sick and tired of her destroying my roses.” The neighbor then pointed the rifle at the cat. Out of instinct, the woman dived toward the rosebushes to save her beloved cat. At that same moment, the neighbor’s hand shook, and the rifle accidentally went off. The bullet narrowly missed the woman’s head but struck the cat. Seeing her cat being shot, the woman was overcome with grief and became very emotionally distraught, which caused her to suffer a heart attack. Fortunately, the cat survived the shooting but suffered a gunshot wound to her leg.
    The woman brings suit to recover damages against the neighbor. Which of the following causes of action would afford the woman her maximum recovery?
    (A) Battery.
    (B) Conversion.
    (C) Trespass to chattels.
    (D) Negligent infliction of emotional distress.
A
  1. (D) The key to this question is carefully reading the interrogatory or “stem,” which asks you to determine the cause of action that would afford the woman her maximum recovery. The woman’s cat is worth only $500, so that would be the maximum amount she could recover with a conversion or trespass to chattels claim. Choice (D), therefore, is the correct answer here, because a negligent infliction of emotional distress claim would give the woman a chance to recover for her emotional distress and the damages might be much higher than $500. Choice (A) is incorrect because battery requires harmful or offensive contact to a person. Here, the woman’s cat suffered a harmful contact, but the woman herself did not. Choice (B) is incorrect because the damages awarded for a conversion claim would be $500, because that was the value of the chattel. Choice (C) is incorrect because the damages for a trespass to chattels claim wouLd be measured by the diminished value of the chattel, which could not exceed $500 here. Exam Tip: The distinction between conversion and trespass to chattels (especially with regard to damages) is often tested on the Multistate exam.
How well did you know this?
1
Not at all
2
3
4
5
Perfectly
5
Q
  1. A patient underwent heart bypass surgery at a hospital. Members of the patient’s family had donated blood to be used during the operation. The patient told the doctor she would agree to the operation only if any blood given to her during the operation came from the blood donated by her family. When the doctor performed the surgery, he requisitioned blood from the hospital’s blood bank for the patient’s transfusion instead of using her family’s donated blood. It was the customary practice of doctors in the community to use blood from the hospital’s blood bank during surgery. ft was later determined that the hospital blood given to the patient was tainted with HIV. The hospital had not properly screened the blood for the presence of the HIV virus. The patient did not contract the HIV virus.
    If the patient asserts an action against the doctor for battery, she will
    (A) prevail, because consent to the operation was based upon the use of her family’s blood.
    (B) prevail, because the hospital failed to properly screen the blood to detect its contamination.
    (C) not prevail, because the patient did not contract the HIV virus.
    (D) not prevail, because it was the customary practice of doctors in the community to use blood from the hospital’s blood bank during surgery.
A
  1. (A) A physician must obtain the patient’s consent to provide treatment. In some emergency circumstances, the patient’s consent may be implied, such as where the patient is unconscious and needs immediate treatment. Where a patient’s consent is limited in some manner, however, the physician must respectthe patient’s wishes and will be liable for battery if the scope of the patient’s consent is exceeded. Here, choice (A) is correct because the patient’s consent to the operation was based on the use of family blood, so the doctor did not have actual or implied consent to use blood from the hospital bank. Choice (B) is incorrect because it addresses whether the patient could prevail on a negLigence cause of action against the hospital, but this question is about liability for battery rather than negligence. Choice (C) is incorrect because she could hold the doctor liable for battery, even if she did not become HIV-positive. She would be entitled to damages for the unwanted, offensive contact that occurred when she received the transfusion of hospital blood, regardless of her HIV status. Choice CD) is incorrect because the fact that doctors generally used blood from the blood bank would not eliminate the doctor’s obligation to obtain his patient’s consent. The customary practices of doctors in the community would be very relevant if she were suing the doctor for negligence, but this question is about a battery claim, not negligence.
How well did you know this?
1
Not at all
2
3
4
5
Perfectly
6
Q
  1. A man decided to stop at a drive-through hamburger stand for a late snack. As he drove up to the drive- through line, the manager of the hamburger stand informed him through the intercom system that the restaurant was closing and no further orders would be accepted. She told the man that the last car to be served was the one directly in front of him. The man became angry and yelled into the intercom machine, “Listen, babe, I am hungry. I want two cheeseburgers, a large order of fries, and a Coke.” The manager retorted, “I’m terribly sorry, but we cannot accept your order.”
    Shortly thereafter, the manager handed the food order to the passengers in the car immediately in front of the man’s. When the man saw the manager serving that car, he became very angry, drove his automobile up to the service window and shouted at the manager, “You can’t do this to me.” When the manager laughed, the man suddenly reached into the car’s glove compartment and pulled out a gun. He aimed at the manager and fired the weapon, intending to hit her. The bullet missed the manager but hit a customer, wounding him in the head.
    In an action by the customer against the man for battery, the customer will be
    (A) successful, because the man intended to shoot the manager.
    (B) successful, because there was a “substantial certainty” that the customer would be hit by the bullet.
    (C) unsuccessful, because the man could not foresee that the bullet would hit anyone other than the manager.
    (D) unsuccessful, because the man did not intend to shoot the customer.
A
  1. (A) Under the doctrine of “transferred intent,” if the defendant shoots or strikes at A, intending to wound or kill him, but misses A and hits B instead, the defendant is held liable to B for battery. The intent to commit a battery to A is “transferred” or carried over to create liability to B. The man intended to shoot the manager, but he missed and, instead, hit a customer. The doctrine of transferred intent will enable the customer to hold the man liable for battery. Choice (B) is incorrect because it would be difficult in these circumstances to prove that the man knew the bullet was substantially certain to hit the customer. Given that the doctrine of transferred intent clearly applies, it is unnecessary for the customer to try to prove the man’s knowledge of a substantial certainty. Choice (C) is incorrect because it does not matter whether the man could foresee the bullet hitting someone other than the intended target. The doctrine of transferred intent applies, even if it was compLeteLy unforeseeable that the plaintiff would be hit. Choice (D) is incorrect because the doctrine of transferred intent supplies the intent required for the man’s customer’s battery claim.
How well did you know this?
1
Not at all
2
3
4
5
Perfectly
7
Q
  1. A taxicab driver stopped at a convenience store to buy a snack. While he was handing his money to the store clerk at the cash register, a police officer entered the store. The taxi driver had received several traffic tickets from the police officer, and he felt that the police officer had been very rude to him. Seeking to gain some revenge, the taxi driver immediately began to taunt and berate the police officer, calling him a variety of profane and insulting names. The police officer was going through a series of personal difficulties and was feeling very emotionally fragile. As the taxi driver’s insults rained down on him, the police officer suddenly snapped. He whipped out his gun and fired a shot at the taxi driver. The bullet missed the taxi driver but hit the store clerk, who was standing behind the taxi driver. The clerk survived the shooting but suffered a gunshot wound to his shoulder.
    In an action by the injured clerk against the taxi driver to recover for his personal injuries, plaintiff will most likely
    (A) recover, because the taxi driver’s conduct was the proximate cause of the clerk’s injuries.
    (B) recover, because it was foreseeable that the taxi driver’s conduct would result in the clerk being shot.
    (C) not recover, because the shooting was not a foreseeable consequence of the taxi driver’s conduct.
    (D) not recover, because the police officer intended to shoot the taxi driver, not the store clerk.
A
  1. (C) The taxi driver did not intend for the shooting to occur, so an intentional tort claim would not be successful here. The plaintiff couLd claim that the taxi driver acted negligently, but will be unable to prove that the taxi driver’s conduct was a proximate cause of the shooting ard resulting injury. Proximate cause exists only if the plaintiff was within the general class of people that one could reasonably foresee being hurt, and the plaintiff’s injury was within the general type of harm that one could reasonably foresee occurring. Here, even if it was negligent for the taxi driver to insult and taunt the police officer, it was not foreseeable that the store clerk would be shot as a result of that negligence. In other words, no one could have foreseen that the police officer would snap and try to shoot someone who insulted him. Choice (A) is incorrect because, proximate cause is lacking here. Choice (B) is incorrect because the shooting was unforeseeable. Choice (D) is incorrect because it addresses the police officer’s intent, which would be very relevant for a cLaim against the police officer, but is not what will control whether the taxi driver can be held liable.
How well did you know this?
1
Not at all
2
3
4
5
Perfectly
8
Q
  1. An elderly woman was riding in an elevator from her apartment on the 12th floor to the lobby. When the elevator stopped on the fifth floor, a man entered the elevator smoking a cigar. The man was standing in front of the elderly woman on the elevator when the woman tapped him on the shoulder. When the man turned around, the woman pointed to the “No Smoking” sign and said, “Excuse me, sir, would you mind putting that cigar out?” The man indignantly responded by inhaling heavily on his cigar, and then he blew a big puff of smoke into the woman’s face. When the elevator stopped on the next floor, the man then departed.
    If the woman brings a civil suit against the man for battery, who will prevail?
    (A) The woman, because the smoke touched her face.
    (B) The woman, because she had a reasonable basis for fearing that the man would attack her.
    (C) The man, because he did not touch the woman.
    (D) The man, because his conduct was annoying but did not inflict any bodily injury on the woman.
A
  1. (A) A person is subject to liability for battery if he acts intending to cause a harmful or offensive contact with the person of another. The essence of the plaintiff’s grievance is the offense to her dignity and invasion of the inviolability of her person, so a battery claim can arise even if the plaintiff suffers no physicaL harm to her body. In addition, the contact necessary for a battery claim can be accomplished directly or indirectly. For example, it can be a battery if someone hits you with his fist, but it also can be a battery if someone stabs you with a knife, throws a rock that strikes you, or fires a bullet that hits you. In addition, it can even be a battery if someone blows smoke in your face, if the contact between the smoke and your face is offensive. Choice (A), therefore, is correct, because the woman should be able to hold the man liable for battery. Choice (B) is incorrect because if the woman feared that the man might attack her, that would be a basis for an assault claim, not a battery claim. Choice (C) is incorrect because although the man did not directly touch the woman with his body, he caused the smoke to touch her, and that is sufficient to create a battery claim. Choice (D) is incorrect because battery can occur where a contact is offensive, not just where it is physically harmful.
How well did you know this?
1
Not at all
2
3
4
5
Perfectly
9
Q
  1. A woman lived in a town that has an ordinance that prohibits littering. While the woman was taking a stroll one morning, she passed a man who was standing on the sidewalk and eating a doughnut. As he finished the doughnut, the man wiped his mouth with a paper napkin. He then wadded up the napkin into a small ball and dropped it on the sidewalk. Seeing this, the woman stopped and admonished the man for throwing his trash on the ground. She told him about the town ordinance that prohibits littering and said that a police officer could give him a citation that would require him to pay a fine. The man was very insulted and annoyed to be lectured like this.
    If the man institutes a civil action against the woman, the man will most likely
    (A) recover for battery.
    (B) recover for negligence.
    (C) not recover, because the fact that an ordinance prohibited the man’s conduct would preclude recovery as a matter of law.
    (D) not recover, because the woman’s conduct was customary and reasonable under the circumstances.
A
  1. (D) The man cannot satisfy the elements for any tort claim against the woman. Choice (A) is incorrect because a battery occurs when the defendant intentionally causes a harmful or offensive contact to another person. The man experienced no such harmful or offensive contact. Choice (B) is incorrect because negligence occurs when the defendant’s failure to exercise reasonable care causes harm to another person. The woman did not fail to act with reasonable care. Choice (C) is incorrect because the fact that the man violated the littering ordinance would not necessarily bar him from recovering if he otherwise could prove the elements of a tort claim. For example, if the woman had hit the man to punish him for littering, the man would be able to hold the woman liable for battery despite having violated the littering ordinance. By the process of elimination, that leaves only choice (0), which correctly states that the woman’s conduct was reasonable, and the man will not be able to hold her liable for any tort.
How well did you know this?
1
Not at all
2
3
4
5
Perfectly
10
Q
  1. A boat owner was sailing his boat at sea when a storm advisory was suddenly issued. The boat owner was navigating his craft toward shore when heavy rain and wind gusts began to rock the boat. Unable to reach his own slip, the boat owner docked his boat on a pier owned by a resident who lived in a nearby house.
    The resident objected to the boat owner’s entry onto his private property. The boat owner explained that due to the storm it would be highly dangerous to be at sea. He requested permission to temporarily dock his boat until the storm subsided. The resident refused and cut the rope tying the boat to the dock. As a result, the boat drifted out to sea. The boat was battered by the rocky waves and high winds. Trying to prevent the boat from capsizing, the boat owner fell and broke his leg. The boat was extensively damaged in the storm, as well.
    The boat owner has sued the resident, seeking to recover damages for his personal injury, as well as damage to the boat. The boat owner should
    (A) recover for the damage to the boat, but not recover for his personal injury.
    (B) recover for his personal injury, but not recover for the damage to the boat.
    (C) recover both for damage to the boat and for his personal injury.
    (D) not recover for either damage to the boat or his personal injury.
A
  1. (C) A person who trespasses on the property of another without a privilege can be held liable for doing so. On the other hand, a defender of property can be held liable for using force to prevent entry of someone who is actually privileged to intrude. In this question, the boat owner was privileged to dock his boat on the resident’s property because of the impending storm. Conversely, the resident was not entitled to prevent the boat owner from intruding upon his property. Therefore, the resident will be subject to liability for the boat owner’s personal injury, as well as for the damage to the boat. Choices (A), (B), and (D) are incorrect, as incomplete damages are given. Rememberthat when the defense of necessity is found to exist, damages are awarded for BOTH personal injuries and property loss.
How well did you know this?
1
Not at all
2
3
4
5
Perfectly
11
Q
  1. A man and a woman are adjoining homeowners in a residential neighborhood. The man begins to operate a slaughterhouse in his backyard. The putrid smell from the slaughterhouse travels over the woman’s property. The woman and her family members find the odor to be foul and nauseating. As a result of the man’s slaughterhouse, the value of the woman’s property was diminished by 20 percent, and the woman incurred expenses for prescription drugs to treat her family’s nausea. There is no applicable zoning ordinance preventing the man’s slaughterhouse operation.
    The woman has asserted a nuisance action against the man. The complaint seeks recovery for damages and injunctive relief enjoining operation of the slaughterhouse. The court should
    (A) award damages, but not grant injunctive relief.
    (B) grant injunctive relief, but not award damages.
    (C) grant injunctive relief and award damages.
    (D) neither award damages nor grant injunctive relief, because there was no violation of any ordinance.
A
  1. (C) In an action for an injunction, the court may, in addition to or in lieu of granting an injunction, give one or more of the following remedies: (1) a judgment for damages; (2) a judgment for the recovery of property, or (3) a declaratory judgment. In order to avoid unnecessary litigation, courts having equity powers commonly award damages for torts committed prior to or pending suit, in addition to an injunction, in the same action. The plaintiff would not be required to make an election of remedies (i.e., to choose between seeking damages or injunctive relieO unless awarding both types of relief would be redundant and overcompensate the plaintiff. Choice (C) is thus the best answer because damages would be an appropriate remedy here to compensate the plaintiff for his financial loss. In addition, the court should also grant injunctive relief to prevent the nuisance from continuing in this residential neighborhood. Choices (A), (B), and (D) are incorrect because both injunctive relief and damages can be awarded.
How well did you know this?
1
Not at all
2
3
4
5
Perfectly
12
Q
  1. A rider entered a subway car at the 42nd Street station. Because all of the seats were occupied, the rider stood in the subway car and grabbed a pole to secure his balance. As the subway car was proceeding downtown, the rider glanced at a girl standing next to him. Suddenly, the subway car made an unexpected stop. The rider momentarily lost his balance and grabbed the girl around the waist (to avoid falling). Once the rider regained his balance, he removed his hands from the girl’s waist and grasped the pole again.
    In a civil action instituted by the girl against the rider, he will most likely be found
    (A) liable for battery.
    (B) liable, because the rider believed that the girl consented to the contact under the
    circumstances.
    (C) not liable, because the rider’s conduct was socially acceptable under the circumstances.
    (D) not liable, because the girl was not harmed by the contact.
A
  1. (C) Battery is an intentional tort in which the defendant causes harmful or offensive contact to another person. Whether contact is offensive is judged by an objective standard, meaning that it depends on what an average reasonable person would think. A contact is offensive if it would offend an ordinary person’s reasonable sense of personal dignity, and not if it would only be offensive to someone who is unduly sensitive. Thus, the contact must be unwarranted by the social usages prevalent at the time and place at which it is inflicted. Under the circumstances presented here, the rider’s conduct would be socially acceptable. He intended to touch the girl, but only to prevent himself from falling. He, therefore, would not be liable for battery. Choice (A) is incorrect because the offensive or harmful contact required for a battery is not present. Choice (B) is incorrect because the rider does not even need to raise the issue of whether the girL consented or if he thought she consented. Consent is an affirmative defense to an intentional tort claim, but the case will not get that far because the eLements of a battery claim cannot be established by the plaintiff. Choice (D) is incorrect because a battery requires contact that is either harmful or offensive. The fact that the girl was not harmed here, therefore, does not, by itself, prevent Liability from being imposed. It is the fact that the contact was neither harmful nor offensive that is crucial.
How well did you know this?
1
Not at all
2
3
4
5
Perfectly
13
Q

A fortune teller told fortunes by means of Tarot cards. An elderly woman, who was worried about her failing health, had heard that the fortuneteller was clairvoyant and could see into the future. Consequently, the woman decided to see the fortuneteller in order to have her Tarot cards read. As the fortuneteller was telling the woman her fortune, she suddenly said, “I have a vision. If you give me $25,000 tomorrow, you will live to be 100 years old.” The woman, who was 72 years of age, believed the fortuneteller and gave her the money the next day. The following week, the woman’s physician informed her that she had a serious heart disease and he didn’t expect her to live for more than a year or two.
If the woman asserts a claim against the fortuneteller based on deceit, the plaintiff should
(A) prevail, because she relied to her detriment on the fortune teller’s foretelling.
(B) prevail, if the fortuneteller did not honestly believe that the woman would live to be 100 years of age.
(C) not prevail, unless there was a fiduciary relationship between the parties.
(D) not prevail, unless the fortuneteller warranted the truth as believed.

A
  1. (B) The tort of deceit (also known as intentional misrepresentation or fraudulent misrepresentation) requires proof that a faLse representation has been made (1) knowingly, or (2) without belief in its truth. The intent that becomes important is the intent to deceive, to mislead, or to convey a false impression. This intent, which has been given the name “scienter” by the courts, must be a matter of belief, or an absence of belief, that the representation is true. Certainly, there is no difficulty in finding the required intentto mislead where it appears that the speaker believes his statement is false. Choice (A) is incorrect because reliance would not be sufficient to create liability if the scienter requirement is not satisfied. Choice (C) is incorrect because a deceit action requires that a fiduciary relationship exist between the plaintiff and defendant. Choice (D) is incorrect because if the fortuneteller warranted the truth, then no action for deceit would be present.
How well did you know this?
1
Not at all
2
3
4
5
Perfectly
14
Q

While relaxing at poolside one Sunday afternoon, a homeowner was struck by a golf ball driven by a 14-year-old boy who was playing the ninth hole at the local golf course. The fairway for the ninth hole was 65 feet wide and 437 yards long, with a dogleg in an easterly direction. Between the fairway and the homeowner’s property was a “rough,” containing brush and low-lying trees. The boy had hit a towering shot down the middle of the fairway that deflected off a tree and struck the homeowner in the head.
The homeowner brought suit against the boy for his injuries suffered when he was hit by the golf ball. At trial, the boy offered uncontested evidence that golf balls from the golf course regularly traversed onto the homeowner’s property two to three times a day.
Which of the following statements is most accurate regarding the boy’s liability for trespass?
(A) The boy is not liable, because he did not inten
tionally cause the golf ball to traverse onto the
plaintiff’s property.
(B) The boy would be liable for the unpermitted
intrusion of the golf ball onto the plaintiff’s
property.
(C) Because the plaintiff should have reasonably
anticipated that living next to a golf course
would result in stray golf balls landing on his
property, the boy would not be held liable.
(D) Because the golf ball did not substantially
interfere with the plaintiff’s use and enjoyment
of his land, the boy would not be held liable.

A
  1. (A) The intentional tort of trespass to land occurs where the defendant intentionally causes someone or something to enter or to remain on land of another person. A defendant can also be held liable for negligently causing a trespass to land that injures someone. Here, the boy caused the golf ball to enter the homeowner’s property, but the facts do not indicate that this was intentional or negligent. Choice (B) is incorrect because liability will not exist without proof of intent or negligence. Choice (C) is incorrect because liability could exist, even if the entry of the golf ball was foreseeable, if it was done intentionally or negligently. Choice (D) is incorrect because a negligence or intentional trespass to land claim would not require proof of substantial interference with the plaintiff’s use and enjoyment of his land. That language is associated with the tort of private nuisance, not negligence or trespass.
How well did you know this?
1
Not at all
2
3
4
5
Perfectly
15
Q
  1. After watching a television program about archery, a 15-year-old boy became very interested in that sport. He saved up some money to buy a bow and other archery equipment. He set up a target in his backyard in order to practice. He surrounded the target with stacks of hay bales to stop any arrows that missed the target. After practicing for a few weeks, the boy made great improvements in his technique and accuracy. While practicing one afternoon, however, the boy lost his balance just as he released an arrow. As a result, the arrow flew way off course, going over the target and all the hay bales and flying into a wooded area behind the boy’s house. The boy assumed no one was in the woods, so he was relieved that the errant arrow would not hurt anyone. However, a hiker happened to be in the woods near the boy’s house. As he paused for a moment to take a drink of water, the hiker suddenly saw something out of the corner of his eye. It was the stray arrow shot by the boy. Without even thinking about it, the hiker reflexively ducked out of the way. The arrow narrowly missed the hiker’s head; but as the hiker ducked out of the way, his head struck the limb of a tree, causing a stick to stab painfully into his eye.
    Which of the following causes of action could the hiker successfully assert against the boy?
    (A) Assault, but not battery.
    (B) Battery, but not assault.
    (C) Assault and battery.
    (D) Neither assault nor battery.
A
  1. (D) No intentional tort occurred here. Battery requires proof that the defendant did an act with intent to cause a harmful or offensive contact to another person. Assault requires proof that the defendant intended to put another person in apprehension of imminent harmful or offensive contact. For purposes of intentional torts, “intent” means that the defendant had a desire or purpose for a certain result to occur or that the defendant knew with substantial certainty that the result would occur. Here, the boy did not desire to hit anyone with an arrow or to put anyone in apprehension of being hit. He also did not have knowledge of a substantial certainty that someone would be hit. Choices (A), (B), and (C), therefore, are incorrect because no battery or assault occurred.
How well did you know this?
1
Not at all
2
3
4
5
Perfectly
16
Q

A 12-year-old girl and her friends were playing catch with a baseball in the middle of a field of grass in the park. Near the edge of the field, a woman was sitting in a beach chair and reading a book. The girl threw the ball too far, and it went over her friends’ heads and flew toward the woman. Although the woman did not see the ball coming, it hit the straw hat that the woman was wearing, knocking it from her head. Although the woman was not touched by the ball, she was startled by the ball hitting her hat, and she fell from her chair and broke her arm.
If the woman initiates a suit against the girl to recover damages for her broken arm, the woman will
(A) recover for assault only.
(B) recover for battery only.
(C) recover for assault and battery.
(D) not recover.

A
  1. (D) Intent is a crucial element of intentional torts, including battery and assault. Battery requires proof that the defendant did an act with intent to cause a harmful or offensive contact to another person. Assault requires proof that the defendant intended to put another person in apprehension of imminent harmful or offensive contact. For purposes of intentional torts, “intent” means that the defendant had a desire or purpose for a certain result to occur or that the defendant knew with substantial certainty that the result would occur. Here, the girl did not have the required intent. Choices (A), (B), and (C), therefore, are incorrect because nO battery or assault occurred.
How well did you know this?
1
Not at all
2
3
4
5
Perfectly
17
Q

A sportsman was the owner of an old dilapidated stadium, which was located on the outskirts of the city. The stadium, which was built in 1932, had been the home stadium for the local professional baseball team for 30 years. However, in 1962, the baseball team franchise moved to another city. Since 1962, the stadium was left unattended and had deteriorated to such an extent that the walls were in danger of collapsing.
Last month, an earthquake struck the city. The earthquake, which registered 6.9 on the Richter scale, caused considerable damage in the city and caused the stadium to collapse. As the stadium crumbled to the ground, a large section of the press box fell on top of a car that was parked nearby. The auto was crushed, causing its gas tank to rupture. As a result, a large quantity of gasoline spilled along the street and flowed downhill. The gasoline collected in front of a homeowner’s home, which was located about a mile from the stadium.
Two hours after the earthquake struck, a pedestrian was walking in front of the homeowner’s home, smoking a cigarette. When he discarded his lighted cigarette butt in the street, the gasoline exploded. The explosion blew the windows out of the homeowner’s home. The homeowner, who was sitting in the living room watching television, was struck by the flying glass and injured.
If the homeowner asserts a claim for his injuries against the sportsman, which of the following is the sportsman’s best defense?
(A) The earthquake was an act of God.
(B) The sportsman’s negligence, if any, merely created a passive condition and was not the active
cause of the homeowner’s injury.
(C) The sportsman could not reasonably have been
expected to foresee injury to a person in the
homeowner’s position.
(D) The pedestrian’s act of discarding the lighted
cigarette in the street, which sparked the explosion, was the proximate cause of the homeowner’s injury.

A
  1. (C) Foreseeability and proximate causation are specific areas that are highly tested. Students must be able to analyze multiple negligent acts that follow directly (i.e., an uninterrupted chain of events) orindirectly (i.e., where a period of time passes between intervening events) from the initial negligent conduct of the defendant, and then determine, by applying a test of foreseeability, whether or not the defendant’s initial conduct remains as the proximate, or legal, cause of the plaintiff’s harm. In this question, it must be determined whether the homeowner, the plaintiff, was a foreseeable plaintiff to whom the sportsman owed a duty. According to the majority view, a defendant owes a duty of care only to those plaintiffs who are foreseeably within the risk of harm created by the defendant’s conduct (i.e., within the “zone of danger”). If the sportsman could not reasonably have been expected to foresee injury to a person in the homeowner’s position, then no duty would be owed and, therefore, the sportsman’s negligence could not be the proximate cause of the homeowner’s harm. Here, when the sportsman negligently failed to keep the stadium in safe condition, he could have reasonably foreseen that an earthquake might cause it to collapse and injure someone who was in the stadium or next to it. He could not have reasonably foreseen that the condition of the stadium would cause someone to be injured, like the homeowner, who was watching television in his house a mile away. Choice (A) is incorrect because a defendant can be held liable for foreseeable consequences of negligence even if the particular manner in which the harm occurred involved an unexpected natural force, like an earthquake or weather. Choice (B) is incorrect because a defendant can be Liable even if its negligence is a “passive” rather than an “active” cause of the plaintiff’s harm. Choice (D) is incorrect because an injury may have more than one proximate cause. The question is whether the sportsman’s negligence was “a” proximate cause of the plaintiff’s injury, not whether it was “the” proximate cause. The fact that the pedestrian was the most proximate or direct cause of the injury, therefore, would not necessarily preclude the sportsman from being held Liable.
How well did you know this?
1
Not at all
2
3
4
5
Perfectly
18
Q

A tall building was under construction in the downtown business district of a city. A lawyer drove her car to the downtown area to go to a meeting with a client. She was running late for the meeting and could not find a legal parking spot, so she decided to park illegally in front of a fire hydrant. This was in violation of a local ordinance that prohibited parking within 50 feet of a fire hydrant. The lawyer figured that it was better to get a ticket than to miss her meeting with an important client. The spot where she parked was next to the site of the construction of the new building.
While the lawyer was at her meeting, an accident occurred at the construction site. A large crane was being used to lift a load of bricks. A cable on the crane broke, and the bricks fell. Most of them landed on top of the lawyer’s car. The load of bricks that fell on the lawyer’s car caused the gas tank of the car to rupture and explode. Shrapnel from the explosion flew in all directions and injured a pedestrian who was talking on the sidewalk near the lawyer’s car.
If the pedestrian sues the lawyer and relies on the doctrine of negligence per Se, which of the following, if true, is the lawyer’s best defense?
(A) Payment of a small fine is the only penalty provided in the ordinance for those who park too close to fire hydrants.
(B) The police never issued a ticket to the lawyer for parking in front of the fire hydrant.
(C) The purpose. of the parking ordinance was to
facilitate access to the hydrant by fire trucks,
not to protect against accidents like the one
that occurred when bricks fell on the lawyer’s
car.
(D) The pedestrian would not have been injured if
the construction company had properly maintained the crane.

A
  1. (C) For a negligence claim, the jury usually must assess all the facts and circumstances to decide whether the defendant acted with as much care as a reasonable person. In most states, however, the doctrine of “negligence per Se” provides a shortcut to proving that the defendant failed to exercise reasonable care. A defendant who violated a statutory standard of care will be treated as negligent per Se, meaning that the defendant’s conduct will automatically be deemed to be negligent. Breach of a statutory duty will be negligence per se only if two requirements are met: (1) the plaintiff must be a member of the class of persons meant to be protected by the statute; and (2) the plaintiff’s injury must be of the type of harm the statute was designed to prevent. Here, if the defendant lawyer can show that the plaintiff pedestrian’s injuries were not of the type intended to be prevented by enactment of an ordinance prohibiting parking within 50 feet of a fire hydrant, she will not be negligent perse. Therefore, choice (C) is correct. Exam Tip: A small minority of jurisdictions follow the rule that violation of a criminal statute is merely evidence of negligence, but not negligence per se. Students should follow the majority rule on the MBE, unless the facts say otherwise. Choice (A) is incorrect because negligence per se can apply even if the criminal penalty for violating a statute is only a minor fine. Choice (B) is incorrect because negligence per se would not require proof that the lawyer received a ticket for violating the ordinance. Choice (D) is incorrect because the lawyer’s liability would not depend on whether the construction company was negligent. In other words, even if the construction company was negligent and that was a cause of the accident, that would not necessarily preclude the lawyer from being heLd liable, as well.
How well did you know this?
1
Not at all
2
3
4
5
Perfectly
19
Q

A wife and her husband were having dinner at a restaurant when the wife excused herself to go to the bathroom. The restaurant was owned and operated by a chef. As the wife was walking past a table where another customer was seated, she slipped and fell on an egg roll that had been lying on the floor for quite some time, although the chef was unaware that it had fallen onto the floor. When she fell, her head struck a serving tray that was located in the aisle. The fall caused the wife to suffer a severe concussion. The customer knew that the egg roll was on the floor, and although he could have done so, he did not warn the wife.
If the wife asserts a claim against the chef for the injuries she suffered from the fall, she will most likely
(A) recover, because the egg roll on the floor constituted an unsafe condition of the premises.
(B) recover, because the egg roll was on the floor for a substantial period of time before the accident.
(C) not recover, because the chef did not know that the egg roll was on the floor.
(D) not recover, because the customer could have prevented the injury by warning the wife of the presences of the egg roll.

A
  1. (B) A person who possesses Land generally owes a duty to exercise reasonable care for the safety of invitees. An invitee is someone who enters land that is held open to the general public or who enters the land in connection with some potential business dealing that would benefit the person who possesses the land. The obligation to exercise reasonable care extends to everything that threatens the invitee with an unreasonable risk of harm—care against negligent activities, warning of known latent dangers, precautions against foreseeable dangers, and inspection of the premises to discover possible dangerous conditions that are not known. As a patron in a restaurant, the wife should be classified as an invitee. The chef thus owes a duty to exercise reasonable care for her safety. In this very tricky Torts question, students must choose the answer choice that is the most precisely correct and is neither over-inclusive nor under-inclusive. Choice (B) is correct because the chef has a duty to act reasonably in inspecting the premises, and that includes inspecting to discover an unsafe condition within a reasonable period of time. Therefore, because the egg roll had been on the floor for a substantial period of time, the chef would be liable. Choice (A) is incorrect because a business like a restaurant cannot be expected to know immediately about every unsafe condition, such as a piece of food on the floor. Choice (C) would be correct if the wife were merely a licensee, because then the property owner would merely have a duty to warn her about known dangers. She is an invitee, however, so the duty owed to her is broader and includes warning her about dangerous conditions the owner either knew or should have known about through reasonable inspection. Choice (D) is incorrect because an omission to act by a third person does not relieve a defendant of liability.
How well did you know this?
1
Not at all
2
3
4
5
Perfectly
20
Q

The head coach of the local high school football team was also employed as a physical education teacher at the high school. It was a late autunm afternoon, and the football team was practicing for its upcoming game against its archrival. While the team was practicing, the skies darkened as a storm approached. Minutes later, it began to rain heavily.
Even though the storm intensified, the coach did not stop the practice because he had no reason to believe the storm presented a danger to his players. Suddenly, a bolt of lightning struck and killed the team’s star player. This jurisdiction has abolished governmental immunity, and a school board may be sued as a private entity.
If the player’s parents bring a wrongful death action against the coach and the high school board, will they prevail?
(A) Yes, because the player was injured while engaged in a school activity.
(B) Yes, because under the circumstances the coach would be strictly liable.
(C) No, because the coach was not negligent by continuing the practice.
(D) No, because lightning is an act of God.

A
  1. (C) Although the player’s death was an accident, the coach (and the school board) may be held liable for the wrongful death if it were negligent for the coach to continue practice in spite of the storm. However, the facts state that the coach had “no reason to believe the storm presented a danger to his players,” and that the lightning came “suddenLy” out of what was otherwise a storm involving only rain. If he had no reason to know of the danger, he did not act unreasonably and, therefore, was not negligent. Choice (A) is incorrect; even though the player was, in fact, injured during a school activity, since the coach was not negligent, there would be no liability. Choice (B) is incorrect because the coach was not engaged in a type of abnormally dangerous activity that would give rise to strict liability. Those types of behavior would include such activities as blasting, storage/transportation of dangerous chemicals or explosives, or selling a defective product. Because the coach was simply coaching his team, strict liability will not attach. Choice (D) is incorrect for two reasons. First, an intervening act of God does not cut off liability unless it is unforeseeable, and there is no showing that lightning was unforeseeable in the area. Second, since the coach was not negligent, causation does not matter.
How well did you know this?
1
Not at all
2
3
4
5
Perfectly
21
Q
  1. A husband and wife were shopping at a local department store when the wife saw a dress that she especially liked. The wife, who weighed more than 300 pounds, was unable to find a size large enough to fit her. She then saw a store clerk and asked if the store carried the dress in her size. The clerk looked at the wife and said, “You look like a hippopotamus, and I’m sorry, but we don’t carry this dress in the hippo size.” Another customer in the store overheard the comment and began to laugh. The wife became very upset and hurried out of the store.
    If the wife asserts a tort action against the department store based upon the clerk’s actions, the wife will
    (A) win, because the statement was overheard by
    another customer.
    (B) win, because the clerk’s conduct was extreme and outrageous.
    (C) lose, because the clerk was merely statingan opinion.
    (D) lose, because the wife only suffered hurt feelings.
A
  1. (D) In order to recover for slander, the plaintiff must plead and prove special damages (i.e., pecuniary loss). To recover for infliction of emotional distress, the plaintiff must suffer severe emotional distress. Choice (D) is, therefore, correct because the wife would not have a claim if she suffered only hurt feelings. Choices (A) and (C) are incorrect because they relate to the possibility of the defendant being held liable for defamation, but the wife cannot recover on such a claim without sufficient damages. Likewise, choice (B) is incorrect because it suggests that the wife could recover for infLiction of emotional distress, but that would not be possible unless the wife suffered severe emotional distress.
How well did you know this?
1
Not at all
2
3
4
5
Perfectly
22
Q
  1. On Thanksgiving Day, a father was carving a turkey for his family when he seriously cut his hand. The knife severed an artery, causing the father to lose a lot of blood. The mother telephoned their family doctor, who instructed her to drive the father to the local hospital. The doctor indicated that he would meet them there.
    A short time later at the hospital, the doctor treated the father and bandaged his hand. However, due to the fact that the father had lost a considerable amount of blood, the doctor ordered an immediate blood transfusion. After determining that the father’s blood type was A-positive, the doctor orally instructed a nurse, who was employed by the hospital, to obtain the necessary blood for the transfusion. The nurse followed the doctor’s instructions and telephoned the hospital’s blood bank department and ordered a unit of A-positive blood.
    The person in the blood bank who received the nurse’s call correctly wrote down the order for A-positive blood. However, someone working at the blood bank, whose identity cannot be determined, made an error in filling the order and labeling the unit of blood for the father. As a result, the blood bank sent a unit of B-positive blood mislabeled as A-positive blood. After being administered the wrong blood, the father died.
    The hospital has a written rule in effect whereby all orders for blood transfusions must be made in writing by the physician in charge of the patient’s treatment. Although the doctor was aware of the hospital rule, he, like many other doctors at the hospital, frequently ordered blood transfusions by oral instructions.
    If the mother asserts a wrongful death action against the doctor, she will most likely
    (A) prevail, because the doctor would be responsible for the blood bank’s error in administering the wrong blood.
    (B) prevail, because the doctor did not follow the hospital’s rule regarding blood transfusion orders.
    (C) not prevail, because the doctor acted in the same fashion as many other doctors at the hospital.
    (D) not prevail, because the unidentified person in the blood bank who was responsible for sending the wrong blood type was not an employee of the doctor.
A
  1. (D) Under the doctrine of respondeat superior, a master is generaLLy hetd vicariously liable for aLl tortious conduct of his servant that is within the “scope of employment.” The blood bank employee (who mislabeled the blood) was not an employee or servant of the doctor. Therefore, the doctor would not be responsible for that negligence. Choice (A) is incorrect because the doctor will not be held liable because the doctrine of respondeat superior does not apply. Choice (B) is incorrect because the doctor’s failure to submit the blood transfusion order was not an actual cause or a “but for” cause of the patient’s death. The blood bank received the correct order (for A-positive blood) from the nurse. Despite having received the order correctLy, the blood bank then sent the wrong blood, which is an error that would have occurred regardless of whether the doctor submitted the order in writing or orally. The patient thus would have died because of the blood bank’s error, even if the doctor had submitted the order in writing and, therefore, his violation of the hospital rules was not a cause of the patient’s death. Choice (C) is incorrect because the fact that the doctor did what many other doctors had done would not necessarily shield him from liability. However, as stated above, the doctor was not the actual cause of the father’s injury, so it does not matter whether the doctor acted negligently.
How well did you know this?
1
Not at all
2
3
4
5
Perfectly
23
Q
  1. A patient was in the hospital to be treated for an illness. The patient’s doctor prescribed a mild anesthetic drug, to be given to the patient intravenously, to reduce the pain being experienced by the patient. The patient was supposed to receive a saline solution containing 18.5 milligrams of the drug. A nurse arranged to have a pharmacy technician prepare the solution and drug, and then the nurse administered the solution and drug to the patient. The solution that was administered to the patient contained 185 milligrams of the drug, rather than just 18.5. Shortly after receiving the drug, the patient had a heart attack and died. The doctor, nurse, and pharmacy technician were all employed by the hospital.
    If the patient’s family brings a wrongful death claim against the hospital, which of the following must the family prove in order to recover damages?
    (A) The family must identif’ which of the three hospital employees involved—the doctor, nurse, and pharmacy technician—was actually responsible for the mistake in the dosage of the drug.
    (B) The family must prove the specific negligent act that resulted in the wrong dose of the drug being administered.
    (C) The family must show that hospital administrators were negligent in hiring or supervising one or more of the employees involved.
    (D) The family must prove that the patient would not have died if given 18.5 milligrams of the drug.
A
  1. (D) The doctrines of respondeat superior and res ipso loquitur can be used together in this situation. Respondeat superior means that an employer will be vicariously Liable for torts committed by its employees within the scope of their employment. Res ipso Ioquitur means thata plaintiff can recover on a negligence claim, even if he or she is unable to prove exactly what was done negligently, if the plaintiff can show that the accident in which the plaintiff was injured is the sort of thing that ordinariLy does not occur unless there was negligence, and the agency or instrumentality that caused the injury was within the exclusive control of the defendant. In such a situation, res ipsa loquitur permits the jury to infer, based on the circumstantial evidence, that the defendant was negligent, even if the plaintiff cannot prove exactly what negligent act occurred. In this question, the hospital is responsible under respondeatsuperior for any torts of the doctor, nurse, or physician-technician. Even if the plainfiffs cannot prove exactly what happened that was negligent, the plaintiffs can rely on res ipsa loquitur because a patient ordinarily does not receive the wrong dose of medication unless someone was negligent. Choice (A) is incorrect because it is not necessary to show the identity of a particular employee of the hospital. There is an inference that it was an employee, and that is enough. Choice (B) is incorrect because res ipsa loquitur will allow the plaintiffs to recover through an inference of negligence even if they cannot determine exactly what negligent act occurred. Choice (C) is incorrect because the family can hold the hospital liable for the tortious conduct of one or more of the employees, even without proof that the hospital was negligent in hiring or supervising any of the employees. Choice (D) is correct because the plaintiffs do need to prove that the dosage error was an actual cause or a “but for.” cause of the patient’s death. In other words, they must prove that the patient would have lived if given the correct dose.
How well did you know this?
1
Not at all
2
3
4
5
Perfectly
24
Q

One fall weekend, an outdoorsman went on a camping trip to a remote wilderness area deep in the northern part of the state in which he lived. While on the trip, the outdoorsman went deer hunting. He was in a very desolate area, surrounded by a densely wooded terrain, that was located at least 100 miles from any habitation. After a rather uneventful morning with nothing at which to shoot, the outdoorsman suddenly spotted a bald eagle; a nearly extinct bird. The bald eagle was listed as an endangered species, and to shoot one in this state was a criminal offense.
Unable to resist the temptation, the outdoorsman took a shot at the bald eagle. The bullet missed the bald eagle but struck a hermit, who had moved to the woods a few months earlier to escape from the stresses of society. The hermit had been napping in a secluded area. The bullet hit the hermit in the eye and permanently blinded him. The outdoorsman was unaware of the hermit’s presence.
If the hermit asserts a claim against the outdoorsman to recover damages for his injury, the hermit will
(A) prevail, because his injury was caused by the
outdoorsman’s unlawful act.
(B) prevail, because firearms are dangerous instru
mentalities imposing strict liability on the user.
(C) not prevail, because the outdoorsman had no
reason to anticipate the presence of another
person in such a remote area.
(D) not prevail, because the outdoorsman did not
intend to shoot the hermit.

A
  1. (C) A popular Multistate testing area deals with proximate, or legal, cause. In order to be liable for negligence, the defendant’s conduct must constitute the legal or proximate cause of the plaintiffs harm or injury. The majority view is that a defendant’s duty of care is owed only to foreseeable plaintiffs (i.e., those individuals who are within the risk of harm created by the defendant’s unreasonable conduct). Choice (C) is the best answer because it addresses the fact that the hermit was an unforeseeable plaintiff to whom no duty of care was owed. Choice (A) is incorrect because the statute prohibiting people from shooting bald eagles was meant to protect the bald eagles, not to protect a person like the hermit, who was accidentally hit. Choice (B) is incorrect because there is no basis for imposing strict liability here. Moreover, even a strict liability claim would require that the plaintiff was a foreseeable person, and the hermit was not a foreseeable victim of the defendant’s conduct here. Choice (D) is incorrect because the outdoorsman’s lack of intent would prevent him from being held liable for intentional torts, but would not help him to avoid liability for negligence.
How well did you know this?
1
Not at all
2
3
4
5
Perfectly
25
Q
  1. A hunter sat quietly in a forest, hiding behind some brush and waiting for deer to pass by him. After hours of waiting and not spotting any deer, the hunter decided to give up and go home. Disappointed that he never even got a chance to take a shot at a deer, the hunter decided to fire a few practice shots before walking back to his car. He aimed his rifle at a small tree at the top of a nearby hill and fired three shots.
    At that moment, a state police officer happened to be passing nearby. The officer was part of a police team searching the forest for a fugitive who was believed to be armed and dangerous. When the police officer heard the gunshots, he thought he was being attacked. He quickly pulled his pistol out of his holster and fired it toward the brush from which he had seen the muzzle flash from the hunter’s rifle. One of the bullets fired by the police officer struck the hunter and seriously wounded him.
    If the hunter asserts a claim against the police officer to recover damages for his injury, the hunter will
    (A) prevail, because the hunter had no intent to harm the police officer and, therefore, the police officer was not entitled to fire back.
    (B) prevail, because the hunter had not committed any crime entitling the police officer to use force likely to cause death or serious bodily injury.
    (C) not prevail, if the police officer reasonably believed that he was under attack.
    (D) not prevail, because the police officer was not the original aggressor.
A
  1. (C) Self-defense is one of the defenses that can be used to avoid liability for battery or other intentional torts. The key requirement is reasonableness. The person who acts in self-defense must reasonably believe that he is in danger and must use no more than a reasonable amount or degree of force under the circumstances. Furthermore, a reasonable mistake as to the existence of the danger does not vitiate the defense. In other words, what matters is whether it reasonably appeared that self-defense was necessary, not whether it was actually necessary. Even though the police officer was mistaken when he thought he was under attack, he would have a valid defense if it reasonably appeared to him that he was in danger and needed to defend himself by shooting back at the attacker. Choice (A) is incorrect because even though the hunter did not intend to shoot anyone, the police officer still was entitled to act in self-defense if he reasonably believed he was under attack. Choice (B) is incorrect because if the police officer reasonably believed that he was in danger and needed to protect himself, he would be entitled to shoot, regardless of what crimes, if any, the attacker may have previously committed. Choice (D) is incorrect because even if the police officer was not the original aggressor, he would be entitled to use self-defense if he reasonably believed it was necessary to protect him from danger.
How well did you know this?
1
Not at all
2
3
4
5
Perfectly
26
Q
  1. A pregnant woman was sitting on her front porch watching her husband mow the lawn. A friend of the woman had known the woman for years and knew that she was an easily excitable person. The friend also hated the woman’s husband. Knowing that the woman was present, the friend drew a pistol and threatened to kill the husband. The woman suffered severe emotional distress as a result of witnessing this incident and soon afterward had a miscarriage.
    In an action by the woman against the friend for mental anguish resulting in her miscarriage, the woman will
    (A) lose, because the friend did not intend for the woman to suffer a miscarriage.
    (B) lose, because the friend’s actions were directed against the husband, not the woman.
    (C) win, because it was highly probable that the friend’s extreme and outrageous conduct would cause emotional distress to the woman.
    (D) win, because she is the husband’s wife.
A
  1. (C) A person is liable for intentional infliction of emotional distress if he does something that is extreme and outrageous and that intentionally or recklessly causes severe emotional distress to someone. In general, only the person who is the target of the extreme and outrageous conduct can recover. However, a plaintiff can recover if she was present when the defendant did something extreme and outrageous to the plaintiff’s immediate family member, and if the defendant knew or should have known that defendant was present and likely to suffer severe emotional distress from witnessing the event. Here, the woman suffered severe emotionaL distress because she was present when the friend did something extreme and outrageous to her husband. She, therefore, can hold the friend liable because the friend knew or should have known she would suffer such emotional distress, since he knew she was an easily excitable person. Choice (A) is incorrect because it is not essential that the defendant intended that the woman suffer a miscarriage, provided that he did know or have reason to know she would suffer severe emotional distress from witnessing the threat to her husband’s life. Choice (B) is incorrect because the woman could recover, even though she was not the target of the friend’s threat, through the exception for family members described above. Choice (D) is incorrect because it is not enough that the woman is the husband’s wife. Alt of the other requirements described above must be satisfied, including that the friend knew or should have known that the woman would suffer severe emotional distress. Choice (C) is a better answer than (D) because choice (C) provides the additional fact that the friend should have realized his extreme and outrageous conduct would cause emotional distress to the woman.
How well did you know this?
1
Not at all
2
3
4
5
Perfectly
27
Q
  1. A child was playing with a ball in the front yard of his family’s home. He accidentally threw the ball too far, and it hit a neighbor’s new car. The neighbor was enraged and threatened to kill the child. The child ran inside his house and told his mother what had occurred. Although the child was only mildly upset by the incident, the child’s mother was extremely distressed. She suffered an emotional breakdown and needed months of therapy to recover.
    If the mother sues the neighbor for the mental anguish suffered, she will
    (A) recover, even though she was not present when the threat was made.
    (B) recover, because she would not have suffered the emotional trauma had it not been for the neighbor’s threat to her child’s life.
    (C) not recover, because she was not present when the threat was made.
    (D) not recover, because the neighbor did not touch her child.
A
  1. (C) Where a defendant’s extreme and outrageous conduct is directed at a third person, rather than at the plaintiff herself, the plaintiff generally cannot recover for intentional infliction of emotional distress. An exception to that general rule applies where the plaintiff is present when the extreme and outrageous conduct occurs, the target of the extreme and outrageous act is an immediate family member of the plaintiff, and the defendant knows or should know the plaintiff is present and will suffer severe emotional distress from witnessing what happens to her family member. That exception does not apply here because the mother was not present when the threat was made to her child. Choice (A) is incorrect because in order for the mother to be successful, she will have to show that she was present at the time of the confrontation between the neighbor and her child. Choice (B) is incorrect because she cannot recover for her emotional distress, even if hearing about the neighbor’s conduct caused that distress, because she was not present when the threat to her child was made. Choice (D) is incorrect because if she had been present when the threat was made, she could be able to recover for intentional infliction of emotional distress, even though the neighbor did not touch her child.
How well did you know this?
1
Not at all
2
3
4
5
Perfectly
28
Q
  1. A prominent judge lived next door to a father. Recently, the judge had sentenced the father’s son to six months in prison on a narcotics charge. One afternoon while the judge was mowing his lawn, the father decided to avenge his son’s conviction. The father set up his water sprinkler behind some shrubbery separating their adjoining properties. As the judge was mowing his lawn and came within reach of the water sprinkler, the father turned on the sprinkling device. The judge did not see the water coming toward him. The water hit the judge in the back and drenched him.
    The judge would be able to recover against the father for which of the following tort or torts?
    (A) Negligence and battery.
    (B) Battery and assault.
    (C) Negligence and assault.
    (D) Battery and trespass to land.
A
  1. (D) A person is liable for battery if he does an act with the intent and result of causing harmful or offensive contact to the plaintiff. A person is liable for trespass to land if he does an act with the intent and result of causing someone or something to enter or remain on someone’s land. Because the father intended to douse the judge with water (from the sprinkling device), he would be liable for a battery. In addition, by directing water over the judge’s property, the father would be liable for trespass. Choice (A) is incorrect because negLigence is the wrong cause of action when the defendant acts intentionally. Choice (B) is incorrect because assault requires that someone was intentionally put in apprehension of an imminent harmful or offensive contact. Here, the judge did not see the water coming, so he was not put in apprehension of being hit by it. Choice (C) is incorrect because neither negligence nor assault is an appropriate claim here, for the reasons just given.
How well did you know this?
1
Not at all
2
3
4
5
Perfectly
29
Q
  1. Each year, the local boys’ and girls’ club sponsors a “Baseball Card Bonanza.” During the three- day event, collectors of baseball cards gather, exchanging and swapping old cards. A boy and a girl, both 12-year-old youngsters at the bonanza, were looking at each other’s cards when they were approached by a 32-year-old collector. The collector asked the boy and girl if he could look through their baseball cards. The boy and girl nodded affirmatively.
    As the collector was flipping through the boy’s cards, he found a 1948 card featuring Branch Rickey, the former owner of the Brooklyn Dodgers. The collector, who knew the card was worth more than $500, offered the boy 50 cents for the card. The boy thought the card had very little value because he knew that Rickey was not a major league baseball player. The boy accepted the 50 cents and gave the Rickey baseball card to the collector. The next day the boy found out that the baseball card was worth
    $600.
    If the boy asserts a claim against the collector for deceit, will the boy prevail?
    (A) Yes, because the collector did not disclose the true value of the card.
    (B) Yes, because the boy was the true owner of the card and was entitled to the benefit of the bargain.
    (C) No, because the collector made no false representations of fact.
    (D) No, because the boy was not justified in relying on the collector’s offer.
A
  1. (C) The elements of the tort action for deceit may be summarized as follows: (1) a false representation made by the defendant (usually this representation must be one of fact); (2) knowledge or belief on the part of the defendant that the representation is false; (3) an intention to induce the plaintiff to act or to refrain from action in reliance upon the misrepresentation; (4) justifiable reliance upon the representation on the part of the plaintiff; and (5) damage to the plaintiff, resulting from such reliance. In the present hypothetical, because the defendant did not make any false representation of fact, the correct answer is choice (C). Choice (A) is incorrect because non-disclosure of value does not create liability for deceit. Choice (B) is incorrect because “benefit of the bargain” speaks to the potential recovery of the plaintiff should the tort of deceit be established. Since the boy cannot establish that a deceit took place, he is not entitled to this remedy. Choice CD) is incorrect because justifiable reliance alone is not enough to show deceit. The plaintiff must prove that the defendant made a false representation of fact.
How well did you know this?
1
Not at all
2
3
4
5
Perfectly
30
Q

On Tuesday, a homeowner purchased a new riding lawnmower from a department store. The next day, he was cutting the grass in his back yard with the mower when it started to rain. He stopped the mower and turned off the motor switch. He went inside and decided to wait until the rain stopped before mowing the rest of the lawn. A few minutes later while the homeowner was inside his house, the lawnmower suddenly lurched forward, rolled down a hill, and entered the property of his neighbor. The mower cut the neighbor’s prize rosebushes, destroying them.
It was later determined that the lawnmower’s motor switch was defective. Even though the homeowner had turned the starter switch to the off position, the engine did not shut off. Because the mower was built with a new silent rotary engine, the homeowner had no reason to know that it was still running when he went into the house. The homeowner would not have been able to discover the defect by any feasible means of inspection.
If the neighbor asserts a claim against the homeowner for trespass, will the neighbor prevail?
(A) Yes, because the lawnmower entered onto the neighbor’s property.
(B) Yes, because the homeowner is strictly liable for the damage caused by the defective mower.
(C) No, because the homeowner was using the mower for its intended purpose.
(D) No, because the defective motor switch was not discoverable by reasonable inspection.

A
  1. (D) There are several theories through which a plaintiff could hold a defendant liable for something (like the defendant’s lawnmower) entering the plaintiff’s land. One is the intentional tort of trespass to land, which requires proof that the defendant intended for the thing to enter the plaintiffs land. Another would be a negligence claim if the defendant’s failure to exercise reasonable care caused the thingto enter the plaintiff’s land. The defendant also could be strictly liable if his abnormally dangerous or ultrahazardous activities caused the thing to enter the plaintiff’s land. However, if none of those theories applies, the defendant will not be liable. Here, the lawnmower’s entry onto the neighbor’s land was not the result of any intentional act, negligent conduct, or abnormally dangerous activity by the homeowner. Choice (A) is incorrect because the entry here was accidental and not the result of an intentiona’ tort, negligence, or abnormally dangerous activity. Choice (B) is incorrect because strict liability for defective products, under section 402A of the Restatement (Second) of Torts, applies only to those engaged in the business of selling the product that is defective. That type of strict liability, therefore, cannot apply to the homeowner, who is merely an owner and user of the mower, not a seller. Note that “abnormaLly dangerous activity” type of strict liability is also not present here. Choice (C) is incorrect because the fact that the homeownerwas using the mower for its intended purpose is not the crucial consideration. For example, if he was negligent, the fact that he was using the mower for its intended purpose would not permit him to avoid liability. He is not liable because no intentional tort, negligence, or basis for strict liability is present, not because he was using the mower for its intended purpose.
How well did you know this?
1
Not at all
2
3
4
5
Perfectly
31
Q
  1. A football player was the star fulllack for the local college football team. After missing two practices, the football player was dropped from the team by the head football coach. Following his dismissal, the football player met with the coach and asked if he could rejoin the team. The coach said that the football player was despised by the other players and under no circumstances could he return to the team. As the football player was leaving the coach’s office, feeling very dejected, the coach then said to him, “Hope you decide to transfer, because everybody hates your guts around here.”
    Later that same evening, the football player wrote a suicide note in which he stated, “The coach is responsible for my despondency. If I can’t play football, I don’t want to live.” After swallowing a bottle of Quaalude barbiturates, the football player fell unconscious in his dormitory room. Moments later, the football player’s roommate entered the room and saw his limp body on the floor. The roommate read the suicide note and then attempted to administer aid. Failing to revive him, the roommate picked up the football player and carried him to the college’s first aid center. The football player received prompt medical attention and soon recovered from his drug overdose.
    If the football player asserts a claim against the coach based on intentional infliction of emotional distress, the football player will most likely
    (A) prevail, because the coach intended to cause him to suffer emotional distress.
    (B) prevail, because the coach’s remark did, in fact, cause the football player to suffer emotional distress.
    (C) not prevail, because the football player’s drug overdose resulted from his own voluntary act.
    (D) not prevail, because the coach acted reasonably under the circumstances, since everyone on the team hated the football player.
A
  1. (C) This example illustrates the major problem most students encounter on the Multistate—choosing between two conceivably correct answers. Choices (A) and (B) are incorrect because courts impose liability for intentional infliction of emotional distress only where the defendant’s conduct has been extreme and outrageous. That is a stringent requirement, demanding that the conduct really was beyond all possible bounds of decency so as to be regarded as atrocious. The coach’s conduct was unkind, but not that extreme and outrageous. Choice (D) is incorrect because it speaks to a reasonableness standard, which is a negligence concern. Since the suit is for intentional infliction of emotional distress, negligence is not in an issue. Consequently, the correct answer is choice (C) because the required outrageousness is not present here.
How well did you know this?
1
Not at all
2
3
4
5
Perfectly
32
Q
  1. A stockbroker became despondent because the economy was terrible and all of his clients had lost substantial amounts of money on their investments. The stockbroker decided to end his life by jumping off a bridge. He walked to the middle of the bridge, climbed over the railing, stood on the tiny ledge on the edge of the bridge, and prepared to jump. A taxi driver who happened to be crossing the bridge at that moment saw what the stockbroker was doing. The taxi driver slammed on his brakes, jumped out of his taxi, started running toward the stockbroker, and yelled, “Don’t do it!” Pleased to hear that someone cared about him, the stockbroker changed his mind about committing suicide and decided that he wanted to live. At that moment, however, the stockbroker started to lose his balance, and his feet began to slip off the small, grimy ledge of the bridge. Just as the stockbroker started to fall, the taxi driver reached him and grabbed the back of his jacket. The taxi driver hung on tightly to the jacket and nearly was pulled over the side of the bridge, but he managed to hang on. Using every bit of strength he could muster, the taxi driver pulled the stockbroker back up and onto the bridge. The stockbroker was relieved to have been saved from falling. Unfortunately, the taxi driver suffered a severe injury to his back from the strain of hanging on to the stockbroker and pulling him back up onto the bridge. A statute in the jurisdiction makes attempted suicide a crime.
    If the taxi driver asserts a claim against the stockbroker to recover damages for his injuries, will he prevail?
    (A) No, because the stockbroker did not intend to harm anyone other than himself.
    (B) No, because attempted suicide was a criminal offense in the jurisdiction.
    (C) Yes, because the stockbroker put himself in a position of peril.
    (D) Yes, because he succeeded in saving the stockbroker’s life.
A
  1. (C) Under the “rescue doctrine,” the fact that someone will try to come to the rescue when others are in danger is regarded as a foreseeable occurrence. As Judge Cardozo put it, “danger invites rescue” [Wagner v. International Railway, 232 N.Y. 176, 180 (1921)1. A rescuer who is injured during the rescue attempt, therefore, can recover against the person whose negligence created the need for someone to come to the rescue. This doctrine applies even when the defendant endangers no one’s safety but his own. In this question, the stockbroker put himself in a position of periL and can be held liable for the rescuer’s injuries. Choice (A) is incorrect because it does not matterwhetherthe defendant intended to hurt himself oranyone else. Intent is not necessary. The fact that the plaintiff endangered someone (himself or another person) is enough to create the need for the rescue and to put the rescuer at risk. Choice (B) is incorrect because the rescue doctrine applies regardless of whether the defendant’s conduct is a criminal offense. Choice (D) is incorrect because saving the defendant’s life is not an element required for the plaintiff to assert a claim based on the rescue doctrine.
How well did you know this?
1
Not at all
2
3
4
5
Perfectly
33
Q
  1. On Thursday morning, two plainclothes police officers were shot to death while trying to arrest a bank robber who had just robbed a bank. Following the killings, the police issued an “all-points-bulletin” for the arrest of a 40-year-old Caucasian male (height, 6 feet; weight, 150-155 pounds), who had been seen near the location of the bank robbery.
    On Friday, a security guard at a local department store was walking down Main Street when he noticed a tall, African American man who fit the bank robbery suspect’s description in age, height, and weight. The security guard approached the person (and believing him to be the bank robbery suspect), pulled a gun and arrested him. The security guard held the man in custody until the police arrived a short time later. The police officer informed the security guard that he had arrested the wrong person. The man had told the security guard that he was not the man who had committed the bank robbery and shot the police officers. The man was humiliated by the false arrest.
    If the man asserts a claim for assault against the security guard, he will
    (A) succeed, if the man saw the security guard pointing the gun at him.
    (B) succeed, if the security guard’s mistaken belief was unreasonable.
    (C) not succeed, because the security guard did not intend to injure the man.
    (D) not succeed, because the man did not suffer any injury from the security guard’s act.
A
  1. (A) As a general rule, either a Law enforcement officer or a private citizen may arrest without a warrant to prevent a felony or breach of the peace that is being committed—or reasonably appears about to be committed—in his presence. Once the crime has been committed, the private person may still arrest, but his authority depends upon the fact that the person actually did commit the crime, and he must take the full risk of a mistaken arrest. The killing of the police officers did not occur in the security guard’s presence. As a result, he must take the full risk for falsely arresting the man. Therefore, by pointing the pistol at the man, the security guard would be liable for assault. Choice (B) is incorrect because the reasonableness of the security guard’s belief merely goes to the issue of damages and not to whether Liability exists for the tort of assault. Choice (C) is incorrect because even if the security guard had hoped not to have to shoot, he did intend to put the man in apprehension of being injured. Choice (D) is incorrect because injury is not required for assault. It is enough that the plaintiff is placed in fear or apprehension of being injured, as the man was here.
How well did you know this?
1
Not at all
2
3
4
5
Perfectly
34
Q
  1. A man and a brother were identical twins. The man was having an affair with a woman. The woman’s husband was aware of his wife’s relationship with the man. One morning the husband saw the brother, whom he mistakenly believed was the man. The husband walked toward the brother and said, “I’m going to punch you in the face, you rat.” The brother immediately realized that the husband had mistaken him for his brother. The brother had enough time to inform the husband of his mistake but didn’t say anything. When the husband came closer, the brother punched him in the mouth.
    If the husband brings suit against the brother for battery, he should
    (A) prevail, because the brother did not inform him of his mistake.
    (B) prevail, because the husband’s threat was accompanied by an overt act when the husband approached the brother.
    (C) not prevail, because the husband was the initial aggressor.
    (D) not prevail, because the brother used reasonable force given the threat.
A
  1. (A) The Torts questions on the MBE are becoming exceedingly more difficult. In prior years, Torts was a relatively easy subject area. This is no longer the case. Here, choice (D) is an attractive answer because this initially seems to be just a simple self-defense question. To be sure, a person is privileged to use reasonable force to protect himself or herself from unlawful bodily harm or injury. However, this question has a subtle “twist” involving mistake. If a person is about to be attacked, but knows the attack is based on a mistake as to the person’s identify, the person is not privileged to await the attack and then use force to defend himself if he has time to correct the mistake and prevent the attack. In other words, the husband thought he was attacking the man; the brother knew that, and the brother had a chance to correct the husband’s mistaken belief; yet the brother failed to do so. As a result, the brother is not privileged to assert self-defense under these circumstances. Choice (B) is incorrect because the brother does not have the privilege to use self-defense here, regardless of whether the husband’s threat was accompanied by some overt act. Choice (C) is incorrect because the brother had time to inform the husband of his mistake, even though the husband was the initial aggressor. Choice (D) is incorrect because the brother does not have the privilege to use self-defense here because he had time to inform the aggressor of his mistake as to identity and failed to do so.
How well did you know this?
1
Not at all
2
3
4
5
Perfectly
35
Q
  1. A retired cattle rancher was flying his private plane to visit his daughter and grandchildren. While carefully and skillfully operating his airplane, he suddenly realized that one of his engines was gone. He was then forced to make an emergency landing under the reasonable belief that it was necessary to do so for the protection of himself and his airplane. He landed in a farmer’s orange grove and managed to steer the plane and land it between the trees in order to not cause damage to any of them.
    If the farmer brings suit against the rancher for trespass, the most likely result is that the farmer will
    (A) recover, because the property belonged to a private person.
    (B) recover, because the rancher had reasonable grounds to believe that the orange grove belonged to a private person.
    (C) not recover, because the rancher was privileged to enter the land of another under the circumstances.
    (D) not recover, because the rancher was carefully and skillfully operating his airplane when the engine went out.
A
  1. (C) Where a person acts for the protection of himself or his property, he is privileged to enter another’s property to prevent a threatened harm from taking effect. Here, the pilot was privileged to land his plane in the orange grove in order to protect himself and his airplane. Note that if the pilot had caused harm to the property, such as by damaging some of the orange trees, the pilot would be liable for that harm. Choice (A) is incorrect because the fact that the property belonged to a private person does not take away the privilege to trespass to avoid harm. Choice (B) is incorrect because, again, the fact that the orange grove belongs to a private person will not remove the privilege to enter the land to avoid harm. Choice (D) is incorrect because the fact that the pilot was operating the plane skillfully and carefully would not save him from liability for trespass. His privilege to enter the land is based on the necessity of saving himself and his airplane, not the fact that he was flying the plane carefully.
How well did you know this?
1
Not at all
2
3
4
5
Perfectly
36
Q
  1. A hiker was enjoying a journey through a large forest when he was caught in an unexpected blizzard. As it grew dark and the snow continued, he realized he was completely lost. He felt sure that he would freeze to death during the night. When all hope seemed lost, he stumbled across a small cabin. He knocked on the door, but no one answered. He tried to open the door, realized it was not locked, and went inside. In the cabin, he found some food to eat and some blankets that kept him warm through the night. By the next morning, the snow had stopped, and he was able to make his way out of the forest.
    If the owner of the cabin sues the hiker, seeking to recover damages for the cost of the food and for the use of his cabin, what is the most likely outcome of the case?
    (A) The hiker does not owe anything because the importance of his survival outweighs the property interests of the cabin owner.
    (B) The hiker must pay for the cost of the food and the rental value of the cabin for one night.
    (C) The hiker must pay for the use of the cabin, but nothing else.
    (D) The hiker must pay for the cost of the food, but nothing else.
A
  1. (D) The privilege of private necessity is a defense to intentional tort claims. It applies where a person needs to enter or use another person’s property in order to avoid a threatened harm to himself or his own property. For example, if you are about to drown in a flood, you can use a person’s boat to save yourself. While you will not be liable for trespass merely for entering or using the other person’s property, you will be obligated to pay for any actual harm that you cause to it. The hiker acted out of private necessity, to save himself from dying in the blizzard. He, therefore, is not liable merely for entering and using the cabin. He is required, however, to pay for the actual harm caused, so he must pay for the food that he ate. Choices (A), (B), and (C) are incorrect because the hiker will be liable for the cost of the food, but nothing else.
How well did you know this?
1
Not at all
2
3
4
5
Perfectly
37
Q
37. A 26-year-old graduate student knew that a classmate was a staunch supporter of animal rights. As they were leaving class one afternoon, the graduate student walked beside the classmate and chanted, “Animal experimentation is great.” The classmate angrily confronted the graduate student and said, “Let’s settle this outside in a fist fight.” The graduate student replied, “You’re on.”
The two students went outside and “squared off.” The graduate student threw the first punch and hit the classmate in the face with his bare fist. Unknown to the graduate student, the classmate placed a set of brass knuckles on his fist and hit the graduate student in the face with them. The force of the blow broke the graduate student’s nose. It was later determined that the use of the brass knuckles inflicted exactly the same damage as if the classmate had hit the graduate student with his bare fist.
The best argument for rejecting the defense of consent in an action by the graduate student for either assault or battery is that
(A) the classmate’s use of the brass knuckles exceeded consent under the circumstances.
(B) the classmate was only acting in self-defense.
(C) the graduate student’s insults provoked the classmate’s use of excessive force.
(D) the classmate’s punch would have inflicted the same harm with or without the use of the brass knuckles.
A
  1. (A) Consent is generally a defense to intentional tort claims. Consent is to the defendant’s conduct, rather than to its consequences. Thus, if the plaintiff willingly engages in a boxing match, he does not consent to be killed, although he does consent to the defendant hitting him if he can and, consequently, if death unexpectedly results, his consent to the act will defeat any action for the invasion of his interests. However, a plaintiff who agrees to a fistfight does not consent to being hit with brass knuckles, which is the same invasion by an act of a different character. In sum, a defendant’s privilege is limited to the conduct to which the plaintiff consents. The graduate student’s best argument, therefore, is that the use of brass knuckles exceeded the scope of the consent in this situation. Choice (B) is incorrect because self-defense is not applicable when the issue is consent that has been exceeded. Choice (C) is incorrect because words alone cannot justify exceeding the scope of consent and using excessive force. Choice (D) is incorrect because it was the use of the brass knuckles that is the issue, not the extent of injury that occurred or would have occurred without them.
How well did you know this?
1
Not at all
2
3
4
5
Perfectly
38
Q
  1. A center and a quarterback were 14-year-old eighth graders at the local high school. They were both members of the school’s junior varsity football team. The center weighed 170 pounds and was stocky in build. The quarterback was lanky and weighed about 145 pounds. The center and the quarterback often engaged in friendly tests of strength, such as arm wrestling and weight lifting contests, to see who was stronger.
    One afternoon, both youngsters were in the school gymnasium getting ready for football practice. As they were putting on their football uniforms, the center turned to the quarterback and said, “Hey, weakling, why don’t you punch me in the chest with your best shot?” The quarterback replied, “No, I’m afraid I might hurt you.” The center then said, “Are you kidding? You can’t hurt me. C’mon, give me your best shot.” The center stood up, stuck out his muscular chest, and prepared to have the quarterback hit him. The quarterback proceeded to cock his fist and then punched the center in the chest as hard as he could. Immediately thereafter, the center slumped to the floor, gasping for air. Realizing that the center was seriously hurt, the quarterback tried to render assistance but to no avail.
    Seconds later the center stopped breathing and died. Unknown to either the center or the quarterback, the center had a defective heart and suffered a heart attack as a result of the blow to the chest.
    In a wrongful death action, the quarterback will likely
    (A) be held responsible, because he committed a battery by hitting the center in the chest.
    (B) be held responsible, because the center’s consent was ineffective.
    (C) not be held responsible, because the center’s consent was effective.
    (D) not be held responsible, because the quarterback delivered the punch with greater strength than the center anticipated.
A
  1. (C) The consent of the person injured will ordinarily bar liability from being imposed for intentional interference with the person or property. Because the center consented to being punched in the chest, the quarterback is not liable for battery. Choice (A) is incorrect because a battery can only exist where the plaintiff does not give permission or consent to be touched. Here, the plaintiff consented to the touching. Choice (B) is incorrect because the center’s consent was effective. Even though the center was 14 years old, a minor may consent. For example, a 14-year-old boy who plays a tackle football game consents to physical contact, and teenagers may consent to engage in fistfights or similar physical encounters. Choice CD) is incorrect because the consent is to the plaintiff’s conduct, not the consequences. In other words, what matters is that the center consented to be punched in the chest, not whether he could anticipate the extent of the harm that would result.
How well did you know this?
1
Not at all
2
3
4
5
Perfectly
39
Q
  1. An owner of an apartment building in a rundown section of town knew that one of his tenants was dealing drugs from his apartment. He also knew that the tenant’s actions had led to some violent confrontations in the past, including one where a gun was discharged in the hallway outside the tenant’s apartment. Because the tenant regularly paid his rent on time, the owner didn’t take any action to try to evict the tenant. One day, a rival drug dealer drove by the tenant’s apartment and fired his gun at the tenant’s front window. One of the bullets passed through the tenant’s living room and went through the wall into the apartment next door. The bullet struck and injured a neighbor who occupied the adjoining apartment.
    If the neighbor asserts a claim against the owner to recover damages for his injury, he should
    (A) recover, because the owner knew that the tenant was dealing drugs from his apartment.
    (B) recover, because the owner was aware that such danger existed because of the tenant’s drug-dealing activities.
    (C) not recover, because the owner is not liable for the criminal acts of third persons.
    (D) not recover, because the neighbor is outside the zone of foreseeable danger.
A
  1. (B) As a general rule, a person is not held liable for failing to control the conduct of a third person unless a special relationship exists between the parties. Relationships that may be treated as special for this purpose include: (1) innkeeper and guest; (2) common carrier and passenger; (3) school and pupil; (4) landlord and tenant; (5) jaiLer and prisoner; and (6) employer and employee. In this regard, a landlord has the responsibility to act for the protection of his tenants and will be held liable if his failure to exercise reasonable care increases the risk of harm to the plaintiff through the criminal ortortious acts of others. As a result, the owner will be subject to liability if he was aware that such a. danger existed on account of his tenant’s drug-dealing activities. Choice (A) is incorrect because it does not provide the best statement of the reason for the special relationship to exist between an owner and his tenants. The fact that drugs were being dealt from the apartment may not alone subject the owner to liability. However, since the owner was aware that dealing drugs from the apartment created a danger, the owner has the responsibility to act for the protection of his tenants. For that reason, Choice (B) is the best answer. Choice (C) is incorrect because a special relationship exists, imposing a duty on the owner to act in preventing harm to the plaintiff through the criminal or tortious conduct ofotherthird persons. Choice (D) is incorrect because if the owner is aware that such danger exists, he has a duty to prevent that danger from harming his tenants.
How well did you know this?
1
Not at all
2
3
4
5
Perfectly
40
Q
  1. A property owner owned a 30-acre tract of land located on the outskirts of a community. After prosperity burst upon the community, the property owner subdivided the property into 30 lots and built townhouses on each of the lots. After selling all 30 of the lots, the various homeowners formed a homeowners’ association. Thereafter, the homeowners’ association adopted a Declaration of Covenants, which provided, inter alia, that each homeowner was required to pay an annual assessment charge to be used “for the promotion of the health, safety, and welfare of residents within the subdivision and for the enhancement of education, social life, and community welfare.”
    The homeowners’ association then properly and legally assessed each homeowner an assessment charge of $750. An owner of a townhouse in the subdivision refused to pay the assessment. The townhouse owner was experiencing financial problems and claimed that she couldn’t afford to pay the $750. As a result, the president of the homeowners’ association sent the townhouse owner the following letter:
    “Article 1 of the Declaration of Covenants requires all homeowners to pay their annual assessment charge on the first day of January for the said year. Because of your failure to make this payment, the homeowners’ association shall have the right to collect the amount due by action of law. Your prompt attention to this matter is greatly appreciated.”
    Three weeks elapsed, and the president of the homeowners’ association did not receive any response from the townhouse owner. He then wrote her a second letter that stated:
    “Before it institutes legal action, this is the homeowners’ association’s final demand for your $750 assessment charge. It is unfortunate that deadbeats like you ruin our community.”
    A copy of this letter was mailed to the other
    29 residents of the development. The president
    even sent a copy of the letter to the editor of the
    county newspaper, which had a circulation of
    20,000 subscribers. In its next edition, the county
    newspaper published the president’s letter on its
    editorial page.
    If the townhouse owner sues the president for defamation, she will probably
    (A) not prevail, because the president was acting to protect a legitimate public interest.
    (B) not prevail, because the statement involves only an expression of opinion.
    (C) prevail, because the president obviously acted with malice by sending the letter to the newspaper.
    (D) prevail, because the language of the president’s letter would be highly offensive to reasonable persons.
A
  1. (B) The tort of defamation requires the plaintiff to prove that the defendant intentionally communicated (i.e., published) a defamatory matterto some third person, who reasonably understands that the plaintiff’s reputation is lowered in the estimation of at least a substantial minority of the community, or such that third persons are deterred from associatingwith the person to whom the matter refers. Therefore, only statements of fact can be actionable as being defamatory. The townhouse owner is suing the president for calling her a “deadbeat.” She will not prevail, because this is simply a statement of the president’s opinion of the townhouse owner. Choice (A) is incorrect because even though the common law provides a conditional privilege for fair comment on matters of general public interest, the president’s letter regarding failure of one single homeowner to pay annual maintenance dues for her townhouse does not involve a matter of such public concern as to reasonably permit publication to 20,000 newspaper subscribers. Choice (C) is incorrect for failing to address the more fundamental requirement that only statements of fact are actionable, regardless of malice. Choice (D) is incorrect because even if the president’s letterwas highly offensive to reasonable persons, it would not be defamatory, since it was merely a statement of opinion.
How well did you know this?
1
Not at all
2
3
4
5
Perfectly
41
Q
  1. A manufacturer of snowmobiles produced one model, Model IA, which was a very popular snowmobile. On each of the Model 1 A snowmobiles produced, the manufacturer installed a speed-regulating device that prevented the snowmobiles from exceeding 50 m.p.h. The manufacturer distributed the snowmobile through wholesalers and retailers across the country. In its sales brochures and advertising materials, the manufacturer advises all of its distributors against making any modifications on snowmobiles without first conferring with the manufacturer’s staff of engineers.
    The operator of a shop in a mountain community had the exclusive right to distribute Model 1A snowmobiles in the area. Amid complaints from previous buyers regarding Model lA’s speed limitations, the operator removed the speed regulators from all the Model 1A snowmobiles in stock.
    A buyer purchased from the operator one of the Model 1A snowmobiles that had the speed regulator removed. One day, the buyer was driving the Model 1A snowmobile through a snow-covered field, at speeds in excess of 70 m.p.h. The snowmobile hit a snow-covered rock, causing the buyer to lose control of the snowmobile, crash into a tree, and suffer serious injuries.
    If the buyer institutes a strict liability in tort action, he will most likely recover against
    (A) the manufacturer only.
    (B) the manufacturer, even though it was unaware of the removal of the speed regulator.
    (C) the operator only.
    (D) both the manufacturer and the operator.
A
  1. (C) Under Section 402A of the Restatement (Second) of Torts, a defendant engaged in the business of selling a product may be held strictly liable for harm caused by the product being in a defective and unreasonably dangerous condition. This strict liability can apply to a seller at any level of the chain of distribution of the product. In other words, it can be imposed on a manufacturer, a wholesale distributor, and a retail dealer. A defendant will only be strictly liable, however, if the defect existed at the time the defendant sold the product. For example, a manufacturer will be strictly liable only for defects that existed when the product left its hands and not for defects created later by a downstream seller altering or mishandling the product. In the present case, the alleged defect is the lack of speed regulators. That defect existed when the operator sold the product, but not when the manufacturer sold the product, so only the operator can be held strictly liable. Choice (A) is, therefore, incorrect. Choice (B) is incorrect because regardless of whether the manufacturer knew about the regulators being removed, the product was not defective when sold by the manufacturer and, therefore, the manufacturer is not strictly liable. Note that another cause of action, such as negligence, might succeed if the alteration of the product was foreseeable and a manufacturer exercising reasonable care would have taken steps to prevent the alteration from occurring. This question, however, refers only to strict liability in tort. Choice (D) is incorrect because the manufacturer would not be liable because the snowmobile was not defective until the operator removed the speed regulators.
How well did you know this?
1
Not at all
2
3
4
5
Perfectly
42
Q
  1. A buyer purchased a shovel at a local hardware store. The shovel had a metal blade and a wooden handle. While the buyer was using the shovel to dig out a tree stump in his back yard, the handle broke in half, causing the buyer to fall and injure his back.
    The buyer brought a negligence claim against the manufacturer of the shovel. He proved that the shovel broke because of a manufacturing defect that made the handle too weak and that the manufacturer acted unreasonably in allowing this to happen. While conceding that the handle had a defect that caused it to break, the manufacturer established that if the buyer had been using the shovel carefully, he would not have fallen down and would not have been injured when the shovel broke.
    Assume that the case is governed by the law of a jurisdiction that follows traditional contributory negligence rules.
    Who is most likely to prevail?
    (A) The buyer, because the manufacturer conceded that the shovel was defective.
    (B) The buyer, because the manufactuter failed to exercise reasonable care in the production and inspection of its product.
    (C) The manufacturer, because the manufacturer did not guarantee that the shovel would never break.
    (D) The manufacturer, because the buyer’s negligence in the way he used the shovel was one of the causes of his injury.
A
  1. (D) Traditionally, contributory negligence was a complete bar to a negligence claim. In other words, if a plaintiffs injury was caused both by a defendant’s negligence and also by the plaintiff’s own failure to act reasonably, the plaintiff would not be able to recover any damages. However, while contributory negligence was a defense to a negligence claim, it was not a defense to a strict liability in tort claim against the manufacturer or seller of a defective product. The traditional rule of contributory negligence has been replaced in most states by comparative fault, under which the jury has the task of allocating percentages of fault to each party. Comparative fault means that a plaintiff will be able to recover despite being negligent, although the plaintiff’s negligence may reduce the damages awarded. Comparative fault is a defense to negligence claims and strict liability claims concerning defective products. On the Multistate Bar Exam, you are instructed to assume that comparative fault applies unless the question states otherwise. This question specifies that traditional contributory negligence rules apply. Under those rules, contributory negligence would be a complete defense to a negligence claim. This question says that the buyer is bringing a negligence claim, and so the manufacturerwill prevail if there was contributory negligence. Choice (A) is incorrect because the buyer is suing for negligence. If he were bringing a strict liability in tort claim instead of a negligence claim, then choice (A) would be correct. Choice (B) is incorrect because the buyer’s negligence claim is barred by the defense of contributory negligence. Choice (C) is incorrect because it does not matter whether the manufacturer guaranteed that the shovel would not break. The manufacturer could be liable for negligence or selling a defective product even if it never gave any guarantees.
How well did you know this?
1
Not at all
2
3
4
5
Perfectly
43
Q
  1. After winning a big antitrust case, an attorney and a few associates decided to celebrate and have a few drinks at a popular downtown watering hole. After having two gimlets (a cocktail containing vodka and lime juice), the attorney left his friends and drove home.
    The attorney, who was a bit tipsy, began driving in an erratic and reckless manner. He was traveling at an excessive speed through a residential section of town when he approached a sharp curve in the roadway. Trying to negotiate the turn, the attorney lost control of his vehicle and veered off the road, landing on the front lawn of a woman’s property.
    If the woman asserts a claim against the attorney for intentional trespass, she will most likely
    (A) prevail, because the attorney was operating his car recklessly.
    (B) prevail, because the attorney entered onto her property.
    (C) not prevail, because the attorney did not damage her land.
    (D) not prevail, because the attorney did not intentionally enter onto her property.
A
  1. (D) A person is subject to liability for intentional trespass if he intentionally enters land possessed by another person, regardless of whether he causes any actual harm to the Land. Based upon the given facts, the attorney did not intentionally drive his vehicle onto the woman’s property. Rather, he Lost control of his vehicle whUe trying to negotiate a sharp curve in the highway. The woman could try to sue for negligence because the attorney was operating his car in a reckless manner, but the attorney would be liable for negligence only if he caused some damage to the land or other actual harm. The woman, therefore, cannot prevail on an intentional trespass claim without proof of intent, so the woman will not be able to hold the attorney liable for this incident. Choice (A) is incorrect because the fact that the attorney was driving recklessly is not sufficient to generate liability for intentional trespass. Note that the attorney would be treated as having intentionally entered the woman’s property if the attorney had acted with knowledge that he was substantially certain to wind up on the woman’s property. Although the attorney was reckless, he did not have the knowledge of a substantial certainty that would make him liable for an intentional trespass. Choice (B) is incorrect because even though the attorney entered the woman’s property, he did not do so intentionally. Choice (C) is incorrect because damages are not a required element of an intentional trespass.
How well did you know this?
1
Not at all
2
3
4
5
Perfectly
44
Q
  1. A man owned a beautiful tract of land in a mountain resort. The property was purchased by the man and used as a family vacation retreat. About 200 yards of the man’s property bordered along a lake shoreline. A fisherman lived on a stream that flowed along one boundary of the man’s land and ran into the lake. When the man acquired ownership of the property, he had a channel dredged across his land from the stream to the lake at a point some distance from the mouth of the stream. The fisherman erroneously believed that the channel was a public waterway. Because the channel served as a convenient shortcut to the lake, the fisherman made frequent trips across the channel in his fishing boat. In no way did the fisherman’s use of the channel cause any harm or damage to the man’s property.
    After the man learned about the fisherman’s use of the channel, he requested that the fisherman desist further entry onto the waterway. The fisherman, who until that time was unaware of the man’s ownership claim, agreed. Nonetheless, the man brought suit against the fisherman to recover damages for trespass.
    Judgment is likely to be for whom?
    (A) The fisherman, because when he used the channel he believed that it was a public waterway.
    (B) The fisherman, because he caused no damage to the man’s land.
    (C) The man, but recovery is limited to nominal damages for the fisherman’s intentional use of the channel.
    (D) The man, because the fisherman should have known that the waterway was not open for public use.
A
  1. (C) Mistake is no defense to a claim for the intentional tort of trespass to land. A person is liable for trespass if he enters another person’s land because of mistaken belief that it is his own, even if the mistake is completely understandable and reasonable. In other words, trespass requires only that the defendant intended to enter the piece of land in question, not that the defendant intended to enter a place that the defendant believed was someone else’s land. Here, the fisherman erroneously believed the channel was a public waterway when, in fact, it was owned by the man. Even if this was a reasonable mistake made in good faith, the fisherman is still liable fortrespass, although the damages will be nominal because no real harm was caused. Choice (A) is incorrect because a mistaken understanding about who owned the property in question is no defense for an intentional trespass. Choice (B) is incorrect because damages is not a required element of an intentional trespass. Choice (D) is incorrect because the fisherman’s knowledge is irrelevant to his liability for trespass. As stated above, mistake is no defense to an intentional trespass, so the fisherman is liable even if he should not have known that the waterway was not open to the public.
How well did you know this?
1
Not at all
2
3
4
5
Perfectly
45
Q
  1. A pharmacist was employed by a drug store. A long-time customer of the drug store presented a prescription to the pharmacist for the medication Clinoril, which was prescribed by her physician to treat her arthritic condition. The pharmacist was in a rush that day because of a backlog of prescriptions to fill, and he misread the customer’s prescription. He filled her prescription with Clinitest, a caustic substance not for internal use. He labeled the container with the instructions for Clinoril: “Take one tablet twice a day.”
    The customer followed the instructions, consumed the Clinitest, and suffered severe injuries to her stomach and esophagus.
    If the customer brings a strict liability in tort action against the pharmacist, what is his best defense?
    (A) It was the cashier and not he who personally received the money for the medication.
    (B) He was not a seller of the product upon whom strict liability may be imposed.
    (C) He exercised reasonable care under the circumstances.
    (D) The drug store was not the manufacturer upon whom ultimate liability falls.
A
  1. (B) Section 402A of the Restatement (Second) of Torts provides for strict liability to be imposed on a person who sells a defective product. It can apply to a seller at any stage of the chain of distribution of the product, such as the manufacturer, wholesale distributor, or retail dealer. It only applies, however, to someone who is engaged in the business of selling the product. For example, while strict liability would be imposed on the company that manufactured and sold a product, it would not be imposed on an employee of the manufacturing company who merely worked in the factory and put together the product. Likewise, a retail store can be held strictly liable for selling a defective product, but strict liability would not be imposed on the employees of the store. In this question, the pharmacist’s best defense would be that he was merely an employee of a company that sold the product, but was not himself the seller. Choice (A) is incorrect because the fact that the pharmacist did not personally receive the money from the customer is not crucial. The cashier did receive the money, but the cashier is also just an employee and is not subject to strict liability. Choice (C) is incorrect because whether the defendant exercised reasonable care is irrelevant in a strict liability action. It is significant for a negligence claim, but not for strict liability. Choice (D) is incorrect because other sellers, as well as manufacturers, are held strictly liable for defective products that cause harm.
How well did you know this?
1
Not at all
2
3
4
5
Perfectly
46
Q
  1. A woman had her kitchen renovated. She purchased a new dishwasher, and the contractor doing the renovations installed it in the woman’s kitchen.
    Two months later, the woman was entertaining a friend at her home. As they talked, they noticed that the dishwasher was operating strangely, repeatedly stopping and starting. They saw sparks start flying out from underneath the dishwasher. The friend said, “It must have a short circuit or something. Don’t touch it or you might get electrocuted.” The woman asked if she should go down to the fuse box in her basement to cut off the electricity. The friend decided that he would take a look inside the dishwasher first. When he touched the metal handle on the dishwasher’s door, he received a violent electric shock. The dishwasher had an internal wiring defect that caused it to malfunction. The contractor who installed the dishwasher failed to adequately ground the machine; if he had done so, the electric current would have been led harmlessly away. The machine carried instructions for proper grounding, but the contractor did not follow them.
    All of the relevant events occurred in a jurisdiction that follows traditional contributory negligence and assumption of risk rules.
    If the friend brings a strict tort liability claim against the manufacturer of the dishwasher for his injuries, the probable result is that the friend will
    (A) recover, because the dishwasher was defectively made.
    (B) recover, because the manufacturer is vicariously liable for the improper installation.
    (C) not recover, because the friend assumed the risk by trying to open the dishwasher.
    (D) not recover, because the friend was not the purchaser of the dishwasher.
A
  1. (C) Implied assumption of risk traditionally was a defense to a negligence claim or a strict Liability claim concerning a defective product. Implied assumption of risk meant that the plaintiff voluntarily chose to encounter a known risk. For example, if a person knew that his car had defective brakes but decided to drive the car anyway, the person would be barred from holding liable the manufacturer or auto dealer that sold the car. In most states, implied assumption of risk is now simply handled as part of comparative fault. In other words, if a plaintiff voluntarily chose to encounter a known danger, that is not necessarily a total bar to liability but, instead, it is simply a reason the jury may choose to assign some fault to the plaintiff when it decides each party’s share of the comparative fault. Here, the friend knew the dishwasher was malfunctioning and even saw sparks flying out of the dishwasher. He knew there was a risk of receiving an electric shock, but he chose to touch the dishwasher anyway. The question says that traditional assumption of risk rules appLy and, therefore, the friend’s voluntary choice to encounter the known risk of electric shock completely bars him from holding the manufacturer liable. Note that if the question did not specify that traditional rules applied, you would need to assume that pure comparative fault applies, and the friend’s decision to touch the dishwasher would be dealt with as part of the comparative fault allocation, and it would be likely to reduce the friend’s recovery by some percentage, but not bar liability completely. Choice (A) is incorrect because implied assumption of risk would bar the friend’s claim. Choice (B) is incorrect because there is no reason the manufacturer would be vicariously liable for the improper installation of the product and, in any event, implied assumption of risk would bar the claim. Choice (D) is incorrect because an injured person can bring products liability claims even if he was not the purchaser or consumer of the product. No “privity of contract” is required, and even a random bystander who happens to be injured by the product can sue and recover damages.
How well did you know this?
1
Not at all
2
3
4
5
Perfectly
47
Q
  1. A buyer purchased a gallon of nonfat milk from a market. The milk, which was sold in a plastic container, had been processed and packaged by a dairy. That evening, the buyer was preparing dinner and poured himself a glass of milk from the container purchased at the market. He then sat down to eat his meal and proceeded to take a mouthful of milk. As he did so, the buyer felt something furry lodge in his mouth. He immediately spat out the furry object and saw that it was a dead mouse. The buyer suffered severe emotional distress but did not suffer any physical illness or injury.
    If the buyer asserts a claim against the dairy based on negligent manufacturing, he will most likely
    (A) recover, because he suffered severe emotional distress.
    (B) recover, under the doctrine of res ipsa loquitur.
    (C) not recover, because the dairy’s negligence only caused mental disturbance.
    (D) not recover, because the buyer’s proper cause of action is for intentional infliction of mental distress.
A
  1. (C) Based on the holding in Sullivan v. H. RHood& Sons, 168 N.E.2d 80 (Mass. 1960), no recovery will be allowed for mental disturbance alone unaccompanied byphysical injury upon finding a dead mouse in a milk bottle. Where the defendant’s negligence causes only mental disturbance, without accompanying physical injury, illness, or other physical consequences, and in the absence of some independent basis for tort liability, the great majority of courts still hold that there can be no recovery. The rationale for denying such recovery is based upon the notion that such mental harm is often temporary and relatively trivial. Because claims may easily be falsified or exaggerated, merely negligent, rather than intentional, conduct should not be the basis for imposing a disproportionate financial burden on a defendant. Choice (C) is thus correct. Choice (A) is incorrect because severe emotional distress is not compensable under negligence principles in these circumstances. Choice (B) is incorrect because res ipso loquitur merely helps a plaintiff to prove that there was negligence, but does not overcome the lack of physical injury that bars the plaintiff from recovering here. Choice (D) is incorrect because there is no indication that the milk company intentionally put a mouse in the bottle, so liability for intentional infliction of emotional distress could not be imposed.
How well did you know this?
1
Not at all
2
3
4
5
Perfectly
48
Q
  1. On Friday night, a driver was injured in an automobile accident after his vehicle was sideswiped by an unidentified motorist. Following the accident, the driver was admitted to the hospital where he was treated for his injuries. While at the hospital, the driver was diagnosed with a broken back. He was put in a body cast and then placed in traction. The driver’s physician assembled the traction apparatus and positioned the driver’s left leg in an overhead stirrup at a 40-degree angle. The driver was instructed to lie on his back on the bed.
    On Saturday morning after breakfast, the driver requested a bedpan from a nurse. She was placing the bedpan under the driver’s body when the stirrup, which was holding his leg, broke. This caused the driver’s leg to fall against the metal framing along the side of the bed, fracturing his tibia. The traction apparatus was defective because the manufacturer had not included a safety latch on the stirrup device to prevent its falling off if not securely fastened in place.
    If the driver asserts a claim against the manufacturer, he will most likely
    (A) prevail, because the stirrup broke while the traction apparatus was in normal use.
    (B) prevail, because the manufacturer’s failure to include a safety latch made the traction apparatus defective.
    (C) not prevail, because the traction apparatus was no longer within the control of the manufacturer.
    (D) not prevail, if the physician did not properly attach the stirrup when assembling the traction apparatus.
A
  1. (B) Often in torts questions, the cause of action is not designated, and the question simply reads: “If A asserts a claim against B, judgment for whom?” As a consequence, you must first determine the proper cause of action. Here, the plaintiff is suing the manufacturer of the traction device. In accordance with Section 402A of the Restatement (Second) of Torts, a manufacturer or seLLer of goods will be strictly liable for injuries caused by a defective product. Thus, since the traction apparatus was defective because of its failure to include a safety latch, the driver will prevail. Choice (B) is, therefore, the best answer. Choice (A) is incorrect because the mere fact that the apparatus broke during normal use does not by itself make it defective, nor does it prove that the manufacturer was negligent. Choice (C) is incorrect because a seller of a defective product remains strictly liable even after it leaves his control. Choice (D) is incorrect because a negligent act of a third party (like the physician) does not necessarily bar the manufacturer from being held liable. The physician’s negligence would block the manufacturer from being held liable only if the physician’s conduct was such an unforeseeable and significant cause of the accident that it must be treated as a supervening cause that breaks the chain of proximate causation between the manufacturer and the injured plaintiff. The facts do not indicate that the physician’s actions were unforeseeable, so choice (0) is not the best answer here.
How well did you know this?
1
Not at all
2
3
4
5
Perfectly
49
Q
  1. A woman broke her leg while skiing. She was taken to a hospital where she was treated by a doctor who determined that the woman would need surgery to set the broken bones so that they would heal properly. During the surgery, the doctor inserted several metal screws into the woman’s leg to hold the bones together. Although the hospital charged the woman a substantial fee for the doctor’s services, the bill did not include a separate charge for the cost of the bone screws.
    A few months after the surgery, the woman’s leg still was not fully healed. The doctor eventually concluded that the screws that were inserted into the woman’s leg had been defective. The woman was forced to have a second surgery to replace the defective screws.
    If the woman asserts a strict tort liability claim against the doctor and the hospital for using defective screws in her surgery, she will most likely
    (A) prevail, because the doctor and the hospital provided a product to her that was defective and unreasonably dangerous.
    (B) prevail, because the doctor and the hospital probably would have realized the screws were defective if they had carefully examined them.
    (C) not prevail, because the doctor and the hospital did not manufacture the screws and, therefore, the defect in the screws was not their fault.
    (D) not prevail, because the doctor and the hospital will be treated as service providers rather than product sellers.
A
  1. (D) Under Section 402A of the Restatement (Second) of Torts, a person can be held strictly liable for selling a product that is defective and unreasonably dangerous. This form of strict liability applies only to the sale of products, and not to the provision of services. Where some type of product or device (such as a pacemaker, prosthetic item, or artificial joint) is implanted into a patient during surgery, most courts hold that the doctor and hospital should be treated as providing health care services and not as a selLer of a product. That means the patient may bring a strict liability claim against the manufacturer of the item, but the patient will need to sue the doctor or hospital for negligence (i.e., medical malpractice) rather than strict liability. Here, the patient’s strict liability claim against the doctor and hospital will fail because the defendants will be treated as providing health care services, rather than selling a defective product. Choice (A) is wrong because the doctor and hospital will not be treated as having sold the bone screws, and so strict liability will not apply. Choice (B) is incorrect because the question asks about strict liability, and whether the doctor and hospital should have realized the screws were flawed through a careful inspection is relevant to a negligence claim but not strict liability. Choice (C) is wrong because, in general, all sellers of a defective product, not just the manufacturer, will be subject to strict liability. Strict liability does not require proof that anyone was at fault, so the fact that the doctor and hospital were not at fault is irrelevant here.
How well did you know this?
1
Not at all
2
3
4
5
Perfectly
50
Q
  1. A consumer purchased a new station wagon from a car dealer. The car dealer performed a thorough inspection of the station wagon before the sale was completed. One evening, the consumer was driving along a city street when he saw the traffic light facing him turn from green to amber. He sped up, hoping to cross the intersection before the light turned red. However, he quickly realized that he could not do so and applied the brakes, which failed because of a defect in the brake mechanism. The consumer then swerved to avoid hitting a bus that was crossing the intersection at a right angle to him. As a result of the swerve, the consumer’s car rode up on the sidewalk and overturned, pinning a pedestrian. Both the consumer and the pedestrian were severely injured.
    If the consumer asserts a claim against the car dealer based on strict liability in tort, will the plaintiff prevail?
    (A) Yes, because the brakes failed while the consumer was driving his car.
    (B) Yes, because the brakes failed because of a defect present when the consumer purchased the car.
    (C) No, because the consumer contributed to his own injury by speeding up.
    (D) No, because the car dealer carefully inspected the car before selling it.
A
  1. (B) Under Section 402A of the Restatement (Second) of Torts, a person engaged in the business of selling a product can be held strictly liable if the product causes harm because it is defective and unreasonably dangerous. Thus, if the car was sold with defective brakes, the car dealer will be strictly liable for the consumer’s injuries. ConsequentLy, choice (B) is the best answer. Choice (A) is incorrect because the mere fact that an accident occurs or a product fails to work properly is not enough to make strict liability apply. The plaintiff must show that the product had a defect at the time it was sold. Choice (C) is incorrect because under pure comparative fault principles (which apply on the Multistate, unless a question indicates otherwise), the fact that the consumer’s own negligence contributed to his injury would resuLt in the jury reducing his damages by some percentage but would not bar his claim completely. Note that if the question indicated that traditional contributory negligence rules apply, choice (C) would still be wrong because contributory negligence was not a defense to strict liability claims. Choice (D) is incorrect because strict Liability means the seller of the car would be liable if it was defective at the time of sale, even if the seller exercised reasonable care to inspect it.
How well did you know this?
1
Not at all
2
3
4
5
Perfectly
51
Q
  1. A farmer purchased a new pickup truck from a local automobile dealership. A few weeks later, the farmer was using the truck to haul a load of supplies to his farm. As he drove down a small hill toward an intersection with a stop sign, he applied the brakes, but they failed to work, and the truck did not slow down. The farmer could not stop for the stop sign. He saw a pedestrian crossing the street at the intersection and tried to steer around him, but was unable to do so. The pickup truck struck the pedestrian and seriously injured him.
    If the injured pedestrian asserts a negligence claim against the auto dealership that sold the truck to the farmer, and if it is conclusively proven that the brake failure resulted from a manufacturing defect in the car, will the pedestrian prevail?
    (A) No, because the pedestrian was not a purchaser or user of the truck.
    (B) No, if the farmer was negligent in being unable to steer around and avoid hitting the pedestrian.
    (C) Yes, because the auto dealer placed a defective car into the stream of commerce.
    (D) Yes, if the defect could have been discovered through the exercise of reasonable care by the auto dealer.
A
  1. (D) The seller of a product can be held liable for negligence if he fails to exercise reasonable care, and for strict liability if he sells a defective product. Obviously, a seller’s negligence may take a number of forms. Most frequently, it consists merely in failing to exercise reasonable care to inspect the goods to discover defects, or in preparing them for sale. Therefore, the auto dealer can be held liable for negligence if it failed to discover a defect in the truck that would have been detected by a seller exercising reasonable care. Choice (A) is incorrect because the products liability claims (negligence and strict liability) can be brought by people other than the purchaser or user of the product. Even a mere bystander who happens to be injured by the product can bring these claims. Choice (B) is incorrect because even if the farmer did a poor job of steering the truck after the brakes failed, that negligence by the farmer would not prevent the auto dealer from being held liable for its own negligence that was also a cause of the accident and injury. Choice (C) is incorrect because this question asks about a negligence cause of action rather than strict liability.
How well did you know this?
1
Not at all
2
3
4
5
Perfectly
52
Q
  1. A scientist used his car to transport a large quantity of highly flammable petroleum derivatives that he needed for his scientific research. The petroleum derivatives were sold in ordinary glass gallon jugs.
    Shortly after putting the jugs in the back of his car, the scientist was driving along a city street. He was thinking about a difficult scientific question and not paying attention to his driving. As a result, he lost control of the car and drove up onto the sidewalk. The car flipped over. The glass jugs in the back of the car were broken, and the chemicals in them spilled out onto the sidewalk.
    Moments later, a doctor who witnessed the accident came running over to render medical assistance. As he approached the overturned car, however, the doctor slipped on the petroleum derivatives that had spilled onto the sidewalk. The doctor fell and fractured his ankle.
    If the doctor asserts a claim against the scientist based on strict liability, will the doctor prevail?
    (A) Yes, because the scientist was engaged in an abnormally dangerous activity by transporting highly flammable petroleum derivatives in his car.
    (B) Yes, because the transportation of flammable petroleum derivatives in glass jugs necessarily involves a high degree of risk of serious harm.
    (C) No, because the doctor assumed the risk by voluntarily acting as a Good Samaritan.
    (D) No, because it was unforeseeable that the doctor’s injury would result from the type of harm threatened.
A
  1. (D) Usually, where a type of conduct is subject to strict liability, it is obvious what sort of harm is expected to result from the conduct. For example, the keeping of vicious animals invoLves the risk that human beings or other animals will be attacked; the risk of abnormally dangerous things and activities, such as high tension electricity or blasting, is obvious. However, in some situations, conduct that is subject to strict liability will result in an unexpected type of harm. In these circumstances, strict liability will not be imposed if the harm that occurs is not within the scope of the risk that makes the conduct subject to strict liability. Here, transporting highly flammable chemicals in one’s car is an abnormally dangerous or ultrahazardous activity because it is so likely to result in a fire or explosion. The possibility of someone slipping on the chemicals is not the risk that makes that activity abnormally dangerous, and so strict liability will not apply. Choice (A) is incorrect because even though the transport of highly flammable chemicals is an abnormally dangerous activity, someone slipping and falling on the chemicals is not the sort of risk that would be reasonably foreseen and that makes that activity abnormally dangerous. Choice (B) is incorrect because even though the transportation of the flammable petroleum in glass jugs is very dangerous, the plaintiffs slip and fall are not the foreseeable consequences of that dangerous activity. Choice (C) is incorrect because assumption of risk means that one voluntarily encounters a known risk. Although the doctor may have assumed certain risks in going to help at the accident scene, such as the risk of a possible explosion or fire, he did not know there was a risk of falling because of a slippery substance on the sidewalk.
How well did you know this?
1
Not at all
2
3
4
5
Perfectly
53
Q
  1. A company manufactures a popular dandruff shampoo. On the shampoo’s box cover is a warning label advising consumers to make a “patch test” before applying. The label states that a “patch test” is necessary because a small percentage of the population may be allergic to the chemicals contained in the product and susceptible to suffering scalp irritation.
    A Russian immigrant recently moved to the United States from Moscow. The immigrant cannot read or speak English. One day, the immigrant purchased a bottle of the shampoo from a drug store. She looked at the label warning but did not understand it. After throwing away the box cover, she used the shampoo without making a “patch test.” Minutes later, she began experiencing an allergic reaction and scalp irritation. This was followed by hair loss attributed to the shampoo.
    If the immigrant brings suit against the company for strict products liability, she will most likely
    (A) win, because she suffered injury from her use of the product.
    (B) win, because the manufacturer was aware that a small percentage of the population would suffer an allergic reaction to the shampoo.
    (C) lose, because she didn’t read or speak English. (D) lose, because she assumed the risk by not making the “patch test.”
A
  1. (C) A product may be unreasonably dangerous if there is an inadequate warning regarding its use. The warning on the shampoo bottle was adequate. American manufacturers are not required to place warnings in different languages on products sold within the United States. The plaintiff, therefore, will lose because the injury was attributable to her inability to read English, not to any flaw in the product or its warnings. Choice (A) is incorrect because even though the plaintiff suffered injury from using the product, the manufacturer’s warning was adequate; therefore, no liability can be imposed. Choice (B) is incorrect because while manufacturers are obligated to warn about the presence of ingredients to which a substantial number of people may be allergic, the manufacturer in this instance provided an adequate warning on the bottle. Choice (D) is incorrect because assumption of risk exists where the plaintiff is subjectively aware of a risk and chooses to encounter it anyway. In other words, the plaintiff must actually know about the danger. Here the plaintiff could not read the warning, so she was not aware of the risk.
How well did you know this?
1
Not at all
2
3
4
5
Perfectly
54
Q
  1. A woman was driving home after attending a concert. While talking on her cellphone, she momentarily took her eyes off the road. The woman then drove through a red light and struck a police car driven by a police officer, injuring him. At the time of the accident, the police officer was returning from an emergency dispatch.
    The police officer sued the woman to recover damages suffered in the auto accident. At trial, the police officer presented evidence that he was returning from an emergency scene when the woman drove through a red light and struck his patrol car. At the conclusion of the plaintiff’s presentation of evidence, the woman moved for a summary judgment claiming the “firefighter’s rule” barred recovery.
    The motion for summary judgment should be
    (A) granted, because the accident would not have occurred but for the emergency.
    (B) granted, because the police officer assumed the risk of injury by answering the emergency call.
    (C) denied, because the police officer’s injury was not related to any special danger of his job.
    (D) denied, because the firefighter’s rule does not apply to police officers.
A
  1. (C) The “firefighter’s rule” or “professional rescuer’s rule” provides that a professional rescuer (such as a firefighter or police officer) generally cannot hold someone liable for neglige.nce that creates a need for the rescuer’s services. For example, if you negligently start afire, firefighters come to put it out, and if one of them is injured while doing so, you will not be liable for the firefighter’s injuries. The policy rationales for this rule include: (1) the fact that we all pay taxes to cover the costs of services such as firefighting and police; (2) professional rescuers will be entitled to workers’ compensation benefits for on-the-job injuries; and (3) we do not want fear of liability to discourage people from caLling for help when they need it. This rule only applies, however, to risks that are inherent in the professional rescuer’s job and that are the very reason forthe rescuer’s presence at the scene. In other words, if you negligently start a fire in your house, you will not be liable for injuries that a firefighter suffers trying to put out the fire. However, if you happen to drive past a house where firefighters are working to extinguish a fire and you carelessly run over one of the firefighters, you will be liable. In this question, the woman did not do something negligent that created an emergency requiring the assistance of police officers. Instead, she simply ran a red light and hit a police officerwho happened to be driving through the intersection at that moment. This sort of accident is not a risk inherent in police work, and the woman’s negligence was not the reason the police officer was at that location. As such, the woman’s motion for summary judgment will be denied. Choice (A) is incorrect because the firefighter’s rule does not apply here, even though the police officer was returning from an emergency call, because that emergency call was completely unrelated to the plaintiff. The rule would apply if the plaintiff negligently created an emergency situation and then the police officer was injured while responding to it. Choice (B) is incorrect because the police officer did not assume the risk of being hit by a driver who had driven through a red light. The police officer was simply returning from the emergency calL at the time and had no reason to suspect such an occurrence might happen. Choice (D) is incorrect because police officers and other professional rescuers are covered under this rule, not just firefighters.
How well did you know this?
1
Not at all
2
3
4
5
Perfectly
55
Q
55. A first-class passenger on an airplane flight from Atlanta to Chicago was seated next to a middle-aged salesman who was returning home after attending a business convention in Atlanta. The passenger, who was extremely exhausted after a long and hectic day, tried to sleep during the flight but was constantly being annoyed by the salesman. Once the flight departed, the salesman started ordering drinks of Scotch and water from the flight attendant, and became inebriated. When the passenger refused to talk to him, the salesman became very abusive to the passenger. Although there were many empty seats available on the plane in the first-class section, the passenger decided to remain seated next to the salesman. Finally, after the flight attendant had served the saleman his tenth drink of Scotch, he became belligerent and punched the passenger in the mouth. The passenger’s two front teeth were knocked loose and she suffered a cut lip.
If the passenger asserts a claim against the airline based on negligence, she will most likely
(A) prevail, because, as a first-class passenger, she was owed a special duty of care by the airline.
(B) prevail, because the flight attendant should have been aware that her conduct caused an unjustifiable risk of harm.
(C) not prevail, because the airline is not vicariously liable for the tortious conduct of its passengers.
(D) not prevail, because the passenger assumed the risk by not moving to another seat away from the salesman.
A
  1. (B) A person generally does not have a duty to protect others from being harmed by third parties. For example, if you see someone being attacked by a criminal, you might choose to help the victim, but you generally do not have a legal obligation to do so. A legal duty does exist, however, if certain “special relationships” are involved. You might have a special relationship to the crime victim that obligates you to protect him, oryou might have a special relationship to the criminal that obligates you to control him. One of the special relationships traditionally recognized is the relationship between a common carrier (such as a railroad or airline) and its passengers. The common carrier has a duty to its passengers to take reasonable actions to protect them against unreasonable risk of physical harm. This duty to protect extends not only to risks arising out of the common carrier’s own conduct, but also to risks arising from acts of third persons, whether they be innocent, negligent, intentional, or even criminal. The flight attendant’s failure to stop serving drinks to the salesman in light of his inebriated state and abusive behavior posed an unreasonable risk of harm to the passenger. The airline will be vicariously liable for the flight attendant’s negligent conduct under the doctrine of respondeat superior. The passenger will, therefore, prevail. Choice (A) is incorrect because the airline has no greater duty to first-class passengers than to other passengers. Choice (C) is incorrect because even though the airline is notvicariously liable forthetortious conduct of its passengers, it is vicariously liable for the tortious conduct of its employees, and thus the airline can be liable for the negligent conduct of the flight attendant in serving alcohol to an already intoxicated passenger. Choice (D) is incorrect because in order for assumption of risk to apply, the plaintiff must be actually (i.e., subjectively) aware of the risk. Here, the plaintiff did not know that the salesman would strike her.
How well did you know this?
1
Not at all
2
3
4
5
Perfectly
56
Q
  1. A customer went into a bar to have a drink. The bar was crowded because a championship boxing match was being shown on the television sets in the bar, and many people had gone to the bar to watch it. During the second round of the boxing match, someone bumped into the customer, causing him to spill a drink on his pants. “Hey, watch what you’re doing, jerk!” the customer said. The person who had bumped the customer was an employee of the bar hired as a bouncer to deal with unruly customers. Angered by the customer’s statement, the employee immediately turned around and punched the customer in the mouth.
    If the customer asserts a battery claim against the owner of the bar, the customer will most likely
    (A) prevail, because it was reasonably foreseeable that fights would occur in a crowd of people who were drinking a lot of alcohol.
    (B) prevail, because the person who punched the customer was employed as a bouncer.
    (C) not prevail, because the owner did not touch the customer.
    (D) not prevail, because the owner should not be responsible for the intentional acts of the employee.
A
  1. (B) Under the doctrine of respondeat superior, an employer can be held vicariously liable for an employee’s tortious conduct that is within the scope of employment. Although respondeat superior usually applies to make an employer liable for an employee’s negligence, it also can make an employer liable for an employee’s intentional tort. An intentional tort would be particularly likely to be within the scope of empLoyment if the employee in question is doing a type of work, such as being a bouncer or security guard, that involves some use of physical force. Here, the bouncer would be liable to the customer for battery, and the owner of the bar would be vicariously liable for the battery. Choice (A) is incorrect because the fact that fights were reasonably foreseeable would be very significant for a negligence claim, but not crucial for an intentional tort claim. Choice (C) is incorrect because the owner could be liable for battery through respondeat superior even though he did not personally touch the customer. Choice (D) is incorrect because, as stated above, the owner would be liable for battery through respondeat superior.
How well did you know this?
1
Not at all
2
3
4
5
Perfectly
57
Q
  1. One Sunday afternoon, a sports fan went to a professional football game. While standing in line at a concession stand, he got into an argument with three men ahead of him in the line. The three men were friends attending the game together. One of the men punched the sports fan in the mouth. A fight ensued. During the fracas, one of the other three men hit the sports fan over the head with a bottle, which caused him to suffer a serious concussion. The sports fan does not know for certain which of the three men struck him with the bottle.
    If the sports fan asserts a claim against the man who punched him in the mouth to recover damages for the head injury, will the sports fan prevail?
    (A) Yes, because the man who punched him in the mouth was the instigator who struck the first blow that started the fracas.
    (B) Yes, because the man who punched him in the mouth and the other men were acting in concert.
    (C) No, because the man who punched him in the mouth did not actually strike the sports fan with the bottle.
    (D) No, because the sports fan cannot offer proof as to which of the men struck the sports fan over the head with the bottle.
A
  1. (B) Whenever two or more persons commit tortious acts in concert, each becomes subject to Liability for the acts of the others, as well as for his own acts. Since the three men who fought with the sports fan were acting in concert, each would be liable for the head injury suffered by the sports fan, regardless of which of the three actually hit the sports fan’s head with the bottle. Choice (A) is incorrect because it does not matter that the man who punched him in the mouth was the instigator, unless he and the other two men were acting in concert. Choices (C) and (D) are incorrect because whenever two or more persons commit tortious acts in concert, each becomes subject to liability for the acts of the others as well as his own act. Since the man who punched him in the mouth and the other two men were acting in concert, the man who punched him in the mouth would be liable even though he himself did not strike the sports fan with the bottle.
How well did you know this?
1
Not at all
2
3
4
5
Perfectly
58
Q
  1. A baseball fan purchased two tickets for a World Series baseball game. The fan contacted his best friend and invited him to go to the game. The friend, who was a fanatic baseball fan, eagerly agreed. The fan told the friend that the game started at 7:00 p.m. and that he would pick him up at about 5:00 p.m. so they could get there early to watch batting practice.
    They were driving to the game together when the fan sped up to cross an intersection while the traffic signal was changing from amber to red. As he reached the intersection, the fan was traveling at 50 m.p.h. although the posted speed limit was 25 m.p.h. Simultaneously, a car entered the intersection on red and collided with the fan’s vehicle. The friend suffered a broken pelvis in the collision. This jurisdiction has adopted the following “modified” comparative negligence statute:
    “A negligent plaintiff is entitled to obtain a recovery provided plaintiff’s negligence is not equal to or greater than that of the defendant’s; otherwise no recovery is permitted.”
    Suppose the friend brings suit against the driver of the car that entered the intersection on the red light to recover damages for his injury. Ajury returned a special verdict with the following findings: (1) The fan was 55 percent negligent in speeding; (2) The driver was 45 percent negligent in driving through the red light; and (3) The friend suffered $100,000 in damages. As a result, the court should enter a judgment for the friend in the amount of
    a. $100,000.
    b. $55,000.
    c. $45,000.
    d. nothing, because the fan was more negligent
    than the driver.
A
  1. (A) On all standardized examinations (whether it be the MBE, SAT, LSAT, or GRE), the examiner’s main goal is to hide the correct answer. The examiners try to accomplish this by employing “distracters” and “red herrings.” Here, the comparative negligence statute is the distracter. A comparative negligence statute only applies when there is a negligent plaintiff and one or more negligent defendants. Here, there were two negligent parties (the fan and the driver), but neither of them is the plaintiff. The plaintiff (the friend) was just a passenger who was riding in one of the cars and did nothing negligent. The comparative negligence statute, therefore, is irrelevant. The two negligent parties (the fan and the driver) are joint tortfeasors who can be held jointly and severally liable for the full amount of the plaintiff’s damages. Choice (A) is correct because the friend may recover 100 percent of the damages from the driver, who could then seek contribution from the fan. Choices (B), (C), and (D) are incorrect because the fan and the driver are joint tortfeasors who would be held jointly and severally liable for the full amount of the friend’s damages.

“Modified” v. “Pure” Comparative Negligence

“Pure” comparative negLigence is simple. It means that where a jury assigns some fault to the plaintiff and some fault to the defendants, the plaintiff can recover for all the fault assigned to the defendants, no matter how high or low it may be compared to the plaintiff’s own fault. For example, even if the plaintiff was 99 percent at fault and the defendant was only 1 percent at fault, the plaintiff can still recover 1 percent of the damages for the plaintiffs injury.

In jurisdictions with “modified” comparative negligence rules, there is a cutoff point at which the plaintiffs share of the fault becomes so high that the plaintiffs claim is barred, and the plaintiff will recover nothing. In some jurisdictions, the plaintiff’s recovery is barred if the plaintiffs share of the fault is greater than the defendant’s share (e.g., if the plaintiff was 50 percent at fault and the defendant was 50 percent at fault, the plaintiff could recover 50 percent of the damages, but if the plaintiff was 51 percent at fault and the defendant was 49 percent at fault, the plaintiff would recover nothing). In other jurisdictions, the pLaintiffs recovery is barred if the plaintiffs share of the fault is equal to or greater than the defendant’s share (e.g., if the plaintiff was 49 percent at fault and the defendant was 51 percent at fault, the plaintiff could recover 51 percent of the damages, but if the plaintiff was 50 percent at fault and the defendant was 50 percent at fault, the plaintiff would recover nothing). Moreover, jurisdictions that have modified comparative fault rules also differ on how the comparisons are made if there are multiple negligent defendants. In some states, the plaintiffs share of the fault is compared separately with that of each individual defendant. Under the “unit rule” used in some other jurisdictions, the plaintiffs share of the fault is compared with the sum of all the defendants’ shares.

Example: Plaintiff suffers $100,000 damages and is 40 percent at fault. Defendant 1 is 10 percent at fault and Defendant 2 is 50 percent at fault. Analysis: Plaintiff can recover $60,000 in a pure comparative negligence jurisdiction. In a modified comparative negligence jurisdiction in which the plaintiffs share is compared separately to each defendant’s share, the plaintiff would be able to recover nothing from Defendant 1 (because the plaintiffs 40 percent share exceeds the 10 percent share of Defendant 1), but would be able to recover $50,000 from Defendant 2 (i.e., 50 percent of the $100,000). By contrast, under the unit rule, the plaintiffs share of the fault would be compared to the sum of all the defendants’ shares. Under that rule, the plaintiff could recover $60,000 from the defendants because the plaintiffs negligence would be less than the aggregate negligence of both defendants. Exam Tip: On questions involving multiple defendants, the facts will indicate whether to apply the unit rule in modified comparative negligence cases.

How well did you know this?
1
Not at all
2
3
4
5
Perfectly
59
Q
  1. At approximately 3:00 p.m. on December 3, a motorist was speeding down a street at the same time that a pedestrian was crossing the street. When the pedestrian started to cross the street, she saw the traffic light was red. She momentarily stopped, thought about waiting until the light turned green, but then decided to cross anyway. As the pedestrian was halfway across the street, the motorist, who was driving 50 m.p.h. in a 25 m.p.h. zone, struck her with his vehicle. The pedestrian suffered a broken pelvis and internal injuries in the accident. While in the hospital, the pedestrian’s insurance company paid $10,000 of her medical expenses. Six months later, the pedestrian brought suit against the motorist to recover damages for all her medical expenses. This jurisdiction has a “modified” comparative negligence statute in effect.
    Suppose a jury returned a special verdict with the following findings: (1) The motorist was 60 percent negligent in speeding; (2) The pedestrian was 40 percent at fault in crossing the street against the red light; and (3) The pedestrian suffered damages from the accident totaling $50,000. After the verdict, the court was advised that the pedestrian’s insurance company had already paid the pedestrian $10,000. As a result, the court should enter a judgment for the pedestrian in the amount of
    (A) $50,000, because the motorist’s negligence was greater than the pedestrian’s.
    (B) $30,000, the proportion of the pedestrian’s damages caused by the motorist’s negligence, but the payment of $10,000 in hospital expenses will be disregarded under the collateral source rule.
    (C) $20,000, the proportion of the pedestrian’s damages caused by her own negligence, but the payment of $10,000 in hospital expenses will be disregarded under the collateral source rule.
    (D) $10,000, the proportion of the pedestrian’s damages caused by her own negligence, less the $10,000 in hospital expenses already paid by the insurance company.
A
  1. (B) Students must be cognizant of the distinction between “modified” comparative negligence and “pure” comparative negligence. In a modified comparative negligence jurisdiction, there will be a cutoff point at which the plaintiffs share of the fault becomes so great that the plaintiffs recovery is barred completely. In some jurisdictions, that cutoff point will be when the plaintiffs share of the fault is greater than that of the defendant’s. In other jurisdictions, it will be when the plaintiffs share is equal to or greater than that of the defendant’s. In the present question, the pedestrian’s negligence was less than that of the motorist’s negLigence, so the pedestrian will be entitled to recover in a modified comparative negligence jurisdiction. Choice (B) is correct because her recovery will be $30,000 (i.e., the total amount of damages of $50,000 reduced by 40 percent, which is the percentage of fault attributable to the plaintifl. Under the “collateral source” rule, a plaintiffs recovery is not reduced by virtue of the plaintiff having received compensation from sources other than the defendant. For example, if people feel sorry for the injured plaintiff and give her gifts, those gifts do not reduce the amount that the defendant owes for causing the plaintiffs injury. Likewise, the $10,000 paid by the pedestrian’s insurance company does not reduce her recovery from the motorist. Choice (A) is incorrect because under comparative negligence, the plaintiff recovers only for the portion of the fault assigned to the defendant. Choice (C) is incorrect because the amount that the pedestrian can recover is the proportion of the damages caused by the motorist’s negligence ($30,000), not the pedestrian’s own negligence ($20,000). Choice (D) is incorrect for the same reason that choice (C) is incorrect, and also because the damages will not be reduced by the $10,000 payment made by the pedestrian’s insurance company.
How well did you know this?
1
Not at all
2
3
4
5
Perfectly
60
Q
  1. A buyer purchased a new van from a car dealer. Two weeks later, the buyer was driving to work when the brakes suddenly failed. The buyer tried to stop the van for a red light, but the brakes failed to operate. As a consequence, the buyer drove through the red light and collided with a car driven by a woman.
    Subsequently, the woman asserted a claim against the buyer to recover for the injuries she suffered in the accident. At trial, the only evidence offered by the plaintiff concerning the cause of the accident was the testimony of an engineering expert. He testified that a manufacturing defect had caused the brakes to suddenly fail.
    Based on the facts stated above, a motion by the buyer to dismiss at the end of the woman’s case should be
    (A) granted, because the woman presented no evidence that the buyer was negligent.
    (B) granted, because the woman was neither the user nor the consumer of the defective product.
    (C) denied, because the buyer had a non-delegable duty to maintain the brakes in a safe condition.
    (D) denied, because the buyer is strictly liable for injuries caused by a manufacturing defect in an auto that he had purchased.
A
  1. (A) This question deals with burden of proof and requires a two-step analysis. First, it is necessary to recognize that the woman is bringing a negligence action against the buyer. Certainly, the woman cannot be suing the buyer for strict liability because the buyer is not a manufacturer or seller of the automobile. Next, on the issue of negligence,’ the question becomes the sufficiency of the evidence. In civil suits, unlike criminal prosecutions, the burden of proof does not require that the jury be convinced beyond all reasonable doubt, but only that they be persuaded that a preponderance of the evidence is in favor of the party sustaining the burden. Thus, the burden of pro.of of the defendant’s negligence is on the plaintiff to show the defendant’s negligence by a preponderance of the evidence. Because the woman presented no evidence that the buyer was negligent, the court should sustain the buyer’s motion to dismiss. Choice (B) is incorrect because it does not matter whether the woman was a user or consumer of the product. The woman’s claim fails because there is no proof that the buyer was negligent, not because the woman was not a user or consumer of the van. Choice (C) is incorrect because the issue here is not whether a duty existed. The buyer undoubtedly had a duty to exercise reasonable care, in maintaining the van as well as in driving. Although a duty existed, the issue here is whether there was sufficient evidence that the buyer breached the duty, and there is no such evidence. Choice (D) is incorrect because strict liability in tort would apply to a person engaged in the business of selling the defective product, and the buyer was a purchaser, rather than a seller of the van.
How well did you know this?
1
Not at all
2
3
4
5
Perfectly
61
Q
  1. A laboratory manufactures nitroglycerin (a heavy, oily, explosive, poisonous liquid used chiefly in making dynamite) at its main plant. An artist who specialized in making delicate glass sculptures had a studio two blocks away from the laboratory’s plant. Late one evening, there was an explosion at the laboratory’s plant. The force of the explosion caused the artist’s studio to be shaken, which resulted in the destruction of valuable artwork in the studio.
    The artist now asserts a tort action against the laboratory to recover damages. Which of the following, if established, would furnish the laboratory with its best possible defense?
    (A) The laboratory used extraordinary care in the manufacture and storage of nitroglycerin and was not guilty of any negligence that was causally connected with the explosion.
    (B) The laboratory has a contract with the federal government whereby all the nitroglycerin manufactured at its plant is used in U.S. military weapons systems.
    (C) The explosion was caused when lightning (an act of God) struck the plant during an electrical storm.
    (D) The harm that the artist suffered would not have resulted but for the abnormal fragility of the artist’s work.
A
  1. (D) Strict liability may be imposed on one who carries on an abnormally dangerous or ultrahazardous activity. The person will be liable for harm resulting from the activity even if he exercised the utmost care to prevent the harm. The plaintiff must prove, however, that the defendant’s abnormally dangerous activity was both an actual and proximate cause of the plaintiff’s harm. This means the defendant will be strictly liable only if the harm results from the risk that makes the defendant’s activity abnormally dangerous. Strict liability will not be imposed if the harm occurred only because of the abnormal sensitivities of the plaintiff’s activities. Choice (A) is incorrect because the fact that a defendant was very careful will not prevent him or her from behind held strictly liable for an abnormally dangerous activity. Choice (B) is incorrect because even though most courts have held that governments are not subject to strict liability for abnormally dangerous activities, most courts have heLd that this immunity does not carry over to the contractors performing work for the government. Choice (C) is incorrect because strict liability for an abnormally dangerous activity can be imposed even if the harm triggered the operation of an unexpected force of nature. Even unexpected lightning, therefore, would not relieve the laboratory of strict liability for the explosion.
How well did you know this?
1
Not at all
2
3
4
5
Perfectly
62
Q
  1. A woman was seven months pregnant with her first child. One afternoon, the woman was in the kitchen of her home preparing a snack when she glanced out the window and saw a horrible crime being committed. The woman watched in horror as a local gang member pulled out a handgun and used it to shoot the victim, a member of a rival gang. The gang member did not know anyone was witnessing his act of shooting the victim. The woman watched as the victim collapsed and fell dead on her neighbor’s front lawn. As a result of her shock from viewing this horrible incident, the woman suffered a miscarriage.
    If the woman asserts a claim against the local gang member who shot the victim, the woman will
    (A) recover, because she suffered a physical injury, rather than just emotional harm.
    (B) recover, because the gang member acted intentionally in shooting the victim.
    (C) not recover, because the woman was not within the zone of danger.
    (D) not recover, because the gang member was unaware that the woman was watching.
A
  1. (D) The issue presented is whether a plaintiff may recover for intentional infliction of emotional distress where the target of the defendant’s extreme and outrageous act is some other person, not the plaintiff. A plaintiff can recover in either of the following two situations: (1) The plaintiff is present when the defendant does something extreme and outrageous to a third person, the defendant is aware of the plaintiff’s presence, and the plaintiff is a close family relative of the third person who is the target of the defendant’s extreme and outrageous act. (2) The plaintiff is present when the defendant does something extreme and outrageous to a third person, the defendant is aware of the plaintiff’s presence, and the shock of witnessing the incident is so horrible that it causes the plaintiff to suffer bodily injury (such as a heart attack, stroke, etc.) rather than just emotional distress. Here, the woman cannot recover against the gang member for intentional infliction of emotional distress under either of these two rules, because the gang member was unaware of the woman’s presence. Choice (A) is incorrect because the fact that the woman suffered a physical injury is not sufficient to produce liability unless the defendant was aware of the woman’s presence. Choice (B) is incorrect because the fact that the gang member acted intentionally is not sufficient to produce liability unless he was aware of the woman’s presence. Choice (C) is incorrect because zone of danger is only in issue when a plaintiff is claiming emotional distress suffered for what almost happened to them. For example, if a plaintiff claims that she suffered emotionaL distress at aLmost being hit by a speeding car, the plaintiff would have to show that she was within the zone of danger of being hit by the car. Therefore, zone of danger would not apply to this fact situation.
How well did you know this?
1
Not at all
2
3
4
5
Perfectly
63
Q
  1. Three friends decided to go bowling together. They made a friendly wager on the match, with the winner receiving a free beer from the other two guys. In the second frame of the first game, the first friend apparently got a strike when he knocked down all 10 pins. However, the second friend accused the first friend of fouling because his foot went over the line. The first friend denied fouling and wanted to mark down a strike on the scorecard. The second friend refused to give the first friend a strike and wrote in a zero on the scoring sheet. The first friend became enraged.
    The second friend then went to bowl his frame. As the second friend turned his back, the first friend approached from behind with a bowling ball in his hand. The first friend then raised the bowling ball threatening to hit the back of the second friend’s head. The second friend, who had his back turned, did not see the first friend’s actions. The third friend saw what the first friend was about to do and could easily have warned the second friend; but the third friend remained silent. The first friend then struck the second friend on the back of the head with the bowling ball, causing a deep scalp wound.
    If the second friend asserts a cause of action against the third friend, he should
    (A) prevail, because the third friend was aware of the danger.
    (B) prevail, on account of the third friend’s omission to act.
    (C) not prevail, because the first friend was responsible for his injury.
    (D) not prevail, because the third friend was under no duty to warn the second friend of the danger.
A
  1. (D) The fact that the defendant reaLizes or should realize that action on his part is necessary for another’s aid or protection does not by itself impose a duty to take such action. There are situations where a special relationship between the parties (e.g., common carrier—passengers and innkeeper—guests) wiLl impose a duty to act. However, in this question there is no special relationship between the parties. Despite the fact that the third friend was aware of the danger, he did not have a duty to warn the second friend. Choice (D) is thus correct. Choice (A) is incorrect because although the third friend was aware of the danger, he did not have a duty to warn the second friend. Choice (B) is incorrect because the defendant cannot be held liable for an omission unless the defendant had a duty to act. Choice (C) is incorrect because there are situations in which more than one person can be held liable and responsible for an injury to a plaintiff. Here, the reason why the third friend is not liable is not because the first friend is responsible but, instead, because the third friend was under no legal duty to act.
How well did you know this?
1
Not at all
2
3
4
5
Perfectly
64
Q
  1. During a baseball game, a spectator watching the game started taunting one of the players for striking out. Annoyed by the taunts, the player started an argument with the umpire. When the umpire told the player to be quiet and go back to his team’s bench, the player became enraged and savagely attacked the umpire, hitting him in the head with a bat. The spectator who had taunted the player was overcome by humiliation and horror at what occurred, because he was a good friend of the umpire. The spectator suffered no physical injury, but experienced severe nervous shock and brooded over the incident for months.
    If the spectator asserts an action for intentional infliction of emotional distress against the baseball player, the spectator will
    (A) win, because the player’s conduct was extreme and outrageous.
    (B) win, because he was present and witnessed the attack.
    (C) lose, because he was partly responsible for causing the attack to occur.
    (D) lose, because the umpire was not an immediate family member of the spectator.
A
  1. (D) A plaintiff generally cannot recover for intentional infliction of emotional distress unless the plaintiff is the target of the defendant’s extreme and outrageous act. However, a plaintiff can recover for emotionaL distress caused by seeing the defendant do something extreme and outrageous to another person if (1) the plaintiff is present when the defendant does the extreme and outrageous thing to the person, the defendant is aware of the plaintiffs presence, and the plaintiff is a close family relative of the person who is the target of the defendant’s extreme and outrageous act; (2) the plaintiff is present when the defendant does something extreme and outrageous to a third person, the defendant is aware of the plaintiffs presence, and the shock of witnessing the incident is so horrible that it causes the plaintiff to suffer bodily injury (such as a heart attack, stroke, etc.), rather than just emotional distress. Here, the spectator did not suffer any physical injury, so he can recover for intentional infliction of emotional distress only if he is related to the umpire, which, under the facts, he is not. Therefore, Choice (D) is the best answer. Choice (A) is incorrect because even though the player’s conduct was extreme and outrageous, the spectator cannot recover unless he was a family member of the umpire. Choice (B) is incorrect because mere presence is not by itself enough to permit a plaintiff to recover for intentional infliction of emotional distress where the plaintiff was not the target of the defendant’s extreme and outrageous conduct. Choice (C) is incorrect because the spectator’s taunting of the baseball player may have been impolite, but there is no legal rule under which it would bar recovery under tort law.
How well did you know this?
1
Not at all
2
3
4
5
Perfectly
65
Q
  1. While on vacation, two friends went scuba diving in the ocean. Each of them was equipped with a spear gun, which they planned to use to kill tuna. As they were swimming, both men saw what appeared to be a large fin entangled in a nearby coral reef. Simultaneously, they shot their spear guns at the projecting wing-like figure. Unbeknownst to either of the friends, it was not a fish but rather another scuba diver. The victim was struck and slightly wounded by one of the spears.
    The victim asserts a claim for damages against the two friends. At trial, the judge should instruct the jury to rule
    (A) in favor of both defendants if no evidence is presented showing who actually shot the spear that injured the victim.
    (B) against both defendants jointly unless one of them proves that he did not shoot the spear that struck the victim.
    (C) against each defendant for one-half of the amount of damages, because they both acted independently and not jointly.
    (D) against each defendant for one-half of the amount of damages, because they both shot their spear guns simultaneously.
A
  1. (B) A plaintiff is generally required to identify the person who was the cause of the plaintiffs injury. However, the doctrine of “alternative liability” or “alternative cause liability” can assist a plaintiff unable to prove which one out of a group of defendants caused the plaintiffs injury. This doctrine applies where the plaintiff sues a group of defendants and shows that every one of the defendants engaged in the same tortious conduct, one of the defendants caused the plaintiffs injury, and the plaintiff is unable to determine which one was the cause. The burden of proof on the issue of who actually caused the harm will be shifted to the defendants. If a defendant can prove that it was not the cause, that defendant will avoid liability. However, defendants who cannot exculpate themselves (i.e., cannot prove they were not the cause) will be held jointly and severally liable for the plaintiffs injury. This is the rule enunciated in the case of Summers v. rice, 199 R2d 1 (Cal. 1948), where two hunters negligently fired shotguns in a plaintiffs direction. One of the hunters hit the plaintiff in the eye, but the plaintiff could not identify which hunter fired the shot that hit his eye. The plaintiff sued both hunters, and the court held that the hunters would be jointly and severally liable unless they could prove who actually caused the plaintiffs injury. Choice (B) is, therefore, correct. Choice (A) is incorrect because it overlooks the alternative liability rule under which the plaintiff may be able to prevail. Choices (C) and (D) are incorrect because if the doctrine of alternative liability is used successfully here, the defendants each will be jointly and severally liable for the plaintiff’s entire injury, rather than each being liable for half the injury.
How well did you know this?
1
Not at all
2
3
4
5
Perfectly
66
Q
  1. A driver and passenger were driving to work in the former’s automobile one morning. As they were traveling at a speed of 20 m.p.h. (which was within the posted speed limit), the passenger suddenly pointed to an overturned vehicle along the side of the highway and said, “Look at that car upside down.” The driver turned to look at the overturned vehicle. As he was looking toward the side of the road, the driver failed to see an abandoned vehicle with a flat tire in the highway about 200 feet in front of his approaching auto. Seconds later, the driver crashed into the rear of the abandoned auto and was injured. The jurisdiction has a relevant comparative negligence statute in effect.
    If the driver asserts a claim against the owner of the abandoned auto, the most likely result is that the driver will
    (A) recover all of his damages, because the defendant created a dangerous condition.
    (B) recover only a portion of damages, because the abandoned auto was in plain view.
    (C) recover nothing, because he had the last clear chance to avoid the collision.
    (D) recover nothing, because the passenger’s act was a supervening cause.
A
  1. (B) As a general rule, comparative negligence statutes have the effect of apportioning damages based on the parties’ respective degrees of fault. For example, if the defendant’s fault is found to be twice as great as that of the plaintiff, the latter will recover two-thirds of his damages and bear the remainder of his loss himself. The driver was negligent in failing to keep a proper lookout while driving. By the same token, the defendant was also at fault because he left an abandoned auto in the middle of the highway. As a consequence, the plaintiff’s recovery will be diminished in proportion to his negligence. Choice (A) is incorrect because the plaintiff’s recovery will be diminished in proportion to his own negligence. Therefore, the plaintiff will not recover all of his damages. Choice (C) is incorrect because the “last clear chance” doctrine is applied in contributory negligence, not comparative negligence jurisdictions. Choice (D) is incorrect because the accident was a reasonably foreseeable result of the defendant’s negligence, even though the passenger’s action also played a role in bringing about the accident. There are often many events, forces, and actions that intervene between a defendant’s negligence and a plaintiff’s injury. Those intervening things generally are not supervening causes breaking the chain of proximate causation between the defendant’s negligence and the plaintiff’s injury. Although it is difficult to define exactly when an intervening cause will be treated as supervening and cut off the defendant’s liability, one of the most important factors is foreseeability. A foreseeable intervening act will rarely be treated as supervening. Here, the passenger’s action is within the scope of the foreseeable risk created by the defendant’s negligence, because the fact that passengers often briefly distract drivers is one of the reasons it is negLigent to abandon an automobile in the middle of the highway.
How well did you know this?
1
Not at all
2
3
4
5
Perfectly
67
Q
  1. A hardware store had a public pay telephone attached to the outside wall of its building. The telephone was owned and operated by the local telephone company.
    On Thursday, November 25, the store was closed for Thanksgiving. A resident, who lived two doors away, walked to the hardware store to use the telephone. He wanted to call his son and daughter- in-law to wish them a happy Thanksgiving, but his home phone was out of order. The resident picked up the receiver, inserted coins for the cost of the call, and promptly received an electric shock. Although the resident was momentarily stunned, he did not suffer any pain or injuries. Unbeknownst to the resident, a technician employed by the telephone company had incorrectly rewired the telephone the previous day, causing a short circuit in the telephone mechanism.
    If the resident institutes a personal injury action for negligence against the telephone company, he will most likely
    (A) recover, because the technician’s error would constitute the proximate cause of the resident’s injuries.
    (B) recover, because the technician’s error was a substantial factor in causing the resident’s injuries.
    (C) recover, under the doctrine of respondeat superior
    (D) not recover, because the resident did not suffer any injuries.
A
  1. (D) A negligence action requires proof that the plaintiff suffered some actual harm. Consequently, because the resident did not suffer any pain or other harm from the electrical shock, he would be unsuccessful in his negligence action. Choices (A), (B), and (C) are incorrect because in order for a plaintiff to prevail in a negligence cause of action, there must be some harm or injury to the plaintiff’s person or property.
How well did you know this?
1
Not at all
2
3
4
5
Perfectly
68
Q
  1. During a run one morning, a jogger passed a convenience store. He stopped and went into the store to use the bathroom available inside the store.
    Which of the following would best describe the jogger’s legal status in his utilization of the bathroom?
    (A) Licensee.
    (B) Gratuitous licensee.
    (C) Public invitee.
    (D) Business visitor.
A
  1. (C) The jogger’s LegaL status is that of a public invitee. A public invitee is a person who is invited to enter or remain on land as a member of the public for a purpose for which the land is held open to the public. Choices (A) and (B) are incorrect because a person who enters land held open to the public is an invitee, not a mere licensee. If the jogger stopped at a friend’s house, for example, and received permission to enter and use the bathroom in the house, the jogger then would be a licensee. Choice (D) is incorrect because the jogger did not enter the Land in order to carry out any business dealings with the possessor or employees of the possessor.
How well did you know this?
1
Not at all
2
3
4
5
Perfectly
69
Q
  1. A man invited several friends to come over to his house to watch a movie on television. While they were watching the movie, freezing rain fell outside, coating everything with a thin layer of very slippery ice. When the movie ended, one of the man’s guests slipped on the ice as soon as he took a step out of the door and onto the front stoop of the man’s house.
    Which of the following would best describe the duty of care owed by the man to his guest?
    (A) No duty of care.
    (B) A duty to inspect the premises for unknown dangers and disclose their existence to others.
    (C) A duty to warn of any known dangerous condition on the premises.
    (D) An absolute duty of care.
A
  1. (C) First, the guest is a licensee, who, in the broadest sense, includes anyone who comes upon the land with a privilege arising from the consent of the possessor. Next, what duty of care does a possessor owe to a Licensee? As to passive conditions on the Land, it is still the settled rule that the possessor is under no obligation to the licensee with respect to anything that the possessor does not know. The duty is not to maintain the land in a safe condition, but to exercise reasonable care so that the licensee is aware of the danger known to the possessor. Choice (A) is incorrect because the possessor of the land owes a duty to exercise reasonable care with respect to dangerous conditions of which the possessor has knowledge. Choice (B) is incorrect because with respect to a licensee, the possessor of land does not have a duty to inspect and find dangers not already known to the possessor. Choice (D) is incorrect because the possessor of land does not have an absolute duty to a licensee; the duty is merely for the possessor to exercise reasonable care so that the licensee is aware of the danger.
How well did you know this?
1
Not at all
2
3
4
5
Perfectly
70
Q
  1. An owner owned a two-story building, which he leased to a tenant. The tenant established a hardware store on the first floor and equipped the second floor as an apartment in which he lived with his wife and children. The two floors were connected by an outside wooden staircase with a handrail. The staircase was in a dilapidated condition at the time the tenant entered into the leasehold agreement. When the tenant took possession of the building, he notified the owner about the condition of the staircase and insisted that it be repaired. Although the owner orally promised the tenant that he would remedy the condition, he failed to do so.
    Later that evening, the tenant and his wife were receiving some friends for a small dinner party. One of the guests arrived at 7:00 p.m. and climbed the stairs to the second floor apartment. When the guest was half way up the stairway, which had not been repaired, it collapsed, seriously injuring her.
    In a negligence action initiated by the guest to recover for injuries suffered as a result of her fall, she will most likely
    (A) recover against the tenant only, because as a general rule, a lessor of land is not liable to his lessee or to others on the land for physical harm caused by any dangerous condition that existed when the lessee took possession.
    (B) recover against the owner only, because a lessor of land is subject to liability for physical harm caused to his lessee and others upon the land by a condition of disrepair existing before the lessee has taken possession.
    (C) recover against the owner only, because under the public use exception, a lessor who leases land for a purpose that involves the admission of the public is subject to liability for physical harm caused to such persons by a hazardous condition existing when the lessee takes possession.
    (D) recover against the tenant and the owner, because both the lessor and the lessee would be liable to others for their failure to remedy the defective staircase.
A
  1. (D) A lessee, as possessor of the property, has a duty to exercise reasonable care to warn licensees about dangers known to the lessee. A lessor of land, on the other hand, is generally not liable to his lessee or to others on the land for physical harm caused by any dangerous condition, whether natural or artificial, which existed when the lessee took possession. However, exceptions to this general rule incLude:
    (1) situations where the Lessor contracts to repair; (2) undisclosed dangerous conditions known to the lessor; (3) land leased for purposes involving admission of the public; (4) parts of land retained in the lessor’s control, which lessee is entitled to use; or (5) where the lessor makes negligent repairs. Here, the tenant had a duty to exercise reasonable care to warn the guest about the known danger of the handrail of the staircase. Likewise, the owner had a duty because he agreed to repair the handrail. Choices (A), (B), and (C) are incorrect because the plaintiff was owed a duty by both the owner and the tenant to warn of the dangerous condition that existed on the property. This failure to warn resulted in the injury to the guest and, as such, she will have a cause of action against both the owner and the tenant.
How well did you know this?
1
Not at all
2
3
4
5
Perfectly
71
Q
  1. A surgeon operated on a patient in an operating room in the Marina General Hospital. The surgeon was assisted by an intern, who was assigned to the operation by the hospital, and a nurse, who was on the staff of the hospital. During the patient’s convalescence, the patient complained of pain not explicable as an ordinary post-operative symptom. On investigation, it turned out that the intern and nurse, who had worked together in bandaging him, had done so in such a way as to constrict certain blood vessels. The faulty bandaging had caused acute pain and retarded the patient’s recovery.
    After the patient’s eventual recovery, he sued the intern, claiming $20,000 in damages. Before the case went to trial, the patient and intern agreed to a settlement in which the intern paid the patient $12,000, and the patient released the intern from all further claims for liability arising out of the incident.
    If the intern brings suit against the nurse for contribution, the most likely result is that the intern will
    (A) prevail, because one who settles without judgment can recover contribution.
    (B) prevail, because the nurse’s liability is established under res ipsa loquitur
    (C) not prevail, because one who settles without judgment cannot recover contribution.
    (D) not prevail, because the intern’s proper remedy is indemnification, not contribution.
A
  1. (A) As a general rule, most states have enacted statutes that to a greater or lesser extent permit contribution among tortfeasors. A more difficult question is whether a tortfeasor who has settled with the original plaintiff shall be entitled to contribution. It is for this reason that statutes of several jurisdictions Limit contribution to those against whom judgments have been rendered. It is important to note, however, where there is no such provision, it is almost invariably held that one who settles without judgment can recover contribution. See Hargar v. Caputo, 420 Pa. 528 (1966). Choice (A) is, therefore, correct because the intern can recover contribution from the nurse, even though the intern settled rather than being found liable by a court. Choice (B) is incorrect because it does not address the crucial issue of whether the intern can recover contribution despite settling without judgment. Choices (C) and (D) are incorrect because they each state an incorrect resuLt regarding the issue of contribution. Even when no judgment is entered, it is almost invariably held that one who settles voluntarily without judgment can recover contribution.
72
Q
  1. A manufacturer of appliances put a washing machine of a new and advanced design on the market. Two years of experimental use prior to commercial marketing had demonstrated that the machine laundered clothes more cleanly, more rapidly, and at a lower cost than any previous model.
    After nine months of commercial sales on a nationwide basis, the manufacturer discovered that an electrical unit in the machine tended to overheat greatly when the machine was heavily loaded or used continuously for more than one hour. The manufacturer promptly notified all dealers that it was recalling machines still in dealers’ stock for corrective adjustment. It also provided all dealers with forms of notice to previous purchasers, warning the purchasers of the hazard, urging the purchasers to arrange with the dealers to pick up the machines, with full reimbursement to the purchasers (at the manufacturer’s expense); and advising the purchasers who wished to use the machines pending pickup by the dealer to do so only with a clothes load of not more than one-half the machine’s capacity and for no longer than three-quarters of an hour.
    A purchaser of a washing machine, having been notified, arranged on Thursday to have a dealer pick up her machine on the following Monday. Over the weekend, the purchaser was visited by her married daughter and two grandchildren, aged 5 and 3. On Saturday, the purchaser left the house to go shopping. She forgot to warn her daughter about the washing machine. The daughter undertook to do a heavy load of accumulated laundry, including children’s clothing and bedding. She loaded the machine fully and repeatedly for an hour and a quarter. Leaving the laundry room with the children in it, she retired for a brief nap. The machine overheated; its paint caught fire, and the fire spread to the window curtains .The children were felled by the smoke. Awakened by the smell of smoke, the daughter rushed down to the laundry, screaming for help. A neighbor rushed into the house after phoning the fire station. Together they entered the laundry room and lifted and carried out the children. The daughter and the neighbor both suffered injuries from the fire and smoke in rescuing the children. Firemen soon arrived and extinguished the fire.
    If the daughter asserts a claim based on strict liability against the manufacturer for damages, the probable result is that the manufacturer will
    (A) win, because the daughter was not the purchaser of the washing machine.
    (B) win, because the manufacturer issued warnings to dealers and purchasers regarding the defect.
    (C) lose, because the manufacturer negligently designed the washing machine.
    (D) lose, because the washing machine was defective and unreasonably dangerous.
A
  1. (D) Section 402A of the Restatement (Second) of Torts provides that a person engaged in the business of selling a product can be held strictly liable if the product causes harm because it is defective and unreasonably dangerous. In order for this rule to apply, it is not necessary that the ultimate user or consumer have acquired the product directly from the seller. It is not even necessary that the consumer have purchased the product at all. A person injured by the product can recover, even if he is just a random bystander who happened to be near the product but has no relationship to any purchaser, user, or consumer of the product. Here, the injured plaintiff can prevail on a strict liability claim against the manufacturer if the product was defective and unreasonably dangerous. Choice (A) is incorrect because the pLaintiff can recover even though she did not purchase the washing machine. Choice (B) is incorrect because warnings alone may not eliminate the danger posed by a defective product. A manufacturer should eliminate the defect by using a safer design for the product, rather than just giving a warning and hoping that will be sufficient to prevent anyone from being hurt. Choice (C) is incorrect because the question asks about a strict Liability claim, not a negligence claim.
73
Q
  1. A homeowner purchased a new vacuum cleaner. A few days later, the homeowner received a severe electric shock while using the vacuum cleaner. The homeowner realized that there was a short in the wiring of the vacuum cleaner. The homeowner called the store that sold the vacuum cleaner and arranged to return it for a refund. The homeowner was busy, however, and would not have time to make a trip to the store for several days; so he put the vacuum cleaner in a corner in his living room to store it until he had a chance to take it to the store.
    The next day, the homeowner had an old friend from out of town arrive for a three-day visit. The homeowner had a spare room and loved to have overnight guests, so he had invited this old friend to come for a visit and was excited to see her. Shortly after the guest’s arrival, however, the homeowner had to go to work for a few hours, so he told his friend to make herself comfortable in the house while he went to his office.
    While the homeowner was away from the house, his friend got hungry and made herself a snack. After finishing the snack, she realized that she had dropped a lot of crumbs on the carpet. Wanting to be a polite guest, she decided to clean up the mess herself before the homeowner returned. She saw the vacuum cleaner in the corner, plugged it in at an electric outlet, and started to vacuum up the crumbs, when she received a very strong electric shock as a result of the vacuum cleaner’s faulty wiring.
    If the guest asserts a claim against the homeowner, the most likely result is that the guest will
    (A) recover, because the homeowner knew about the hazardous condition of the vacuum cleaner and yet failed to warn the guest.
    (B) recover, because the homeowner had a duty to make a reasonable inspection of the home to discover any unknown dangers.
    (C) not recover, because the homeowner did not create the risk of harm.
    (D) not recover, under the family purpose doctrine.
A
  1. (A) A guest who is present for social rather than business purposes is a licensee. Even though the person is invited and urged to come to the property, the guest is not an invitee. Fora licensee, the possessor of the property owes no duty of inspection and affirmative care to make the premises safe for his visit. UsuaLly, the guest understands when he comes that he is to be placed on the same footing as one of the family, and he must take the premises as the occupier himself uses them, without any inspection or preparation for his safety; he also understands that he must take his chances as to any defective conditions unknown to the occupier, and is entitled at most to a warning of dangers that are known. Here, the homeowner, therefore, can be held LiabLe for failing to warn the guest about a known danger. Choice (B) is incorrect because the homeowner does not have a duty to inspect and discover known dangers when the plaintiff is a mere Licensee. Choice (C) is incorrect because in order for a claim to exist, it is not required that the possessor of the land in question create the risk of harm. What is required is for the possessor to warn of dangers that are known to create a risk of harm. Choice (D) is incorrect because the famiLypurpose doctrine is not relevant to this question. The family-purpose doctrine provides that the owner of a car can be held liable for damages to anyone when the car is driven by a member of the car owner’s famiLy, with or without the car owner’s permission.
74
Q
  1. A city was growing rapidly and becoming a major tourist area because of its warm climate and proximity to the ocean. A businessman, anticipating a continued financial boom, purchased an expensive corner lot, which he planned to convert into an exclusive resort hotel. The businessman hired a builder to perform the construction work according to architectural guidelines provided by the businessman. It was anticipated that construction would take approximately six months.
    During this time, the builder dug a large hole and worked with heavy equipment inside the excavation site, leveling the terrain and building the foundation. The builder also erected a large wall along the sidewalk, which encircled the entire corner. The wall, which was negligently constructed, was made of thinly cut wood nailed together in sections and affixed just under the base of the sidewalk next to the excavation.
    Early one morning, a pedestrian was walking past the construction site on his way to work. As he was rounding the corner, the pedestrian saw a jogger approaching. When they drew even, the jogger accidentally brushed against the pedestrian with the swing of her arms. The pedesthan fell into the wall, which gave way, and he fell into the hole below. The pedestrian suffered serious injuries from the fall.
    If the pedestrian sues the businessman for the personal injuries he suffered in his fall, the most likely result is that the pedestrian will
    (A) recover, because the businessman is liable for permitting a dangerous condition to exist along an area of public use.
    (B) recover, under the doctrine of respondeat superior.
    (C) not recover, because the builder was an independent contractor.
    (D) not recover, because the jogger caused the pedestrian to fall into the wall.
A
  1. (A) As a general rule, an employer is not vicariously liable for the negligence of an independent contractor. Regarding independent contractors, there are two situations when vicarious liability does attach: (1) if the independent contractor is engaged in an activity with respect to which the employer has a non-delegable duty of care; or (2) if the independent contractor is performing an abnormally dangerous activity. With respect to the non-delegable duty exception, work performed in a public area necessarily involves an “inherent danger.”As a consequence, the employer should anticipate the need for specific precautions, such as when a painting is carried on a scaffold above a public street, or when a railing needs to be constructed around an excavation site. In such situations, the employer is held vicariously liable for the negligence of an independent contractor. Choice (B) is incorrect because the doctrine of respondeat superior applies to vicarious liability for torts of employees, not to the more specific rule of liability for non-delegable duties with respect to independent contractors. Choices (C) and (D) are incorrect because this situation, where work is being performed in a public area, involves an inherent danger. Therefore, the employer is held vicariously liable for the negligence of the independent contractor.
75
Q
  1. A pilot decided to fly his small airplane to another city for a weekend visit. He invited one of his best friends to join him as a passenger on the trip. Near the end of the flight, the pilot saw a cloud of fog ahead but decided that he could navigate through it. He got lost in the fog and crashed the plane into the side of a mountain. The pilot and his passenger were both killed instantly in the crash.
    If the passenger’s family brings a wrongful death action against the pilot’s estate, the most likely result is that the plaintiffs will
    (A) recover under strict liability, because the pilot engaged in an abnormally dangerous activity.
    (B) recover under negligence, because the pilot failed to exercise reasonable care in the operation of his airplane.
    (C) not recover, because the passenger did not pay to go on the flight.
    (D) not recover, because the passenger unreasonably exposed himself to a known danger.
A
  1. (B) In modern day society, flying an airplane is so commonplace as not to be regarded as an abnormally dangerous activity. Historically, flying was, of course, regarded at its outset as a questionable and highly dangerous enterprise. This view was encouraged by the fact that the first cases arose in New York, where there was strict liability for any physical invasion of land. In fact, the First Restatement of Torts in 1939 took the position that aviation had not reached such a stage of safety as to justify treating it by analogy to the railroads, and classified it as an “ultrahazardous activity” upon which strict liability for ground damage was imposed. However, with the further development of the industry, later years witnessed a definite reversal of this trend. The correct answer, therefore, is (B), because the plaintiffs will recover for negligence rather than strict liability. Choice (A) is incorrect because flying an airplane is no longer considered an abnormally dangerous activity. Choice (C) is incorrect because even though the passenger was a mere licensee, because he was a non-paying guest in the airplane, the pilot still owed him a duty of care with respect to known dangers like the fog. Choice (D) is incorrect because even if the passenger knew about the fog, his actions in merely being a passenger in the airplane were not unreasonable. Morever, even if the passenger were deemed to have acted unreasonably to some extent in encountering a known danger, that would merely reduce the damages to some extent under comparative fault principles, rather than barring recovery completely.
76
Q
  1. A family had been traveling cross-country on a camping trip when they decided to pull into Yellowstone National Park. During their stay at Yellowstone, the family camped out in a secluded area of the campgrounds surrounded by tall pine trees. Numerous signs had been posted throughout the campgrounds warning campers of the extremely dry conditions prevailing in the forest area due to a summerlong drought. On the morning of their departure, the family cooked breakfast over an open campfire. After breakfast, the family began to pack their gear into the van when the father instructed his 12-year-old son to put out the campfire. The son fetched two pails of water from a nearby pond and tossed the water over the campfire. Believing that he had extinguished the fire, the son joined his family in the van, and they drove away. Unbeknownst to the son and his family, the campfire was still smoldering when they left.
    Shortly thereafter, a wind gust blew the flames from the fire to some nearby brush. Due to the extremely dry conditions, the brush quickly ignited, starting a raging forest fire. The forest rangers dispatched a firefighting team, which included two helicopters equipped to fight the fire. While these helicopters were fighting the blaze from above, the smoke• surrounding one of the helicopters became so intense that the pilot lost all visibility. At the same time, an operator of a small private plane saw the blaze and recklessly descended into the smoke. In the pilot’s attempt to veer out of the intense smoke, his helicopter collided with the operator’s low- flying plane, causing the aircrafts to crash. Both the pilot and the operator were killed in the resulting explosion.
    In a wrongful death action brought by the pilot’s estate against the son and the father, the judgment should be for
    (A) the pilot’s estate, because the son was negligent in failing to properly extinguish the campfire.
    (B) the pilot’s estate, because the father is vicariously liable for the tortious conduct of his child.
    (C) the father and the son, because the operator’s reckless operation of his aircraft would be viewed as a superseding cause terminating the defendant’s liability.
    (D) the father and the son, because the son was only 12 years old and, therefore, cannot be deemed to have acted negligently.
A
  1. (C) A superseding or supervening cause is an unforeseeable intervening cause that relieves a defendant from liability for his antecedent negligence. The superseding event, in itself, becomes the proximate, or legal, cause of the plaintiff’s injury. Simply, if the defendant can foresee neither any danger of direct injury nor any risk from an intervening cause, he is not negligent. In the present case, the operator’s reckless operation of his aircraft would be viewed as a superseding cause terminating the father’s and the son’s liability. Choice (A) is incorrect because the operator’s reckless operation of his aircraft was the superseding cause of the pilot’s death. Choice (B) is incorrect because parents are not vicariously liable for tortious conduct of their children. Choice (D) is incorrect because children can be negligent. If the son failed to exercise the care of a reasonable child of the same age, intelligence, and experience, he would be deemed to have been negligent. Liability for negligence would not exist here, however, because of the operator’s reckless operation of the aircraft being a superseding cause.
77
Q
  1. The state has the following statutes regarding liquor sales in effect:
    “Section 1212. Whoever sells intoxicating liquor to any person who is intoxicated or inebriated is guilty of a misdemeanor. This section shall not apply in the case of sales of intoxicating liquor by a hotel, motel, or inn to any person registered and resident as a guest in such hotel, motel, or inn for consumption upon the premises thereof.”
    “Section 1213. It shall be unlawful for any vendor with a valid state liquor license to sell intoxicating beverages on Sunday. Anyone in violation of this section shall be guilty of a misdemeanor and fined not less than $100 or more than $1,000.”
    The state authorities have regularly enforced the aforementioned dram shop statute and “blue law” prohibiting Sunday liquor sales. At 12:30 a.m. on Sunday morning, a customer entered a liquor store and purchased a bottle of wine. The clerk who made the sale, thought the time was 11:30 p.m. Saturday night because that was the time shown on the store’s wall clock. Unknown to the clerk, the clock was malfunctioning and was running an hour behind. Thus, the clock displayed the wrong time when the clerk sold the wine to the customer. When the clerk made the sale, the customer did not appear to be visibly inebriated. After making the purchase, the customer drank the entire bottle of wine. He then drove off in his car and struck a pedestrian, injuring him. At the time of the accident, the customer was intoxicated from drinking the wine.
    If the pedestrian asserts a claim against the liquor store, will the plaintiff prevail?
    (A) Yes, because the liquor store was negligent per se by selling the wine in violation of the “blue law” prohibiting Sunday liquor sales.
    (B) Yes, because the accident would not have occurred but for the sale of the wine to the customer.
    (C) No, because the purpose of the “blue law” prohibiting Sunday liquor sales was not to prevent accidents caused by drivers driving while intoxicated.
    (D) No, because the clerk did not violate the dram shop statute, and his violation of the “blue law” resulted from the clock’s malfunctioning.
A
  1. (C) Viotation of a statute imposing criminal liability may be “negligent per Se” where:
    (1) the plaintiff is a member of a class of persons sought to be protected by the statute; and (2) the harm suffered was a type the statute was designed to prevent. The pedestrian will not prevail against the liquor store because the purpose of the statute prohibiting the sale of liquor on Sunday was not to prevent drunk driving. If it were, it would have prevented the sale of liquor at more times than just on Sunday morning. Choice (C) is, therefore, correct. Choice (A) is incorrect because negligence per se would not apply unless the plaintiffs injury was the type of harm meant to be prevented by the statute. Choice (B) is incorrect because satisfaction of the “actual cause” or “cause-in-fact” requirement is not, by itself, enough to create liability. Choice (D) is incorrect because the liquor ordinance was a strict liability statute requiring no mens rea, so mistake is no defense.
78
Q
  1. The owner of a drug store hired an independent contractor to renovate the store. Among other things, the renovations included installing a new front door made of plate glass. The contractor purchased the door from a glass door company. The sales representative of the glass door company told the contractor and the owner that plate glass was “super safe” and could withstand considerable pressure. The contractor installed the door, completed the other renovation work, and the store reopened for business later that same month.
    A few weeks later, a shopper and her 9-year- old son went to the drug store to purchase some merchandise. As they approached the store, the son was skipping ahead of his mother. Until he was within a few steps of the door, he thought it was open. When he realized it was, in fact, closed, he slowed down and, while still “going faster than walking,” pushed his hand against the handle to push open the door. His hand slipped off the handle and, at the same time, there was a “kind of exploding noise.” Due to a defect, the glass shattered and broke, cutting the son’s hand and wrist. A subsequent inspection revealed that the glass door had been negligently installed by the contractor.
    If the shopper, on behalf of her son, asserts a claim against the glass door company based on strict products liability, the plaintiff will
    (A) prevail, because the glass door was defective.
    (B) prevail, because the sales representative represented the plate glass as being “super safe.”
    (C) not prevail, because the contractor negligently installed the plate glass door.
    (D) not prevail, because the son negligently pushed open the glass door.
A
  1. (A) Under Section 402A of the Restatement (Second) of Torts, a defendant engaged in the business of selling a product may be held strictly liable for harm caused by the product being in a defective and unreasonably dangerous condition. Here, the glass company can be held strictly liable because the plate glass was defective. Choice (B) is incorrect because, even though strict liability can be imposed on a seller who makes a misrepresentation of a material fact concerning the product, the statement that the plate glass was “super safe” would probably be treated as mere “puffing” or opinion rather than a statement of fact. Choice (C) is incorrect because, even though the contractor is liable for acting negligently, the glass company would still be strictly liable for its sale of a defective product. Choice CD) is incorrect because the son’s negligence would merely reduce his damages under comparative fault principles, rather than barring his claim completely.
79
Q
  1. A grocery store purchased a large mirror to hang on a wall in the store. Along with the mirror itself, the manufacturer provided brackets and bolts to be used for hanging the mirror. The grocery store paid a local carpenter to come in and hang the mirror. A few days after the carpenter installed the mirror, the mirror suddenly fell from the wall because of a defect in the brackets. It struck a shopper on the head, causing serious injuries to him.
    If the shopper asserts a claim against the carpenter based on strict liability in tort, the plaintiff will
    (A) prevail, because the brackets were defective when installed.
    (B) prevail, because it would be abnormally dangerous to maintain such a large mirror on the wall of a retail outlet.
    (C) not prevail, because the carpenter was not engaged in the sale of the mirror.
    (D) not prevail, because the carpenter had no relationship of privily with the plaintiff.
A
  1. (C) Strict tort liability for defective products, under Section 402A of the Restatement (Second) of Torts, applies only to persons engaged in the business of selling the defective product. The carpenter merely installed the mirror and was not engaged in the business of selling the mirror (or any other product), so he cannot be held strictly liable. Choice (A) is incorrect because even if the brackets were defective, the carpenter would not be strictly liable because he did not sell the mirror. Choice (B) is incorrect for a couple of reasons. First, it is not abnormally dangerous to maintain a mirror in a retail establishment. Second, even if it was, the suit was for strict liability in tort, which involves liability for a defective product, not liabiLity for an abnormally dangerous activity. Choice (D) is incorrect because privity of contract between a plaintiff and a defendant is not required for tort claims, including strict liability claims concerning defective products.
80
Q
  1. An accountant had an office on the eighth floor of a downtown building. The accountant wanted to have renovations done to the entrance to the office. He hired an interior designer to do the work. The designer told the accountant that he had several options for the type of glass to be used in the door to the office. He could use half-inch thick plate glass, which would look nice and be relatively inexpensive. On the other hand, he could use tempered glass, which would cost twice as much but would be five times stronger than plate glass, more difficult to scratch, and capable of withstanding a blow from a hammer. The designer explained that the great majority of residential and small business establishments use plate glass for such doors. The accountant chose to go with the plate glass. The accountant ordered a plate glass door from a manufacturer and had the designer install the door when it arrived.
    A few months later, the door shattered when one of the accountant’s clients pulled on the handle to open it. The glass cut the client’s hand.
    If the client asserts a claim against the accountant based on negligence and establishes negligent manufacture of the glass door by the manufacturer that was not discoverable through reasonable inspection, the plaintiff will
    (A) prevail, because the accountant is liable for its supplier’s negligence.
    (B) prevail, because the accountant instructed the designer to install plate glass rather than the more durable tempered glass.
    (C) not prevail, because the accountant could not reasonably have been expected to discover the defect.
    (D) not prevail, because the proper defendant is the manufacturer or the designer, not the accountant.
A
  1. (C) A products liability claim based on negligence requires that the defendant failed to exercise reasonable care in some manner that was an actual and proximate cause of the plaintiff’s injury. This question deals with a situation where negligent manufacture has already been established. To find the accountant Liable for negligence, the plaintiff will have to show that the accountant also failed to exercise reasonable care in some way, such as by failing to detect a defect in the door that a reasonable person would have been expected to discover. Under the facts, the problem with the door could not be discovered through reasonable inspection. Therefore, the accountant is not negligent for failing to discover the defect. Choice (A) is incorrect because negligence is not imputed in the case of seller and supplier. Choice (B) is incorrect because plate glass is generally safe, even though this particular piece was not. Choice (D) is incorrect because the accountant could also be liable for negligence if it is established that he did something unreasonable that led to the client’s injury.
81
Q
  1. Two friends were members of a fraternity at their college. During homecoming weekend, the fraternity held a big party, which was widely attended by students and alumni. At the party, the two friends were horsing around when the first friend pulled out a revolver. He gave the revolver’s cylinder a spin, put the barrel of the gun to the second friend’s head and, intending to scare the second friend, said, “Hey, man, let’s play Russian roulette.” The second friend, who had consumed a fifth of bourbon, was very intoxicated. Thinking that the gun was unloaded, the second friend responded, “Sure, dude.” The first friend then pulled the trigger. The gun just made a clicking sound as the hammer fell on an empty chamber of the cylinder. The two friends both laughed and continued partying.
    Unbeknownst to the second friend, the first friend had placed bullets in two of the revolver’s six firing chambers before making his challenge. The next day, the second friend found out that the gun was, in fact, loaded when the first friend pulled the trigger.
    If the second friend asserts a tort action against the first friend, he will most likely
    (A) prevail, because the first friend intended to place the second friend in apprehension.
    (B) prevail, because there was a substantial certainty that the second friend would be placed in apprehension.
    (C) not prevail, because the second friend was unaware that the gun was loaded.
    (D) not prevail, because the second friend consented to participate in the Russian roulette game.
A
  1. (C) Many students tend to confuse tortious assault with criminal assault. Tortious assault occurs where the defendant does an act with intent to put the plaintiff in apprehension of an imminent harmful or offensive contact, and the result is that the plaintiff is put in apprehension of an imminent harmful or offensive contact. The second friend was unaware that the gun was loaded, so he was not placed in apprehension of an imminent harmful or offensive contact. Therefore, the first friend is not liable for assault, and choice (C) is correct. Choice (A) is incorrect because intent alone is insufficient without the plaintiff being placed in apprehension of imminent harmful or offensive contact. Choice (B) is incorrect because, again, there is no liability for assault unless the plaintiff was put in apprehension of imminent harmful or offensive contact. Choice (D) is incorrect because in order for consent to be a valid defense, the plaintiff must have knowledge about what he is consenting to. The plaintiff only consented to play with an unloaded gun, while, in fact, the gun was loaded.
82
Q
  1. A 10-year-old girl decided to go sledding during a heavy snow and ice storm. She planned to go sledding down Hill Street, a steep and winding thoroughfare. Hill Street, which was located around the corner from the girl’s home, was a popular sledding site because of its precipitous incline. The girl’s parents had given her permission to sled down Hill Street.
    On the way to Hill Street, the girl met a classmate, who was 12, and asked if she wanted to join her. The classmate agreed, and the two girls walked to the top of Hill Street together. There, the youngsters mounted the sled. The girl was sitting in front and controlling the steering mechanism while the classmate was behind her. They started their descent at a relatively good speed.
    As they were about halfway down the road, they approached the intersection of Hill Street and Maple Drive. A driver, who was driving his car along Maple Drive, entered the intersection at the same time as the girls on the sled. When the classmate saw the driver’s auto cross into their path, she shouted to the girl to fall off the sled. The girl became frightened and refused to jump off. Just before the sled collided with the car, the classmate pushed the girl off the sled. Her push caused the girl to fall and break her leg. The classmate couldn’t get off the sled in time and was struck and injured by the driver’s car.
    If the girl’s parents, on behalf of their daughter, assert a claim against the classmate to recover damages for the girl’s broken leg, will they prevail?
    (A) No, if the classmate believed that the girl was in imminent peril when she pushed her off the sled.
    (B) No, unless the classmate acted unreasonably when she pushed the girl off the sled.
    (C) No, if the driver was driving negligently.
    (D) No, because the classmate had the girl’s implied consent to act in an emergency.
A
  1. (B) Courts have recognized that an actor who is confronted with an emergency is not to be held to the standard of conduct normally applied to one who is not in such a situation. This does not mean that a different standard is to be applied in an emergency. The conduct required is still that of a reasonable person under the circumstances. The fact that an emergency demanded very quick action, however, is one of the circumstances to be considered in deciding how a reasonable person would act. A person may be found to be negligent if, under the emergency circumstances, his acts are found to be unreasonable. Based on this rule, choice (B) is the best answer. Even though the classmate was confronted with an emergency, she would, nevertheless, be liable for negligence if she acted unreasonably in pushing the girl off the sled. Choice (A) is incorrect because even if the classmate believed that the girl was in imminent peril, she may still be found negligent if she acted unreasonably under the circumstances. Choice (C) is incorrect because even if the driver was driving negligently, that would not relieve the classmate of liability if her negligence was also a cause of the plaintiffs injury. Choice (D) is incorrect because even if the classmate had the girl’s consent to act during an emergency, it would be consent to reasonable actions and not negligent ones.
83
Q
  1. A child was riding his bicycle on a street, heading from north to south. As he went down a hill, he came to an intersection. At that moment, a car was coming toward the intersection, heading from east to west. When the driver saw the child on the bike, he slammed on his brakes in an attempt to avoid hitting the child. Meanwhile, the child realized that he was in danger of colliding with the car, so he applied the hand brakes on his bike as hard as possible. As a result of trying to stop so suddenly, the child flew over the handlebars of the bike and onto the ground. Neither the child nor his bike touched the car, but the child suffered injuries from falling off the bike.
    If the child’s parents, on behalf of the child, assert a claim against the driver to recover damages for the child’s injuries, they will most likely
    (A) prevail, if the child would have been injured even more severely if he had crashed his bike into the car, rather than falling off the bike.
    (B) prevail, because the child’s action in trying to stop the bike was foreseeable.
    (C) not prevail, because the driver and his car never touched the child or bike.
    (D) not prevail, unless the driver negligently created a risk of harm that caused the child to fall off the bike.
A
  1. (D) A person can be held liable for negligence if he fails to exercise reasonable care under the circumstances, and that is an actual and proximate cause of harm to the plaintiff. Here, the driver would be liable if he negligently created a danger of a collision that caused the child to fall off his bike and suffer injuries. Choice (A) is incorrect because even if the driver forced the child to fall off the bike to avoid more serious injuries from a collision with the car, the driver will only be liable if he is found to have acted negligently. Choice (B) is incorrect because even if the child’s actions were foreseeable, the driver will only be liable if he is found to have acted negligently. Choice (C) is incorrect because even though the driver’s car did not hit the child, the driver can be held liable if his negligence proximately caused the child’s injuries.
84
Q
  1. A young boy and a young girl were playing with a football on the sidewalk next to a street. They were laughing as they pushed and pulled each other, trying to get the ball and keep it away from each other. Just as a car approached on the street, the girl lost her balance and fell off the sidewalk and into the street. The driver of the car tried to swerve to avoid her, but ran over her leg. The girl suffered a broken leg.
    If the girl’s parents assert a claim on her behalf against the driver to recover damages for her injuries, they will most likely
    (A) prevail, unless the driver was driving his vehicle within the posted speed limit.
    (B) prevail, unless the girl fell because the boy negligently pushed her toward the street.
    (C) not prevail, unless the driver was driving negligently when the accident occurred.
    (D) not prevail, unless the driver had the last clear chance to avoid the accident
A
  1. (C) A person can be held liable for negligence if he fails to exercise reasonable care under the circumstances, and that is an actual and proximate cause of harm to the plaintiff. Here, the driverwould be liable if he was negligent and ifthatwas an actual and proximate cause of the girl’s injuries. Choice (A) is incorrect because even if the driver was not speeding or violating any other traffic laws, he may still be liable if he was negligent. Choice (B) is incorrect because the driver may be held liable for negligence even if the boy’s negligence was also a cause of the accident. Choice (D) is incorrect because the “last clear chance” doctrine applies only in the small number of states that still use traditional contributory negligence rules. Unless the question indicates otherwise, you must assume that pure comparative fault applies and, therefore, the doctrine of “Last clear chance” is obsolete.
85
Q
  1. A husband, who was married to a wife, began dating another woman. The husband proposed marriage to the woman, who believed that the husband was single. The woman accepted the husband’s marriage proposal and planned a wedding. The day before the wedding, the husband decided not to go through with the ceremony. He told his wife about his relationship with the woman. He then asked her to tell the woman that they were married and he would no longer be seeing her. The husband knew that the woman had a manic depressive personality and was mentally unstable. When the wife broke the news to the woman, she had an emotional breakdown that required hospitalization.
    The woman sued the husband for breach of contract to marry and intentional infliction of emotional distress. The husband has filed motions to dismiss both causes of action. The court should
    (A) deny the motion to dismiss the claim for intentional infliction of emotional distress, but grant the motion to dismiss the claim for breach of contract because it is against public policy to enforce marriage agreements.
    (B) deny the motion to dismiss the claim for breach of contract to marry, but grant the motion to dismiss the action for infliction of emotional distress, because the husband’s conduct was not extreme and outrageous.
    (C) grant both motions to dismiss.
    (D) deny both motions, because the husband’s conduct was extreme and outrageous and the court may grant relief for breach of contract, as well.
A
  1. (A) The cause of action for breach of a promise to marry has been abolished by the majority of states. With regard to intentional infliction of emotional distress, there must be a showing that the defendant’s conduct is “extreme and outrageous.” One factor that may contribute to conduct being extreme and outrageous is that the defendant knew the plaintiff was especially sensitive or vulnerable to injury through mental distress. The husband was aware of the woman’s mental instability. He, therefore, could be found liable for intentional infliction of emotional distress. Choice (A) is, therefore, correct. Choices (B), (C), and (D) are incorrect because the right of action for breach of promise or contract to marry no longer exists in most states, and the husband could be liable for intentional infliction of emotional distress.
86
Q
  1. Wilshire Street is a public thoroughfare, designated as a one-way street for northbound traffic. Wilshire and Figueroa Streets intersect at right angles. The intersection is controlled by traffic lights. A businessman was driving his car east on Figueroa Street and did not see the traffic light. He entered the intersection at a time when the light was red for eastbound traffic and green for northbound traffic. A woman, in violation of statute, was proceeding south on Wilshire Street, and she entered the intersection without looking for any cross traffic and struck the businessman’s car.
    The businessman suffered damages in the amount of $5,000. The woman, on the other hand, suffered damages in the amount of $1,000. The businessman brought suit against the woman to recover his damages. The woman denied liability and filed a counterclaim against the businessman for her damages. Both drivers were negligent with 30 percent of the negligence attributable to the woman and 70 percent of the negligence attributable to the businessman.
    A statute in the jurisdiction provides: “A failure to exercise ordinary care, which contributes to personal injury or property damage, shall not bar recovery of damages by the injured party, but damages shall be diminished in proportion to the amount of negligence attributable to the person injured. There shall be no set-off of damages between the respective parties.” Which of the following is the appropriate judgment in the case?
    (A) The businessman recovers $5,000 from the woman, and the woman recovers $1,000 from the businessman.
    (B) The businessman recovers $3,500 from the woman, and the woman recovers $300 from the businessman.
    (C) The businessman recovers $1,500 from the woman, and the woman recovers $700 from the businessman.
    (D) The businessman recovers $800 from the woman, and the woman recovers nothing from the businessman.
A
  1. (C) As a general rule, a typical comparative negligence statute (as in this example) has the effect of apportioning damages between the parties according to their respective degrees of fault. In contributory negligence jurisdictions; on the other hand, the plaintiff is precluded from recovering because his own conduct falls below the standard to which he is required to conform. As a result, in contributory negligence jurisdictions, the entire burden of the loss falls on the plaintiff because the plaintiff bears the full risk. In this comparative negligence example, first determine the measure of damages suffered by each party, and then diminish that amount by their respective degrees of fault. The businessman suffered damages in the amount of $5,000, which would then be diminished by 70 percent (orthe degree of fault attributable to him). Therefore, the businessman’s recovery would be $1,500. Similarly, the woman, who was 30 percent at fault, would recover $700 because her damages of $1,000 would be reduced by $300 in proportion to her degree of fault. Choice (A) is incorrect because it suggests that both parties will recover for the full amount of their injuries, overlooking the reduction for their comparative fault. Ctoice (B) is incorrect because the amounts have been calculated using the wrong percentages. The businessman’s recovery should be reduced by 70 percent, not 30 percent, and the woman’s recovery should be reduced by 30 percent, not 70 percent. Choice (D) is incorrect because the statute says that there will be no “setoff’ of damages. This means the businessman will be able to recover $1,500, and the woman will be able to recover $700, rather than having those payments offset against each other, so that the businessman simply receives $800.
87
Q
  1. A truck and a car collided at an intersection. Both drivers were injured, and each one filed a lawsuit against the other for negligence. At trial, the jury found each of the drivers liable for negligence. The jury found that the amount of the truck driver’s injuries was $10,000, and the amount of the car driver’s injuries was $8,000. The jury assigned 50 percent of the fault for the accident to the truck driver and 50 percent of the fault to the car driver.
    Assume that the jurisdiction has a statute in effect that provides: “Contributory negligence shall not bar recovery in an action to recover damages for negligence resulting in injury to person or property if such negligence was not as great as the negligence of the person from whom recovery is sought. Any damages allowed shall be diminished in proportion to the amount of negligence attributable to the person who is seeking recovery.”
    Which of the following is the appropriate judgment in the case?
    (A) The truck driver recovers $10,000, and the car driver recovers $8,000.
    (B) The truck driver recovers $5,000, and the car driver recovers $4,000.
    (C) The truck driver recovers $1,000, and the car driver recovers nothing.
    (D) Neither party recovers anything from the other.
A
  1. (D) This apportionment statute has the effect of limiting recovery to cases where the plaintiff’s negligence is “not as great” as that of the defendant’s. In other words, where the plaintiffs fault is equal to or greater than the defendant’s fault, the plaintiff will recover nothing. Choice (D) is correct because, for the truck driver’s claim, the plaintiffs fault (50 percent) was equal to the defendant’s fault (50 percent). Likewise, for the car driver’s claim, the plaintiff’s fault (50 percent) was also equal to the defendant’s fault (50 percent). Choices (A), (B), and (C) are incorrect because each party’s claim would be barred bythe comparative fault statute in this situation.
88
Q
  1. A driver was having trouble finding a place to park his car near a store at which he needed to make a quick stop to purchase something. He decided to just park along the road next to the store, even though there was not really a lane or space there intended for parking. The driver pulled his car as far off the roadway as he could and figured that it would be fine because he would only be in the store for a few minutes. While the driver was in the store, a car driven by a woman approached the spot where the driver had left his car. When the woman saw the driver’s car and realized it was partially obstructing the road, she tried to avoid hitting it but crashed into it. The accident caused $5,000 worth of damage to the driver’s car and $1,000 worth of damage to the woman’s car.
    Assume that the jurisdiction follows traditional contributory negligence rules and that the woman had the last clear chance to avoid the collision.
    How much, if anything, could the driver recover in a lawsuit against the woman for negligence?
    (A) $5,000, or the full amount of damages to his car.
    (B) $4,000, or the full amount of his damages, minus the woman’s damages.
    (C) $2,500, or half of his damages.
    (D) Nothing, because the driver was contributorily negligent for parking his car so that part of it stuck out into the street.
A
  1. (A) According to the “last clear chance” doctrine, which applies in the small number of states that still use traditional contributory negligence principles, the last wrongdoer is viewed as the worst wrongdoer, or at least the decisive one, and should pay. As a consequence, the driver should recover the full amount of damages if the woman had the last chance to avoid the accident. Choices (B), (C), and (D) are incorrect because the driver will be able to recover the full amount of his damages.
89
Q
  1. A 17-year-old boy was jostling with his 12-year-old brother. A pedestrian witnessed the boy shoving his brother and mistakenly believed that they were fighting. Acting as a Good Samaritan, the pedestrian approached the boys and tried to intercede. The boy stumbled and accidentally fell into the pedestrian, knocking him to the ground. The pedestrian broke his arm as a result of the fall.
    If the pedestrian sues the boy to recover damages for his injury, he should
    (A) prevail, because the pedestrian reasonably believed that the boy was about to injure his brother.
    (B) prevail, based upon the doctrine of transferred intent.
    (C) not prevail, because the pedestrian assumed the risk.
    (D) not prevail, unless the boy was negligent.
A
  1. (D) This is. a rather tricky Torts question dealing with an interplay between negligence and battery. If the boy was throwing a punch at the brother and hit the pedestrian instead, then choice (B) would be correct, and the transferred intent doctrine would be applicable. However, the facts state that “the boy stumbled and accidentally fell into the pedestrian.” Therefore, no intentional tort was committed, and negligence would be the pedestrian’s only possible theory of recovery. Choice CD) is, thus, the correct answer. Choice (A) is incorrect because even if the pedestrian reasonably believed the boy was about to injure the brother, that would not make the boy liable unless he acted negligently. Choice (B) is incorrect because the boy did not have intent to hit or hurt anyone, so there is no intent to be transferred under the doctrine of “transferred intent.” Choice (C) is incorrect because assumption of risk requires that the plaintiff voluntarily exposed himseLf to a known risk of harm. Here, the pedestrian was not aware that the boy would accidentally stumble and fall into him. Moreover, under comparative fault principles, even if the pedestrian did know about the danger of intervening in this situation and acted unreasonably, that would only reduce his damages and not bar his claim completely.
90
Q
  1. A buyer purchased a new dishwasher from an appliance store. This dishwasher had been manufactured by an appliance company. After the dishwasher had been installed in the buyer’s home, it functioned properly for the first month. Then it began to make a loud rattling noise whenever it was in use. The buyer called the appliance store, and a manager there assured her that it was not uncommon for the dishwasher to make such a noise. Moreover, he indicated that the machine’s clamorous sound would eventually disappear with continued use.
    One week later, after a dinner party, the buyer placed her most expensive china set in the dishwasher and turned on the machine. After the dishwasher had been in operation for a few minutes, the buyer heard the rattling noise followed by the sound of breaking china. She immediately stopped the machine. Upon looking inside, she saw that a blade from the dishwasher had broken, destroying her entire china set.
    If the buyer asserts a claim for strict products liability against the appliance company, will she prevail?
    (A) Yes, because the dishwasher was defective.
    (B) Yes, because she was the purchaser of the dishwasher.
    (C) No, because the buyer was not personally injured.
    (D) No, because a reasonably prudent person would have discontinued using the dishwasher after becoming aware of the rattling noise.
A
  1. (A) Under Section 402A of the Restatement (Second) of Torts, a person engaged in the business of selling a product can be held strictly liable if the product causes harm because it is defective and unreasonably dangerous. Choice (A) is, therefore, correct because the buyer can recover for the damage to her property (namely, the china set), even though she herself was not physically injured by the defective dishwasher. Choice (B) is incorrect because it does not matter that the buyer was the purchaser of the product. If the product was defective and caused harm to her, she could hold the manufacturer strictly liable, even if she was not the purchaser of the product. Choice (C) is incorrect because strict Liability for a defective product can apply to property damage as well as personal injuries. Choice (D) is incorrect because even if the buyer’s use of the dishwasher after hearing the noise was unreasonable, that would merely reduce her recovery under comparative fault principles, rather than barring it completely.
91
Q
  1. After graduating from law school, a graduate moved to a city where she had been offered a position in an entertainment law firm. When she arrived, the graduate was told that her job was contingent on passing the state bar examination, which was being offered in July. In preparing for the bar examination, the graduate planned to take a bar review course. Although she had not pre-enrolled, the graduate planned to attend and pay for the course at registration.
    On the morning the course was set to begin, the graduate arrived late after getting stuck in freeway traffic. By the time the graduate got to the course location, registration had already been completed. When the graduate looked into the room where the course was being conducted, she saw the lecture had begun, and everyone was inside busily taking notes. Panicky, the graduate picked up a set of course materials that were lying on the registration table and entered without paying. Although the graduate intended to pay the enrollment fee, no one was at the registration table to take her money. After attending the first couple of classes, the graduate sent a cashier’s check for the full amount of the enrollment fee to the bar review company.
    Unbeknownst to the graduate, the bar review course’s regional director noticed that she had entered the lectures without paying. After ascertaining her identity from a fellow student who knew the graduate, the company filed a complaint with the local prosecutor, and a warrant was issued for her arrest. The complaint charged the graduate with the crime of larceny by trick, for deliberately failing to pay for services rendered. After the complaint was issued, the graduate’s check arrived at the company’s office. The company deposited the check but failed to inform the prosecutor that payment had been received. The graduate was arrested and held in custody overnight before the matter was resolved.
    The graduate suffered severe emotional distress at the publicity she received in the local newspapers and the humiliation she suffered for being arrested and held in police custody. Terribly upset, the graduate was unable to take the bar exam and, thus, lost her job at the entertainment law firm.
    If the graduate asserts a claim against the bar-review company based on infliction of emotional distress, will she prevail?
    (A) Yes, because the graduate did not intend to defraud the company.
    (B) Yes, because the company made no effort to inform the prosecutor that the registration fee had been paid in full.
    (C) No, because the company did not intend to cause the graduate to suffer severe emotional distress.
    (D) No, because filing-the complaint with the prosecutor was not outrageous conduct.
A
  1. (B) Generally speaking, liability for intentional infliction of emotional distress will exist where the defendant’s extreme and outrageous conduct intentionally or recklessly causes severe emotional distress to the plaintiff. The defendant’s conduct must have been so outrageous in character, and so extreme in degree, as to go beyond all possible bounds of decency. In the present case, the company filed a criminal complaint against the graduate, charging her with Larceny. Moreover, an arrest warrant was issued. Under the circumstances, the company was reckless in its deliberate disregard by failing to notify the police that the matter had been resolved. Choice (A) is incorrect because in an action for intentional infliction of emotional distress, the key issue is whether the defendant’s conduct was intentional or reckless, not whether the plaintiffs conduct was intentional. Choices (C) and (D) are incorrect because the company recklessly failed to notify the police that the matter had been resolved.
92
Q
  1. A store owner noticed that an expensive electronic device was missing from the store. He assumed that a shoplifter had taken it. Later that day, the store owner was cleaning up the small room in the back of the store that the employees of the store used to store their personal items while they were at work. The store owner was straightening up the coat rack in the back room, when he accidentally knocked one of the employee’s coats off the hanger. As the coat fell onto the floor, the missing electronic device fell out of the pocket of the coat. It was still in its original packaging, and the store owner checked the serial number on the package to confirm that it was the device missing from the store’s shelves. The store owner knew that the coat belonged to a woman who worked as a cashier in the store. The store owner felt bad, but decided that he had no choice but to call the police. Two police officers arrived at the store. They gathered information from the store owner, who signed a criminal complaint against the woman. The officers then discussed the matter with the woman. Although she denied stealing anything, the officers arrested her and informed her that they would need to take her to the police station. After the police took the woman away, another employee of the store found out that the woman had been arrested. That employee immediately went to the store owner and admitted that he was actually the one who had stolen the device, and that he had hidden it in the woman’s coat, intending to retrieve it later before the end of the woman’s work shift. The store owner promptly called the police and told them that there had been a mistake and that the theft charge against the woman was invalid. The police released the woman, who spent a total of approximately 30 minutes in police custody.
    If the woman asserts a claim for false imprisonment against the arresting officers, will the woman prevail?
    (A) Yes, because the woman had not committed a crime at the time she was arrested.
    (B) Yes, because the woman was arrested and held in custody.
    (C) No, because the woman was detained for only 30 minutes.
    (D) No, because the officers had probable cause to arrest the woman based on the store owner’s complaint and honest belief that the woman had stolen the device.
A
  1. (D) The intentional tort of false imprisonment occurs where the defendant acted with the intent and result of confining the plaintiff within a limited area. To avoid liability for false imprisonment, the arresting officers must show that the arrest was lawfully made with probable cause, reasonably relying upon the store owner’s honest belief that a crime had been committed. Choice (D) is, therefore, the correct answer. Choices (A) and (B) are incorrect because the officers will not be liable for false arrest because the requirements for a vaLid arrest were satisfied, inctuding the existence of probable cause based on the store owner’s belief. Choice (C) is incorrect because false imprisonment can occur even if the plaintiff is confined only for a short time. The confinement must last for an appreciable amount of time, and 30 minutes would be more than enough to be appreciable.
93
Q
  1. A supermarket had just reopened after a six-month renovation period. The renovations included the repair, replastering, and repainting of the entire ceiling and walls by the painting company.
    The day following the reopening of the supermarket, the store’s manager noticed small fragments of plaster on the floor, which appeared to have fallen from a part of the ceiling about 10-square-feet in area. The manager immediately posted signs in that area that read: “Caution: Falling Plaster.” In addition, the manager promptly called the painting company and requested a repairman to attend to the problem.
    That afternoon, a shopper was shopping in the supermarket and wanted to purchase some canned goods in the posted area. Moving quickly, the shopper reached for the desired items. At that very moment, a section of the ceiling fell and struck the shopper, injuring her very seriously.
    If the shopper asserts a claim against the supermarket, she probably will
    (A) prevail, because the supermarket failed to take adequate precaution to ensure the safety of its patrons against such an unreasonable risk of harm.
    (B) prevail, because the supermarket is vicariously liable for the negligence of its independent contractor in the performance of a non-delegable duty.
    (C) not prevail, because the injury was the fault of the painting company.
    (D) not prevail, because the supermarket posted signs warning customers of the risk of harm.
A
  1. (A) A possessor of Land is required to exercise reasonable care to protect invitees, such as customers in a store, from harm. This is a non-delegable duty, meaning that the possessor will be obligated to exercise reasonable care, even where the danger is created by the acts of an independent contractor doing work on the property. Although giving a warning about a danger may be sufficient in some circumstances, reasonable care may require the possessor of the land to do more than just post a warning sign. Here, although the manager posted a warning sign in the potentially dangerous area of the supermarket, more adequate precautions should have been taken under the circumstances. Consequently, the shopper would be entitled to recover damages in negligence against the supermarket. Choice (B) is incorrect because although the supermarket has a non-delegable duty to keep the premises safe for invitees, the supermarket will be held liable for its own failure to fulfill that duty, rather than being vicariously Liable for the contractor’s negligence. Choice (C) is incorrect because the supermarket and the painting company may be liable. Choice (D) is incorrect because posting signs may not be sufficient to constitute reasonable care under the circumstances.
94
Q
  1. A pedestrian was walking down the sidewalk past a building that was being painted. A ladder used by the painters fell and struck the pedestrian, causing an injury to his shoulder.
    In an action by the pedestrian against the painting company based on the theory of negligence, which of the following is the most accurate statement regarding the burden of proof?
    (A) The pedestrian must prove beyond all reasonable doubt that the painting company was negligent.
    (B) The pedestrian must prove by a preponderance of the evidence that the painting company was negligent.
    (C) The painting company must prove by a preponderance of the evidence that it was not negligent.
    (D) There is a rebuttal presumption of negligence on the part of the painting company under the doctrine of res ipsa loquitur.
A
  1. (B) The injured pedestrian must prove by a preponderance of the evidence that the painting company was negligent. In civil actions, unlike criminal prosecutions, the burden of proof does not require that the jury be convinced beyond all reasonable doubt, but only that the jury be persuaded that a preponderance of the evidence is in favor of the party sustaining the burden. Choice (A) is incorrect because reasonabLe doubt is the wrong burden of proof in a civil action. A civil plaintiff must prove negligence by a preponderance of the evidence. Choice (C) is incorrect because the burden of proof in a negligence action is upon the plaintiff, not the defendant. Choice (D) is incorrect because res ipso loquitur does not create a rebuttable presumption of negligence. Instead, it is merely a type of circumstantial evidence creating an inference that the defendant was negligent. The burden of proof does not shift to the defendant and, instead, the plaintiff retains the burden to prove negligence by a preponderance of the evidence.
95
Q
  1. A driver parked his automobile in front of a row of shops, one of which was a barber shop where he planned to get a haircut. Realizing that he did not have any change for the parking meter, the driver entered the closest shop, a convenience store, to get change for a $10 bill. After the driver obtained change from one of the clerks in the convenience store, a section of the store’s ceiling fell on him as he was about to leave the store. The store had failed to discover the weakened condition of the ceiling. The driver suffered serious head injuries from the falling plaster.
    In an action by the driver against the convenience store for negligence, the plaintiff will most likely
    (A) recover, because the store was negligent in failing to inspect its premises and discover dangerous conditions that might harm its invitees.
    (B) recover, because the driver had intended to purchase merchandise when he entered the store.
    (C) not recover, because the driver was merely a licensee in the store.
    (D) not recover, because the store only had an obligation to disclose to invitees any known dangerous conditions on the premises.
A
  1. (A) The owner or occupier of land is under an affirmative duty to protect his business invitees against not only dangers of which he is aware, but also against those that, with reasonable care, he might discover. The basis of liability to the invitee is the implied representation made to the public by holding the land open to them (i.e., that it has been prepared for their reception). Under this theory of liability, even a person who comes onto the premises without the intention of conferring an economic benefit upon the occupier/owner, is classified as an invitee. In this regard, even though the driver did not intend to purchase any items at the store, he would be classified as an invitee because the store is open to the public. Therefore, the driver would recover in negligence as a result of the store’s failure to take adequate safety precautions to protect its business invitees. Choices (B) and (C) are incorrect because the driver would be classified as an invitee, not a licensee, even though the driver did not intend to purchase any items at the store. Choice (D) is incorrect because the duty of the land’s occupier toward invitees is an affirmative duty to protect them, not only against dangers of which he is aware, but also against those that he might discover with reasonable care.
96
Q
  1. A bicycle company manufactured a bicycle that it sold to a retail bicycle dealer, which in turn sold it to a purchaser. Shortly thereafter, while the purchaser was riding the bicycle along a city street, he saw a traffic light facing him turn from green to amber. He sped up, hoping to cross the intersection before the light turned red. However, the purchaser quickly realized that he could not do so and applied the brakes, which failed. To avoid the traffic that was then crossing in front of him, the purchaser turned sharply to his right and onto the sidewalk, where he struck a pedestrian. Both the pedestrian and the purchaser suffered injuries.
    The pedestrian asserted a claim based on negligence against the bicycle company. At trial, it was found that the brake failure resulted from a manufacturing defect in the bicycle, and that the bicycle company failed to conduct a reasonable inspection that would have led to the discovery of the existence of the defect.
    Will the pedestrian prevail?
    (A) Yes, because the bicycle company placed a defective bicycle into the stream of commerce.
    (B) Yes, because the defect could have been discovered through the exercise of reasonable care by the bicycle company.
    (C) No, because the pedestrian was not a purchaser of the bicycle.
    (D) No, because the purchaser was negligent in turning onto the sidewalk.
A
  1. (B) The distinction between negligence and strict liability is important for product liability claims. The manufacturer or seller of a product is Liable for negligence if it fails to exercise reasonable care, and is strictly Liable if the product is defective and unreasonably dangerous. Here, the injured pedestrian is suing the manufacturer for negligence. In the event that the manufacturer failed to exercise reasonable care (or faiLed to make a reasonable inspection) to discover the defect, that would establish negligence on the part of the manufacturer. Choice (A) is incorrect because it addresses strict liability. Choice (C) is incorrect because privity of contract is not required for a negligence (or strict liability) claim. Choice (D) is incorrect because even if the purchaser was negligent, that would not bar the injured pedestrian from holding the manufacturer liable for negligence.
97
Q
  1. A rancher owned a 2,000-acre cattle ranch in a remote section of the state. For some time the rancher’s cattle had been seized upon by roaming packs of wolves. In order to protect his livestock from these attacks, the rancher erected a fence around his property. In addition, the rancher installed electrical wiring on the fence. The wiring transmitted an electric current, which would emit a shock upon contact with the fence. Although the voltage was calculated to frighten animals away from the fence, it was of such low intensity that it posed no risk of injury to a human being, even a small child. Consequently, the rancher did not post any notice of the electrical wiring device.
    One day, a cowboy was riding his horse along the outskirts of the rancher’s property, when a gust of wind blew his hat onto the fence. The cowboy, who had undergone major heart surgery, wore a pacemaker. When the cowboy touched the top of the fence to retrieve his hat, the electric current short- circuited his pacemaker, resulting in the cowboy’s death by electrocution.
    In a wrongful death action by the cowboy’s estate against the rancher, the installation of the electrical wiring device would most likely constitute
    (A) the legal cause of the cowboy’s death, but not the cause-in-fact.
    (B) the cause-in-fact of the cowboy’s death, but not the legal cause.
    (C) the legal cause and the cause-in-fact of the cowboy’s death.
    (D) neither the legal cause nor the cause-in-fact of the cowboy’s death.
A
  1. (B) It is important to understand the difference between proximate, or legal, cause and causation in fact. Ask yourself, “Has the conduct of the defendant caused the plaintiff’s injury?” In other words, would the plaintiff have suffered the same injury but for the defendant’s negligence? This is a question of fact. Once it is established that the defendant’s conduct has, in fact, been one of the causes of the plaintiff’s injury, there remains the question of whether the defendant should be legally responsible for what he has caused. Were the plaintiff and his injury within the scope of the foreseeable risks that made the defendant’s conduct negligent? Here, the rancher’s installation of the electrical wiring device would constitute a cause-in-fact, but not a legal cause, of the cowboy’s death. The rancher’s conduct was the cause-in-fact of the cowboy’s death because the cowboy would not have been injured but for the installation of the electrical wiring device. However, the rancher’s conduct is not a legal cause of the injury because the cowboy is an “unforeseeable” plaintiff outside the zone of foreseeable danger from the defendant’s conduct. The facts state that the electrical wiring device was of such low intensity that it posed no risk of injury to a human being. Therefore, the rancher would not foresee a trespasser with a pacemaker being harmed by the fence. Note: If presented with a question where a city dweller installed an electrical wiring device on a fence bordering the sidewalk with heavy pedestrian traffic, then legal cause might exist, because it would be more foreseeable that a pedestrian with a pacemaker might come into contact with the fence next to a crowded city sidewalk. Choices (A), (C), and (D) are incorrect because the rancher’s installation of the electrical wiring device would be a causein-fact, but not a legal or proximate cause of the cowboy’s death.
98
Q
  1. A train pulled into a busy station. Some passengers got off the train, and others got on. A few moments later, a train conductor shouted, “All aboard!” to indicate that the train was about to depart the station. At that moment, a man, carrying a large cardboard box, came running through the station toward the train “Wait for me!” he shouted as he ran toward the train. The man with the box reached the edge of the platform, next to the train, just as the train started to move. The man hesitated, unsure of whether he should give up or go ahead and jump onto the train even though it was moving. A railroad worker, who happened to be standing next to the train, decided to be helpful. He said, “You can make it,” and gave the man a shove toward the open door of the train. The man stumbled and dropped the box that he was carrying. Instantly, the box made a loud screeching noise. The box contained powerful fireworks, and the impact of the box hitting the ground had caused a large rocket to ignite. The rocket flew out of the box and shot across the train station, glancing off the ceiling and finally crashing into a wall of a small coffee shop on the far side of the train station, about 150 feet from the train. The rocket’s collision caused a shelf on the wall to fall. A large container of coffee that was on the shelf fell and hit a customer who was sitting in the shop and enjoying a cup of coffee. The coffee shop customer suffered an injury to her shoulder as a result of being hit by the container of coffee.
    If the coffee shop customer asserts a negligence claim against the railroad, which of the following elements of her claim will the plaintiff be unable to satisfy?
    (A) Duty.
    (B) Breach.
    (C) Cause-in-fact.
    (D) Proximate or legal causation.
A
  1. (D) This question is based on the famous case of Palsgraf V. Long Island Railroad Co.,
    162 N.E. 99 (N.Y. 1928). In that case, the court concluded that the plaintiff could not recover because she was beyond the scope of the foreseeable danger created by the defendant railroad’s conduct. If the railroad’s employees were negligent for pushing a passenger onto the train, that merely created a foreseeable risk for the passenger and, perhaps, for others standing very close to him. In his majority opinion, Judge Cardozo focused on the element of duty. He asserted that a person only has a duty to exercise reasonable care for the safety of those in foreseeable danger. “The risk reasonably to be perceived defines the duty to be obeyed,” he contended. In his dissent, judge Andrews argued that a person owes a duty of care to everyone, not just to foreseeable plaintiffs. In Andrews’s view, the key issue was really proximate cause, rather than duty. The majority view in modern tort Law is a mixture of Cardozo’s and Andrews’s views. In particular, most courts today analyze a situation Like the one in Palsgraf as an issue of proximate cause, as Andrews suggested, but they use Cardozo’s reasoning to conclude that proximate cause does not exist if the plaintiff was outside the scope of the foreseeable risk created by the defendant’s conduct. Most courts today, therefore, would say that the answer here is (D), and that the element the plaintiff will be unable to establish is proximate cause. Choices (A), (B), and (C) are incorrect because Palsgraf is primarily read today as a case about proximate cause, even though Judge Cardozo saw it more as a duty issue.
99
Q
  1. A person who was traveling checked into a hotel for the night. While he was watching television in his hotel room, a thief picked the lock on the hotel room door, entered, hit the traveler over the head, and stole his wallet. The traveler, who suffered a concussion, was hospitalized for three days. The lock on the hotel room door fully complied with a local ordinance that prescribed minimum security standards for hotel locks.
    If the injured traveler asserts a claim against the hotel for damages for his injury and loss of his wallet, the traveler will
    (A) prevail, if the lock was defectively designed.
    (B) prevail, because he was a business visitor on the hotel’s property.
    (C) not prevail, because the thief’s act was an intervening cause.
    (D) not prevail, if the hotel used reasonable care in selecting the lock.
A
  1. (D) A person is negligent if he fails to exercise the care of a reasonable person underthe circumstances. In this question, the traveler will prevail if the hotel was negligent in selecting a lock that could be picked by a thief. Choice (A) is incorrect because the hotel is not a seller of a product and, therefore, it cannot be held strictly liable even if the lock is defective. Choice (B) is incorrect because the duty owed a business invitee is not an absolute one, but merely a reasonable duty to inspect and remedy unsafe conditions existing on the premises. In other words, the plaintiff will not prevail unless the defendant was negligent. Choice (C) is incorrect because hotel theft is a foreseeable occurrence, and the duty of an innkeeper toward his or her guests requires the use of reasonable care to prevent physical attacks and thefts of property. Only supervening acts break the chain of proximate causation and relieve the defendant of liability.
100
Q
  1. Two volleyball teams that played in adjoining areas were competing in the National Volleyball Tournament Championship in a distant city. Both teams were staying at the same hotel. The assistant coach of one of the teams learned that the team captain of the other team was staying in the next room. In order to obtain information about the other team’s strategy for the championship game, the assistant coach used electronic devices placed against the wall to listen to and record conversations between the team captain and his teammates. The assistant coach later bragged to others about having eavesdropped on the team captain, and the team captain eventually heard about what the assistant coach had done.
    If the team captain asserts a claim against the assistant coach for invasion of privacy, will the team captain prevail?
    (A) Yes, because the assistant coach was recording the conversation for the use and advantage of his team.
    (B) Yes, because the team captain had a reasonable expectation of privacy in his hotel room.
    (C) No, because the assistant coach’s electronic devices did not physically intrude into the team captain’s room.
    (D) No, because there was publication of the recorded conversations.
A
  1. (B) Invasion of privacy occurs where a person intentionally intrudes, physically or otherwise, upon the solitude or seclusion of another person or his affairs. The plaintiff must show that the intrusion would be highly offensive to a reasonable person. The intrusion may involve a physical invasion into the place in which the plaintiff secluded himself, or it may occur through the use of the defendant’s senses, with or without mechanical aids, to oversee or overhear the plaintiff’s private affairs. In this situation, by electronically recording the team captain’s conversations as to team strategy, the assistant coach unreasonably intruded upon the team captain’s private affairs and will be liable. Choice (B) states the correct rationale. Choice (A) is incorrect because it contains a correct statement of fact, but not a legal explanation for why the assistant coach will be liable, so(B) is a better choice. In fact, choice (A) does not address any of the elements needed to satisfy the tort of invasion of privacy. Choice (C) is incorrect because there need not be a physical intrusion for the tort of invasion of privacy. Here, it is an intrusion by an electronic device into a place in which the plaintiff has secluded himself. Choice (D) is incorrect because publication is not required; it is the intrusion that substantiates the invasion of plaintiff’s privacy. Publication is required for a defamation action.
101
Q
  1. A circus operates in a rural part of the county. It is the circus’s practice to finish each day’s entertainment by discharging a so-called aerial bomb into the sky. After exploding, the aerial bomb emits a spectacular rainbow fireworks display that is seen for miles around.
    One afternoon, a 10-year-old boy and a few friends went to the fairgrounds to see the circus. After paying their admission, they were about to enter the “big top” when the boy came upon an aerial bomb lying on the ground. Ignorant of what the object really was, but in an exploratory mood, the boy applied a match to the fuse of the fireworks device. It exploded and seriously injured the boy.
    After the accident, and before the parents of the boy had retained a lawyer, the adjuster for the circus’s insurance carrier contacted the parents several times, trying to negotiate a settlement of their claim against the circus. On each occasion, the adjuster told the parents there was no need for them to hire an attorney because the applicable civil code did not provide for recovery in such a situation. The adjuster was aware that this information was blatantly false. He also warned the parents that unless they accepted his offered settlement of $5,000, they would receive nothing.
    For over a month, the adjuster continued to harass the parents. Outraged by the adjuster’s actions, the parents sought the advice of a neighbor who recommended that they consult an attorney. The parents went ahead and retained a lawyer who subsequently negotiated a $250,000 settlement with the adjuster’s insurance company for the claims relating to the boy’s injury from the aerial bomb.
    If the parents assert a claim against the adjuster to recover damages for infliction of emotional distress, they will
    (A) recover, because the adjuster’s actions exceeded the bounds of common decency.
    (B) recover, because the adjuster was trying to take unfair advantage of the parents.
    (C) not recover, because the parents did not suffer emotional distress that was severe.
    (D) not recover, because the parents eventually received a $250,000 settlement from the adjuster’s insurance company.
A
  1. (C) Intentional infliction of emotional distress occurs where the defendant, through extreme and outrageous conduct, intentionally or recklessly causes severe emotional distress to the plaintiff. Emotional distress passes under various names, such as mental suffering, mental anguish, mental or nervous shock, orthe like. It includes all highly unpleasant mental reactions, such as fright, horror, grief, shame, humiliation, embarrassment, anger, chagrin, disappointment, worry, and nausea. However, liability for intentional infliction of emotional distress exists only if the plaintiff’s distress is truly severe. In the present example, the facts do not indicate that the parents suffered severe emotional distress. The facts simply state that the parents were “outraged by the adjuster’s actions.” As such, choice (C) is correct because the parents will not recover unless they suffered severe emotional distress. Choice (A) is incorrect because even though the adjuster’s conduct was extreme and outrageous, there can be no recovery unless the plaintiffs suffered severe emotional distress. Choice (B) is incorrect because merely trying to take unfair advantage of an individual does not establish infliction of mental distress. Choice (0) is incorrect because it is irrelevant whether the parents received a settlement of the claims for their son’s injury from the aerial bomb. The question presented herein relates to the adjuster’s liability for infliction of emotional distress, which is a separate cause of action from the tort claim for the son’s injuries.
102
Q
  1. A pedestrian was crossing the street when he was hit by a car. The pedestrian suffered a neck injury. The driver of the car that hit the pedestrian had auto insurance. A claims adjuster from that insurance company contacted the pedestrian and discussed the accident with him. The insurance adjuster said, “You’re not actually entitled to any compensation for these injuries, under the laws of this state; but I feel bad for you, and I’m a nice guy, so I’m willing to approve a $1,000 payment to you.” The insurance adjuster knew that, in fact, the pedestrian would be entitled to compensation under the state’s tort law. The pedestrian rejected the offer, retained an attorney, and wound up winning a $550,000 jury verdict on negligence claims against the driver.
    If the pedestrian asserts a claim against the insurance adjuster for misrepresentation, will the pedestrian prevail?
    (A) Yes, because the insurance adjuster’s $1,000 settlement offer was grossly inadequate.
    (B) Yes, because the insurance adjuster knew that laws of the state provided for recovery in such a case.
    (C) No, because the insurance adjuster’s statements did not cause the pedestrian any monetary loss.
    (D) No, because the insurance adjuster did not have a fiduciary relationship requiring him to protect the pedestrian’s interests.
A
  1. (C) The tort of misrepresentation requires a false representation of a material fact known by the defendant to be false, which induces actual and justifiable reliance by the plaintiff, resulting in monetary loss. Here, the insurance adjuster may have made false statements, but the pedestrian did not rely on them or suffer any loss as a result of them. Choices (A) and (B) are incorrect because neither addresses the necessary reliance upon a material fact that results in monetary loss. Choice (D) is incorrect because even though the insurance adjuster did not have a fiduciary relationship with the pedestrian, the insurance adjuster could still be liable for misrepresentation if the pedestrian had justifiably relied on the false statements and been harmed by doing so. The liability would be for misrepresentation, not breach of a fiduciary relationship.
103
Q
  1. A businessman is the owner of a pet products company, which is engaged in the manufacture and sale of a variety of pet supplies. The businessman’s company manufactures such products as pet furniture, toys, beds, collars, leashes, cages, and vitamins. These items are distributed to pet stores throughout the United States and Europe. For many years, the company has conducted its operations from a large factory located in a small town in the southern part of the state. One of the businessman’s biggest selling products is specially manufactured high-frequency dog-calling whistles. These whistles are sold to dog-training schools and canine divisions of many police departments. Although these whistles are not audible to people, they are audible to dogs over considerable distances.
    Two years ago, a breeder purchased an undeveloped lot in the small town in which the company’s factory was located. On her property, the breeder constructed a pet hotel, which was used as a boarding kennel for dogs and cats. This boarding facility was situated about 100 yards from the company’s factory. Unknown to the breeder, high-frequency sound waves often emanated from the company’s factory when dog-calling whistles were being tested. These sound waves caused the breeder’s dogs to howl uncontrollably for many hours during the day and seriously interfered with the operation of her business.
    The breeder now brings an action against the businessman and the company to recover damages for the interference with her business caused by the high-frequency sound that reaches her kennel. The court should rule in favor of
    (A) the businessman, because the breeder came to the nuisance after his factory had already been in operation for a number of years.
    (B) the businessman, because the breeder’s business is abnormally sensitive to harm caused by the high-frequency sound waves.
    (C) the breeder, because the high-frequency sound waves constitute a trespass to her premises.
    (D) the breeder, because the high-frequency sound waves have seriously interfered with the operation of her business.
A
  1. (B) A public nuisance is a substantial, unreasonable interference with a right of the general public. A private nuisance is a substantial, unreasonable interference with a person’s right to the use and enjoyment of land. For either type of nuisance, the interference must be unreasonable, meaning that it would adversely affect a reasonable person and not just someone who is abnormally sensitive. Here, the high- frequency sound waves are a problem only for the breeder because her business (a kennel) is abnormally sensitive to them and, therefore, she cannot prevail on a nuisance claim against the businessman. Choice (A) is incorrect because “coming to the nuisance” is not an absolute defense. It is merely one factor the court will consider in weighing the equities. The fact that the sound waves are only a problem because of the breeder’s abnormal sensitivity is, therefore, a stronger answer. Choice (C) is incorrect because trespass requires the entry of something tangible (although invisible gases and microscopic particles will suffice) onto the land of another. Sound waves are not sufficiently tangible to produce a trespass claim. Choice (0) is incorrect because in order to be considered a nuisance, the disturbance must substantially and unreasonably interfere with the use and enjoyment of one’s land. Here, the company’s testing of the whistles at the factory was not unreasonable, even though the sound waves substantially interfered with the breeder’s business.
104
Q
  1. A well-known Washington newspaper columnist was researching the background of a lawyer who had been nominated to become attorney general of the United States and was awaiting confirmation by the U.S. Senate. One of the columnist’s eager young apprentices concocted a plan that he hoped would enable him to gain the columnist’s favor.
    Late one evening, without telling anyone, the apprentice broke into the lawyer’s private office and copied several letters pertaining to “dirty tricks” perpetrated by the lawyer during the most recent presidential campaign. When the apprentice presented the columnist with the fruits of his diligent research, the columnist was quite impressed and immediately printed excerpts from the material.
    If the lawyer asserts a claim against the columnist for invasion of right to privacy, the plaintiff will most likely
    (A) prevail, because the apprentice’s action was an unlawful invasion of private facts.
    (B) prevail, because the publication was not newsworthy.
    (C) not prevail, because what the columnist printed was true, thus establishing there was no “false light.”
    (D) not prevail, because the columnist was not involved in the burglary and did not conspire with the apprentice with respect to the burglary.
A
  1. (D) Invasion of privacy occurs where a person intentionally intrudes, physically orotherwise, upon the solitude or seclusion of another person or his affairs. Here, although the burglary was certainly an invasion of the lawyer’s privacy, it was the apprentice who did the burglary. Rather than suing the apprentice, the lawyer has asserted a claim against the columnist. The columnist shouLd not be liable for the apprentice’s invasion of the lawyer’s privacy, unless he was a co-conspirator who had agreed to a plan for the burglary. Choice (A) is incorrect because even though the apprentice invaded the lawyer’s privacy, that does not make the columnist liable. Choice (B) is incorrect because if the columnist had been a participant or co-conspirator with respect to the burglary, the fact that the material published was newsworthy would not necessarily save the columnist from liabiLity. Although there is a constitutional privilege to publish private facts if the matter is one of legitimate public interest, it is not an absolute privilege. An action for invasion of privacy may still exist where there is publication of a picture or information that is taken without the plaintiff’s consent from a private place or where it is stolen or obtained by bribery or other inducement of breach of trust. Choice (C) is incorrect because “false light” is only one type of invasion of privacy, and it is not the most relevant type here. If the columnist had participated in the burglary or conspired to have it done, he could be liable for invasion of privacy, even if his publication did not portray the lawyer in a false light.
105
Q
  1. A truck driver negligently changed lanes on the highway without checking to see if it was clear to do so. As a result, a car that was next to the truck was forced off the highway and crashed into a concrete barrier along the roadway. The driver of the car suffered a broken leg. An ambulance arrived and rushed her to the hospital. A doctor there examined the broken leg and ordered X-rays to be taken. The doctor carelessly misread the X-rays and set the broken bone improperly. As a result of the doctor’s negligence, the driver’s leg never healed completely, and the driver was left with a significant impairment of the use of her leg.
    If the driver of the car brings a negligence action against the driver of the truck, the most likely result is
    (A) the truck driver will not be liable for any damages, because the doctor’s negligence constituted a superseding cause that relieved the truck driver of liability.
    (B) the truck driver will be liable for the harm suffered by the plaintiff when her car crashed, but not the additional or enhanced harm resulting from the doctor’s careless mistake.
    (C) the truck driver will be liable only if his negligence is regarded by the jury as being more severe than the doctor’s negligence.
    (D) the truck driver will be liable for all the harm suffered by the plaintiff, although the doctor may be liable to some extent, as well.
A
  1. (D) Where a plaintiff suffers harm caused by a defendant’s negligence, and then the plaintiff’s injury is worsened by negligent medical treatment of the original injury, courts have long held that the subsequent medical malpractice is foreseeable and ordinarily not a superseding cause that relieves the original defendant of liability. The original defendant, thus, can be held liable for all of the plaintiff’s harm, including the enhancement or aggravation of the injury attributable to the medical malpractice. The doctor will be jointly and severally liable along with the original tortfeasor, but the doctor’s liability will be limited to the “extra” harm inflicted by the doctor’s negLigence and will not cover the original injury that the plaintiff suffered before the doctor even became involved in the situation. Under this rule, the truck driver would be liable for all of the plaintiff’s harm, although the doctor may be held liable, as well, for the aggravation of the plaintiff’s injury. Choice (A) is incorrect because the subsequent medical malpractice will be treated as foreseeable, rather than as a superseding cause. Choice (B) is incorrect because rather than being liable only for the original injury, the truck driver, instead, will be liable for all of the plaintiff’s harm, including the part resulting from the subsequent medical malpractice. Choice (C) is incorrect because the truck driver may be held liable regardless of whether his negligence is found to be more or less severe than the doctor’s negligence. Remember: On the Multistate Bar Exam, you should assume that joint and several liability applies unless the question states otherwise. Here, the truck driver alone will be liable for the original harm suffered by the plaintiff in the crash, and the truck driver and doctor will be jointly and severally liable for the aggravation of the injury that occurred because of the negligent medical treatment.
106
Q
  1. A city ordinance forbids washing vehicles parked on city streets. A man washes his car while it is parked on the street in front of his home. The water forms a puddle in the street, which freezes over during the night. Early the next morning, a car driving on the street slides on the ice and crashes into a tree.
    If the driver of the car sues the man and asserts that the man was negligent per Se, which of the following additional facts would help the man the most?
    (A) The man was not aware of the ordinance.
    (B) The city council enacted the ordinance after someone washing his car was hit by a passing truck.
    (C) The driver lives in the man’s neighborhood and saw the man washing his car the day before the accident.
    (D) The driver is not a resident of the city.
A
  1. (B) To decide whether the “breach” element of a negligence claim is satisfied, a jury generally must consider all the facts and circumstances to assess whether the defendant exercised reasonable care. However, where the defendant violated a statute (or regulation, ordinance, etc.), the defendant’s conduct may be deemed to be negligent per Se, meaning that the person’s conduct is automatically treated as negligent. In order for negligence perse to exist, the plaintiff must show that the accident that occurred was the sort of thing the legislature had in mind when it passed the statute. Specifically, the plaintiff must show that (1) the plaintiff is part of the general class of people that the statute was meant to protect; and (2) the plaintiff’s injury was within the general type of harm that the statute was meant to prevent. Here, if the city council enacted the ordinance after someone was hit by a passing truck while washing his car, that would be a strong indication that the ordinance was meant to prevent people from being struck by passing vehicles while washing their cars, and was not meant to prevent accidents like the one that occurred because the water froze into a puddle on the street. If the accident was not the sort of thing the city council had in mind when it enacted the ordinance, negligence perse would not apply. Choice (A) is incorrect because the man’s unawareness of the ordinance would not be a reason for him to avoid Liability. Some excuses can be used to avoid negligence perse, such as if the defendant was unaware of a crucial fact. For example, if you violate a statute that requires cars to have working taillights, but you are unaware that your taillight has burned out, your unawareness of the fact that your taillight went out will be an excuse that avoids negligence per se. Ignorance of the law, however, is a different story and not a valid excuse. Choice (C) is incorrect because the fact that the driver knew the man had washed his car the previous day would not enable the man to avoid liability. At most, implied assumption of risk would reduce the plaintiffs recovery under comparative fault principles, ratherthan barring it completely. In addition, here, implied assumption of risk probably would not even apply, because the driver was not voluntarily choosing to encounter a known risk, because he did not know about the ice even if he did know the man had washed his car the previous day. Choice (D) is incorrect because the driver may invoke negligence per Se, even if he is not a resident of the city. In enacting the ordinance, the city council presumably meant to protect the safety of visitors to the city, as well as city residents.
107
Q
  1. A patient was admitted to a hospital. He had driven his car despite being intoxicated. As a result, he crashed the car into a tree and suffered a broken nose. After a doctor reset the broken nose, the patient was transferred to a room in the west wing of the hospital. In extreme pain, the patient asked the nurse for a painkiller. Without seeking the doctor’s approval, the nurse administered an injection of morphine, which the nurse should have known to be an excessive dosage. The patient died an hour after the injection; the cause of death was a morphine overdose.
    The nurse’s injection of morphine to this patient would most likely constitute
    (A) a cause-in-fact, but not a legal cause of the patient’s death.
    (B) a legal cause, but not a cause-in-fact of the patient’s death.
    (C) a cause-in-fact and a legal cause of the patient’s death.
    (D) neither a legal cause nor a cause-in-fact of the patient’s death.
A
  1. (C) It is helpful to understand the difference between proximate, or legal, cause and causation in fact. First, ask yourself, “Has the conduct of the defendant caused the plaintiffs injury?” This is a question of fact. Note: Although the defendant’s conduct may be the cause-in-fact of the pLaintiffs harm, it (the tortious conduct) may or may not be the proximate cause. However, if the defendant’s actions constitute the proximate cause, then it must, also, always be the cause-in-fact (of the plaintiffs injury). Here, the nurse’s injection of the fatal dosage of morphine would be (1) the cause-in-fact, and (2) the legal, or proximate, cause of the patient’s death. Choice (A) is incorrect because the death was a very foreseeable result of the nurse’s action and, therefore, the nurse’s negligence was the legal, or proximate, cause of the death. Choice (B) is incorrect because if the defendant’s actions constitute the proximate cause, then it must, also, always be the cause-in-fact of the plaintiffs injury. Choice (D) is incorrect because the nurse’s action was both a cause-in-fact (because the patient would not have died but for the overdose) and a proximate, or Legal, cause of the patient’s death (because the death was a foreseeable result of administering a fatal dose of morphine).
108
Q
  1. Late one night, an accountant walked into a bar and ordered a whiskey sour. The bartender served the drink, even though the accountant looked and acted as though he was already very intoxicated. The accountant soon had consumed five more cocktails, which the bartender served, despite the accountant’s obviously and unmistakably drunken condition.
    After finishing his sixth drink in the bar, the accountant said good night to the bartender, staggered out of the bar, got into his car, and drove away. After weaving back and forth across the road for several blocks, the accountant crashed his car into a pedestrian who was walking on the sidewalk next to the road. The pedestrian suffered serious injuries to his back and legs.
    The bartender’s act of serving drinks to the accountant would most likely be viewed as the
    (A) proximate cause of the pedestrian’s injuries.
    (B) superseding cause of the pedestrian’s injuries.
    (C) direct cause of the pedestrian’s injuries.
    (D) intervening cause of the pedestrian’s injuries.
A
  1. (A) The traditional common Law rule was that a person who provided alcohol to a minor or a visibly intoxicated person was not liable for resulting harm. The customary explanation was that the drinker was the real cause of the harm, and the alcohol provider was too remote to be treated as a proximate cause of the injuries. A few states enacted “dram shop statutes,” which imposed liability on those who serve alcoholto minorsorvisiblyintoxicated people. Even in stateswithoutsuch statutes, however, the majority of courts eventually rejected the traditional rule in favor of holdingthat alcohol providers can be held liable forforeseeable harm resultingfrom serving alcohol to minors or visibly intoxicated people. Here, the bartender should have reasonably foreseen that by continuing to serve alcohol to an intoxicated customer, the drinker might endanger himself and others (especially in the operation of his motor vehicle). As a result, the bartender’s conduct would be viewed as the proximate cause of the pedestrian’s injuries because the automobile accident was a foreseeable intervening force for which the bartender must bear responsibiLity. Choices (B), (C), and (D) are incorrect because the bartender’s actions were the proximate cause, not the superseding, direct, or intervening cause of the pedestrian’s injuries.
109
Q
  1. A 15-year-old boy was sledding down a pathway through a wooded area of land owned by a woman. The boy had frequently used the pathway for sledding in the previous months. The path, made of concrete, led through the woods from a public highway to a pond in the rear of the woman’s property. The pathway was used for sledding and the pond for skating by the residents of the neighboring areas, without the consent of the woman. Furthermore, the woman failed to post any signs forbidding trespassing.
    After the boy had been sledding down the pathway for approximately three hours one morning, he lost control of the sled, and steered it into a tree. The boy suffered serious injuries in the crash.
    If a suit is brought on the boy’s behalf against the woman for the boy’s personal injuries, the plaintiff will
    (A) recover, under the “attractive nuisance” doctrine.
    (B) recover, because the woman knew or should have known of the frequent trespasses.
    (C) not recover, because the boy was a trespasser.
    (Dj) not recover, because a 15-year-old boy should have realized the risk.
A
  1. (D) A 15-year-old boy who had been sledding on this pathway for a number of previous months should have realized the risk involved in sledding there. In order to recover under the “attractive nuisance” or “infant trespasser” doctrine, it is necessary that the child, because of youth, did not discover the dangerous condition or realize the risk involved. Choice (A) is incorrect because the attractive nuisance doctrine does not apply where the child discovered the dangerous condition or realized the risk involved. Choice (B) is incorrect because even if the woman knew or should have known of the frequent trespasses, she would not be liable if the boy should have realized the risk. Choice (C) is incorrect because the boy’s status is not, by itself, enough to bar the claim. He could recover under the attractive nuisance doctrine despite being a trespasser, but not here, because he should have realized the risk here.
110
Q
  1. A farmer has a large field where he grows corn. He hires a crop-duster to spray the field, but the crop- duster mistakenly sprays a strong herbicide, rather than a pesticide, and kills all the corn plants in the field. Several days later, a construction company building a house near the farmer’s field negligently hits a gas line and starts a fire that burns everything in the area, including the farmer’s field.
    If the farmer brings negligence claims against the crop-duster and the construction company in order to recover damages for the destruction of his corn crop, the most likely result is
    (A) only the crop-duster is liable, because its negligence occurred before the construction company’s negligence.
    (B) only the construction company is liable, because its negligence would have destroyed the farmer’s corn crop no matter what the crop- duster did.
    (C) both the crop-duster and the construction company are liable, because each engaged in a negligent action that was sufficient to destroy the farmer’s corn crop.
    (D) neither the crop-duster nor the construction company is liable, because neither is a “but for” cause of the harm.
A
  1. (A) In general, a defendant is liable only if its negligence is a “but for” cause of the plaintiffs injury. That means the plaintiff’s injury wouLd not have occurred but for the defendant’s negligence. Special rules apply, however, in situations where there are two (or more) things that combine to cause the plaintiff’s injury, and any one of them would have been sufficient by itself to cause the injury. In these “duplicative cause” situations, neither thing is actually a but for cause of the plaintiff’s injury, but each will be treated as a cause of the plaintiff’s injury if it was a substantial factor contributing to the occurrence of the injury. For example, if two defendants negligently start separate fires near your property, both fires then combine to burn down your house, and either one of them would have been sufficient by itself to cause the damage, each of the two fires will be treated as an actual cause, even though neither was really a but for or essential cause of the harm. In the duplicative cause situations, two or more causes happen at the same time. On the other hand, a “preemptive cause” scenario occurs where one thing causes the plaintiffs injury, and then another potential cause of the same occurs, but it is too late, because it does not take effect until all the harm has already been inflicted. In that situation, only the first thing will be treated as a cause of the plaintiff’s injury. This question involves a preemptive cause, rather than a duplicative cause situation. The crop- duster’s mistake killed all the corn plants, and the harm was already done before the construction company’s negligence and the fire occurred. Therefore, only the crop-duster will be liable because its negligence occurred before the construction company’s negligence. Choice (B) is incorrect because the construction company’s negligence happened too late to be a cause of the farmer’s loss of his corn crop. Choice (C) is incorrect because even though both the crop-duster and construction company were negligent, the crop-duster’s negligence occurred first and inflicted all the harm before. The construction company’s negligence was too late to cause any of the harm. Choice (D) is incorrect because even though actual cause is generally analyzed through the but for test, special rules apply in duplicative cause and preemptive cause situations. Here, the crop-duster’s negligence will be treated as an actual cause of the farmer’s loss, even though it was not really a but for cause, because the construction company’s negligence would have been sufficient to cause the same damage.
111
Q
  1. A man and a woman are adjoining landowners in an area of large estates located in the “chateau” region of southeastern Louisiana. In 2000, the man inherited his estate from his father, whose family had owned the property continuously since 1812. The woman purchased her estate in 2008. The man had a stable, which housed many valuable racing horses, on his property. The stable, built in 2002, stood on a portion of the man’s land located only 10 feet from the border of the woman’s property. Not infrequently, putrid stenches arose from the man’s property, caused by large accumulations of horse manure, which were left unattended. The woman had often complained to the man of the noises and odors emanating from the stable.
    The woman brought an appropriate action to enjoin the man’s use of the stables. Judgment is likely to be for whom?
    (A) The man, because the woman moved onto her property after the stable had been built.
    (B) The man, because a homeowner is entitled to make reasonable use of his property.
    (C) The woman, because the noise, coupled with the odors, substantially interfered with the use and enjoyment of her land.
    (D) The woman, because the man was negligent in permitting the manure to be left unattended.
A
  1. (C) Offensive odors or loud noises may constitute a private nuisance when they unreasonably and substantially interfere with the use and enjoyment of the plaintiff’s land. Choice (A) is incorrect because “coming to the nuisance” (i.e., the fact that the plaintiff moved in after the stable had been built) is not a valid defense to a private nuisance claim and, instead, is merely a factor that a court would consider in deciding whether to grant an injunction. Choice (B) is incorrect because although a property owner is entitled to make reasonable use of property, if that use creates a substantial interference with the use and enjoyment of another’s land, it may be enjoined. Choice (D) is incorrect because in order for an injunction to be granted, the crucial consideration is if there was a substantial interference with the plaintiffs use and enjoyment of his property, not whether the defendant was negligent.
112
Q
  1. A man and a woman owned adjoining pieces of land. The man moved a giant magnolia tree from another section of his property and had it replanted on his property at a spot just 10 feet from the woman’s property. At that time, the woman protested that by locating the tree so close to her land, the man was increasing the risk of injury to the woman’s greenhouse, in which the woman cultivated prizewinning flowers.
    Hurricanes are quite common in the area. Several
    years after the replanting of the tree, in the midst of
    a hurricane, the tree fell on the woman’s greenhouse,
    destroying rare and valuable plants valued at
    $25,000.
    The woman asserted a claim against the man to recover damages for the harm caused by the tree falling on the greenhouse. During trial, the only evidence the woman presented was that the hurricane uprooted the tree, causing it to fall onto her property, and thereby resulting in the damage as claimed.
    At the end of the woman’s case, the man moved for a directed verdict. The man’s motion will most likely be
    (A) granted, because the woman did not produce any evidence to show that the man was negligent.
    (B) granted, because the woman’s damages resulted from an act of God.
    (C) denied, because hurricanes were common in the area.
    (D) denied, because the trier of fact may still infer liability for trespass.
A
  1. (A) Recently, questions dealing with directed verdicts and summary judgments have appeared on the Multistate. Although Civil Procedure is still not a primary Multistate subject, it can be tested in a “crossover” or secondary fashion. A motion for summary judgment is a pretrial procedure of “going behind the pleadings” to determine whether a genuine dispute actually exists. To obtain a summary judgment, the moving party must show that no genuine issue of material fact exists. A directed verdict, on the other hand, may be requested at the end of a plaintiff’s case or after all the evidence is completed. The moving party is basically arguing that the evidence clearly reveals that he must prevail and that there is no reason to send the case to the jury. In deciding whether a directed verdict should be granted, all the evidence is viewed in the light most favorable to the non-moving party. In the present example, the woman is asserting a claim against the man to recover damages caused by the tree falling on her property. In all likelihood, the proper cause of action would be for trespass. In order to prevail, the woman must prove that the man intentionally, negligently, or as a result of an abnormally dangerous activity, caused the tree to enter her land. Obviously, the man did not intentionally cause the tree to fall, because it was uprooted during a storm, nor was the man engaged in an ultrahazardous or abnormally dangerous activity. Therefore, in order for the woman to prevail, she must prove that the man negligently caused the tree to fall. Because the woman did not produce such evidence, choice (A) is correct. Choices (B), (C), and (D) are incorrect because in order for the woman to prevail, she must prove that the man negligently caused the tree to fall.
113
Q
  1. For many years, a husband and a wife tried to have a child, but the wife was unable to get pregnant. Her family physician had advised her that she was infertile and that it was impossible for her to become pregnant. After missing her period for three months and experiencing other symptoms of pregnancy, the wife consulted the physician. Without administering a pregnancy test, the physician examined the wife and concluded that she had the flu. He prescribed tetracycline, an antibiotic drug, which the wife took for two weeks.
    After discontinuing the tetracycline, the wife continued to experience nausea, fatigue, and other symptoms of pregnancy. She then consulted a different doctor, who administered a pregnancy test, which revealed that the wife was, in fact, four months pregnant. Thereafter, she gave birth to a child. When the child developed teeth, they were black and discolored. At the age of 12, the child learned that the black discoloration of his teeth resulted from the tetracycline that the wife took during her pregnancy.
    If a claim is brought on the child’s behalf against the physician based on malpractice in not administering a pregnancy test to the woman and prescribing tetracycline, judgment is likely to be for whom?
    (A) The physician, because an unborn child does not have legal rights stemming from conduct that occurred before birth.
    (B) The physician, because no duty of care is owed to an unborn child not in existence at the time medical treatment is rendered.
    (C) The child, because a child, if born alive, is permitted to maintain an action for the consequences of prenatal injuries.
    (D) The child, because the wife was negligent in failing to seek proper prenatal care.
A
  1. (C) As a general rule, a surviving child has a right to recover for tortiously inflicted prenatal injuries. While foreseeability of future injury alone does not establish the existence of a duty owing to an unborn infant by its mother’s physicians, it is now beyond dispute that in the case of negligence resulting in prenatal injuries, both the mother and the child in utero may be directly injured and are owed a duty. See Albala v. City of New York, 54 N.Y.2d 269 (1981). Furthermore, the case of Highson v. St. Frances Hospital, 459 NY 2d 814 (1983) followed these principles in holding that a cognizable and independent cause of action exists on behalf of the infant in utero who is born alive, against a physician for prenatal injuries. Because the discoloration of the child’s teeth was proximately caused by the physician’s failure to administer a pregnancy test and by his prescription of tetracycline to his pregnant mother, the child may successfully maintain a cause of action for malpractice against the physician. Therefore, choice (C) is correct. Choices (A) and (B) are incorrect because an unborn child does have the rightto maintain an action fortortiously inflicted prenatal injuries as a surviving child. Choice (D) presumes that if the wife was negligent in failing to seek proper prenatal care, that would eliminate the action against the physician. However the negligence on the part of the physician would in all probability be the proximate cause of the malpractice action, making (C) a better choice than (D).
114
Q
  1. A graduate of law school received notice that she had successfully passed the bar exam. To celebrate passing, the graduate went out with a few friends to a popular campus bar. The graduate’s friend ordered a round of Hula Girls for everyone. A Hula Girl is an extremely potent alcoholic drink consisting. of 2 ounces of dry gin, 1 ounce of French vermouth, and 1 teaspoon of grenadine. After the drink is mixed, it is then served in a glass with an inverted lemon peel, which is cut at the ends to represent a grass skirt.
    Although the graduate had never heard of a Hula Girl, she was in a festive mood and drank it, anyway. A few minutes later, the graduate became very dizzy and attempted to stand up from the barstool. As she tried to walk to the restroom, she became nauseated and vomited over a customer sitting at a nearby table. The customer was embarrassed and greatly humiliated by the incident.
    If the customer asserts a claim against the graduate, the plaintiff will most likely
    (A) prevail, because the graduate’s conduct was extreme and outrageous.
    (B) prevail, because an offensive touching resulted.
    (C) not prevail, because the graduate’s actions were involuntary.
    (D) not prevail, because the graduate was unaware what she was drinking.
A
  1. (C) A person is subject to liability for battery if she acts with the intention of causing a harmful or offensive contact with the person of the other, and if an offensive or harmful contact directly or indirectly results. If offensive contact occurs unintentionally, then there is no liability for battery. Intent exists if the defendant had the purpose or desire of achieving the harmfuL or offensive contact or if the defendant acted with actual knowledge that the harmful or offensive contact was substantially certain to occur. Here, the graduate vomited involuntarily. She did not desire to vomit on the customer, nor could she have known with substantial certainty that one drink would make her sick. She thus lacked the requisite intent to be liable for tortious battery. Choice (A) is incorrect because the act was unintentional. Intent is required for the tort of intentional infliction of emotional distress. Choice (B) is incorrect because, even though an offensive touching resulted, there was no intent to cause that offensive touching. Choice (D) is incorrect because the graduate’s lack of awareness of what.she was drinking may be negligence, but the act was involuntary, thereby lacking the intent necessary for a battery.
115
Q
  1. A woman and her 4-year-old son were Christmas shopping at a toy store. The toy store sells a complete array of toys, games, dolls, hobbies, and crafts. The items were displayed on a variety of tables and shelves, which were easily accessible to the customers. While the woman was walking down one of the aisles, her attention became focused on a doll that was prominently exhibited on an overhead display shelf. When the woman approached the doll display, she reached up to grab the doll. As she did so, the woman failed to see a baseball lying on the floor. She tripped over the baseball and fell down, fracturing her hip.
    If the woman asserts a claim against the toy store for her injuries, will the doctrine of res ipsa loquitur apply on the issue of whether the toy store was negligent and responsible for the baseball being on the floor?
    (A) Yes, because the woman was a business invitee on the premises of the toy store.
    (B) Yes, because the toy store was in control of the premises at the time of the accident.
    (C) No, because another customer may have caused the baseball to be on the floor.
    (D) No, because the baseball was an intervening act that cuts off the toy store’s liability.
A
  1. (C) For res ipsa loquitur to create an inference of negligence, (1) the event must be of a kind that ordinarily does not occur in the absence of someone’s negligence; (2) it must be caused by an agency or instrumentality within the exclusive control of the defendant; and (3) it must not have been due to any voluntary action or contribution on the part of the plaintiff. In applying the doctrine of res ipsa loquitur to the issue of the toy store’s liability for the woman’s fall, the second requirement is crucial, namely, “Was the baseball within the exclusive control of the defendant?” Although the toy store had control of the premises in general, that does not mean it had exclusive control of everything in the store. Another customer may have caused the baseball to be on the floor and, thus, res ipsa loquitur would not apply here. Choice (A) is incorrect because the fact that the plaintiff was an invitee and the store had a duty to make its premises safe is not sufficient to make resipsaloquitur apply. Choice (B) is incorrect because even though the toy store was in control of its premises, it was not in exclusive control of the particular instrumentality—the baseball on the floor—that caused harm here. Choice (D) is incorrect because the baseball would be an existing condition and, therefore, part of the “set stage,” rather than an intervening act.
116
Q
  1. A student lent his classmate his car to attend a concert at a theater. Before going to the theater, the classmate drove to a dealer’s home to buy some marijuana. The dealer lived approximately two miles from the theater. After picking up the marijuana, the classmate then drove to the concert. As he was driving to the concert, the classmate smoked two marijuana cigarettes so that he could be “high” for the show. While the car was parked outside the theater, through no fault of the classmate, it was struck by another car and damaged. Repairs will cost $750. The driver of the vehicle that struck the student’s car fled the scene and cannot be identified.
    If the student asserts a claim against the classmate for the damage to the student’s car, the plaintiff will recover
    (A) the value of the car before it was damaged because the car was damaged while under the classmate’s dominion and control.
    (B) the value of the car before it was damaged because the classmate used the car for a purpose other than that for which it was lent.
    (C) the cost of repairing the car because the car was damaged while under the classmate’s dominion and control.
    (D) nothing, because the classmate was not negligent in causing the car’s damage.
A
  1. (D) The key to Multistate (or other multiple-choice) testing is always to remember that the test maker’s main goal is to hide the correct answer! Here, for example, the test maker is trying to mislead students into thinking that this hypothetical situation involves either trespass to chattels or conversion. On the other hand, this is a straightforward negligence question. The student lent the classmate his car to attend a concert. The classmate drove the carto the concert where it was struck. The distracter, or “red herring,” intended to confuse you is that the classmate went on a diversion and purchased marijuana. However, the accident did not occurwhile he was making unauthorized use of the vehicle. As such, there is no liability for either conversion or trespass to chattels. Exam Tip: In analyzing Torts questions, remember that an essential element of the plaintiff’s cause of action for negligence—or for any other tort, for that matter—is that there must be some reasonable connection (i.e., causation) between the defendant’s act and the damage that the plaintiff has suffered. In sum, the classmate’s diversion was not the cause of the accident. Choices (A), (B), and (C) are incorrect because there was no conversion or trespass to chattels, and so the classmate would not be liable because he was not negligent in a way that caused the car to be damaged.
117
Q
  1. A repairman repaired damaged electrical power lines and replaced old, worn-out equipment whenever necessary for the local municipal electrical company. After a violent tornado had ripped through the city, the electrical company was busily trying to restore electrical power to its customers.
    The morning after the storm, the repairman was perched on a ladder trying to repair a high-voltage power line. As he was removing the cover of the transformer box, the ladder suddenly slipped on the wet ground, even though it had been properly fixed into position. The repairman struggled to maintain his balance and, luckily, the ladder came to rest against the transformer box. A pedestrian, who was walking on the street below, saw the repairman’s predicament and began to climb the ladder to aid him. As the pedestrian was ascending the ladder, his foot slipped, and he fell to the ground. His fall caused the ladder to jar loose the transformer box, which in turn sent the repairman falling to the pavement. The repairman and the pedestrian both suffered serious injuries.
    If the pedestrian asserts a claim against the electrical company for his injuries, he will most likely
    (A) prevail, because he was attempting to rescue the repairman.
    (B) prevail, because a public utility company is strictly liable in tort.
    (C) not prevail, because he was negligent in climbing the ladder.
    (D) not prevail, because the repairman was not negligent in not affixing the ladder properly.
A
  1. (D) When a city performs a service that might as well be provided by a private corporation, and particularly when it collects revenue from it, the function is considered a “proprietary” one, as to which there may be liability for the torts of municipal agents within the scope of their employment. This is true where the city supplies water, gas, or electricity, or where it operates a ferry, an airport, or a public market. The municipal electrical company, therefore, can be liable if its employee’s conduct was negligent. Since the facts indicate that the repairman had properly fixed the ‘adder in place, Choice CD) is the best answer. Choice (A) is incorrect because the plaintiff will not necessarily be able to recover just because he was attempting a rescue. Under the “rescue doctrine” in tort law, the fact that a person will come to the rescue in an emergency is deemed to be foreseeable. Foreseeability alone, however, is not enough to produce liability. The plaintiff here would need to show that the defendant was negligent, not just that the plaintiff’s rescue attempt was foreseeable. Choice (B) is incorrect because there is no basis for strict liability here. The electric company was not selling a defective product. Even if repairing a highvoltage power line might be considered an abnormally dangerous activity, most courts would not impose strict liability for that on a municipally owned company. Choice (C) is incorrect because even if the rescuer were negligent, that would not bar him from recovering if the defendant were also negligent. The plaintiff’s negligence would merely reduce the plaintiff’s recovery under comparative fault principles. In some states, the plaintiff’s recovery would not even be reduced because a rescuer’s fault would be ignored unless the rescuer was grossly or wantonly negligent.
118
Q
  1. A pedestrian, who was walking along Chestnut Street at 10:20 p.m. on the night of December 3, urgently needed to find a restroom. Just ahead, the pedestrian noticed a private dinner club. As the pedestrian approached the club, he noticed a sign over the front door that read: “ADMITTANCE TO MEMBERS ONLY.” Although the pedestrian was not a member of the exclusive club, he entered the dimly lit club, found a door marked “Gentlemen,” and entered the restroom.
    Which of the following would best describe the pedestrian’s legal status when he was in the restroom?
    (A) Trespasser.
    (B) Guest.
    (C) Licensee.
    (D) Invitee.
A
  1. (A) A trespasser is someone who enters land without the owner’s express or implied permission. In the present case, the pedestrian would be classified as a trespasser because he entered the premises without the club’s permission. Choices (B), (C), and (D) are incorrect because the pedestrian entered the premises without permission and would be classified as a trespasser. Guests, licensees, and invitees enter with some permissive status.
119
Q
  1. A man was out taking a walk one evening when he realized that he needed to use a bathroom. The closest building was a private gymnasium. The man approached the building, and saw a sign on the door that said, “Members only—No restroom facilities available for non-members.” The owner of the gym hung up the sign because he knew that people frequently entered the gym just to use the restroom, and he was annoyed by this. However, the owner of the gym knew that the sign was not effective and that people still regularly entered the gym just to use the restroom.
    The man needed to use the bathroom very urgently, so he entered the building despite the sign. The restrooms were near the front door of the gymnasium. The man walked directly to the restroom as soon as he entered the building. The clerk working at the front desk in the gymnasium was busy and did not notice the man enter the building or the restroom.
    After making use of the restroom facilities, the man washed his hands and proceeded to turn on the electric blow dryer. The dryer, because of a malfunctioning heating coil, emitted intense heat, which caused severe burns to the man’s hands.
    The man was unaware of the fact that 15 minutes earlier, a gym member had received similar injuries from the malfunctioning dryer and notified the gym’s owner. The owner immediately taped a “DO NOT USE” sign to the dryer. However, the sign had fallen to the floor and was lying face down under the bathroom sink when the man was making use of the restroom.
    Which of the following would best describe the duty of care that the gym owed to the man?
    (A) No duty of care.
    (B) A duty to inspect the premises for unknown dangers and to disclose their existence to others.
    (C) A duty to warn of any known dangerous conditions on the premises.
    (D) An absolute duty of care.
A
  1. (C) As a general rule, a possessor of land owes no duty of care to undiscovered orunanticipated trespassers. However, once a landowner discovers the presence of a trespasser (or should know of the likelihood of trespassers), then there is a duty to warn the trespasser of any dangerous conditions known to the occupier or owner of the property. In the present case, the gym owner knew of the existence of a dangerous condition (namely, the malfunctioning of the electric dryer in the restroom). As a result, the defendant owed a duty to warn trespassers of this dangerous condition. Choice (A) is incorrect because the presence of trespassers in the restrooms was known or foreseeable to the gym owner. Choice (B) is incorrect because the man was a trespasser, so the duty owed to him is to warn of any known dangers, not a duty to inspect. Choice (D) is incorrect because it is the wrong duty owed to a trespasser. The duty is not absolute, but rather the duty owed is to warn of any known dangers.
120
Q
  1. A seat in a movie theater collapsed, causing the customer sitting in that seat to fall to the ground and suffer a back injury.
    In a personal injury action by the customer against the theater, the customer will most likely
    (A) recover, because the theater was under an absolute duty of care to make the premises safe for the protection of its customers.
    (B) recover, only if the theater had prior knowledge of the dangerous condition of the seat.
    (C) not recover, because the theater was under no obligation to inspect the premises to discover unknown dangers.
    (D) not recover, unless the theater failed to make a reasonable inspection of the seats for the safety of its customers.
A
  1. (D) A person who enters the premises upon business that concerns the occupier, and upon his invitation (express or implied), is an invitee. The occupant of the property is under an affirmative duty to protect invitees not only against dangers of which the occupant knows, but also against those, which with reasonable care, he might discover. The invitee is sometimes called a business visitor; the typical example is the customer in a store or the patron of a restaurant, bank, theater, fair, or other place of amusement. The occupier is not an insurer of the safety of invitees; he must not only warn of latent dangers that the occupier knows of, but he must also inspect the premises to discover possible dangerous conditions of which he does not know. Choice (A) is incorrect because the duty owed to an invitee is to warn and inspect for dangerous conditions, but it is not an absolute duty. Choice (B) is incorrect because the customerwill not recover only if the theater had prior knowledge of the dangerous condition, but also if the theater failed to make a reasonable inspection of the premises. In this respect, the theater has an affirmative duty to inspect the premises and to make them reasonably safe for the protection of its customers. Choice (C) is incorrect because the theater was under a duty to make a reasonable inspection of the safety of the premises for its customers.
121
Q
  1. A customer in a restaurant ordered a bowl of the restaurant’s famous homemade Manhattan clam chowder. While eating the chowder, the customer broke a tooth on a pebble in the soup, which a reasonable inspection would not have discovered. When the customer complained to the restaurant’s owner, the owner admitted that the chowder was not, in fact, homemade, but that it had been poured from a can of chowder purchased from a wholesale distributor of food products.
    In a strict liability action by the customer against the restaurant, he will most likely
    (A) recover, if a reasonable consumer would not expect the presence of such a pebble in the chowder.
    (B) recover, but only if the restaurant had received prior notice of the defective condition of the soup.
    (C) not recover, because a reasonable inspection of the soup would not have disclosed the existence of the pebble.
    (D) not recover, because the soup would not constitute an unreasonably dangerous product when it was served to the customer.
A
  1. (A) In cases where a person is injured because something was in food that was not supposed to be there, a “reasonable consumer expectations” test is used in most states. Many years ago, some courts drew a rigid distinction between “foreign” and “natural” items in food. If the item was natural (such as a bone or piece of shell hidden in a dish of food), the defendant who prepared the food would not be liable. If the item was foreign, however, then LiabiLity could be imposed. Most courts now have rejected the “foreign/natural” test and, instead, ask whether the presence of the harmful item in the food would have been anticipated by a reasonable consumer. If a reasonabLe consumer wouLd not have expected a pebble to be in the chowder, then the plaintiff will be able to recover for his injured tooth. Choice (B) is incorrect because there is no requirement that the defendant have prior notice of the defect. Choice (C) is incorrect because strict liability is available in these circumstances. The chowder would be a defective and unreasonably dangerous product, and so the plaintiff would be able to recover under strict liability even though a reasonable inspection by the restaurant would not have led to discovery of the pebble. Choice (D) is incorrect because Liability could be imposed on the restaurant if the presence of the pebble in the chowder was something that a reasonable consumer would not expect.
122
Q
  1. A law clerk normally worked from 9:00 a.m. to
    5:00 p.m. each day, or an average of 40 hours per week. One morning, the law clerk came to the office at 9:00 a.m. and started preparing a brief for an upcoming trial. After completing his work at 5:00 p.m. that afternoon, he was about to leave the office when the senior partner of the law firm summoned him. The senior partner told the law clerk that the law firm was representing an important client on an urgent matter that needed immediate research. The law clerk was advised that he would be required to stay at the law office that night and prepare a memorandum. When the law clerk hemmed and hawed, the senior partner handed him the file and said, “I don’t care if you stay all night, but you better have this memo on my desk by 8:00 a.m. tomorrow.”
    Following the senior partner’s instructions, the law clerk stayed at the office until 2:00 a.m. preparing the memorandum. By the time he finished, the law clerk was totally exhausted after having worked a total of 17 hours that day. Afterward, the law clerk left the office and started to drive home. Because of his fatigue, he didn’t see a pedestrian crossing the street. His car struck the pedestrian, seriously injuring herS
    The pedestrian has asserted a tort action against the law firm to recover damages for her injuries. Which of the following is the most likely result?
    (A) The pedestrian prevails, because the law firm was negligent for requiring the law clerk to stay at the law office and finish the memorandum.
    (B) The pedestrian prevails, because the law firm is legally responsible for its employees’ actions.
    (C) The pedestrian loses, because the actions of the law firm were not a cause-in-fact of the pedestrian’s injuries.
    (D) The pedestrian loses, because the law firm is not responsible for the law clerk’s negligence in these circumstances.
A
  1. (D) Under the doctrine of respondeat superior, an employer can be held vicariously liable for torts committed by its employees within the scope of their employment. For example, if a furniture company has an employee driving a truck around town to deliver furniture to its customers, the company will be vicariously liable if the truck driver’s negligent driving causes harm to someone. On the other hand, when employees are merely commuting to and from work, they are not acting within the scope of their employment, and their empLoyer is not liable if they drive negligentLy. Here, the Law firm will not be Liable for the accident that occurred while the law clerk was driving home from work. Choice (A) is incorrect because an auto accident was not a foreseeable risk of the Law firm requiring the law clerk to work late for one night. Choice (B) is incorrect because an employer is only LiabLe for torts of its empLoyees committed within the scope of their employment, not for aLl the employees’ actions. Choice (C) is incorrect because causation is just one element of negligence, and is not by itself enough to make the employer Liable.
123
Q
  1. An employee wdrked as a delivery person for a drugstore. As part of the employee’s duties and responsibilities, he would regularly deliver prescriptions and other items (such as toiletries, cosmetics, vitamins, and gift products) to customers. One afternoon while the employee was on duty, he remembered that it was his girlfriend’s birthday. He went ahead and bought her a bottle of perfume from the pharmacy’s cosmetics department. The employee paid the full price of $79.95 for the perfume and had the perfume gift wrapped. He then drove to the girlfriend’s house in the company van to personally deliver the birthday present. This trip took place during the employee’s regular working hours while he was en route to make another delivery from his van. As he was traveling to the girlfriend’s house, he was in such a hurry that he drove through a red light and collided with a vehicle owned and operated by a driver. The driver, who had entered the intersection on the green light, tried unsuccessfully to swerve and stop but was unable to avoid the employee’s vehicle. The driver was injured in the accident, which caused extensive damage to both vehicles.
    If the driver brings suit against the drugstore to recover damages arising from the accident, the driver will probably
    (A) prevail, because the employee paid for the perfume.
    (B) prevail, because the employee was under the control and direction of the drugstore at the time of the accident.
    (C) not prevail, because the employee was acting outside the scope of the employment relationship at the time of the accident.
    (D) not prevail, because the drugstore was not negligent in hiring the employee.
A
  1. (B) Under the doctrine of respondeat superior, a master is vicariousLy liable for the torts of a servant committed within the scope of employment. The servant’s conduct is within the scope of empLoyment ifit is of the kind the servant is employed to perform, occurs substantially within the authorized limit of time and space, and is actuated, at least in part, by a purpose to serve the master. Generally, an empLoyer will be liable even if the injury occurs during an employee’s temporary departure from the performance of the work, provided that the deviation from the work was the sort of thing that is foreseeable and expected of employees. Even though the employee was making an unauthorized deLivery to his girlfriend at the time he drove through the red Light and injured the driver, he was still en route from the drugstore to make another separate, authorized deLivery. The drugstore, therefore, can be held Liable for the employee’s negligence. Choice (A) is incorrect because the fact that the empLoyee paid for the perfume has no bearing on the fact that there was at the time of the accident a master-servant relationship that existed between the employee and the drugstore. Choice (C) is incorrect because the employee was not acting outside the scope of employment, since his deviation from his work was minor and foreseeable. Choice (D) is incorrect because the drugstore can be liable under the doctrine of respondeat superior, even if it was not negligent in hiring the employee.
124
Q
  1. A furniture store had a truck that an employee of the store used to deliver furniture to the store’s customers. One afternoon while the employee was driving the truck to deliver a chair to a customer, the employee negligently ran through a stop sign and hit a car. The driver of the car suffered a leg injury and succeeded in suing the furniture store, recovering $25,000. Assume that all of these events occur in a jurisdiction that has statutes permitting defendants to bring claims for contribution and indemnity.
    In the event that the furniture store brings suit against its employee (the truck driver) to recover for its losses, the furniture store will recover
    (A) nothing, because the furniture store was primarily liable for the entire amount of damages.
    (B) $12,500, because the employer and employee bear equal shares of responsibility for the plaintiff’s injuries.
    (C) $25,000, because the employee was at fault in causing the accident.
    (D) $25,000, unless the furniture store was fully insured against such losses.
A
  1. (C) When one party (such as an employer) is held vicariously liable for harm caused by the negligent conduct of another (such as an employee), the party held vicariously liable is entitled to indemnification from the tortfeasor. Indemnification means that the full amount of the loss ultimately will be paid by the tortfeasor who was most directly responsible for causing it. The furniture store was held vicariously liable for its employee’s negligence and, therefore, it is entitled to be indemnified by the employee for the full $25,000 loss. Choice (A) is incorrect because through indemnification, the furniture store’s loss may be shifted to the employee, the party who was primarily at fault. Choice (B) is incorrect because the employer is entitled to full indemnification, not just contribution or partial reimbursement for some of the loss. Choice (D) is incorrect because the furniture store is entitled to indemnification regardless of whether it had insurance covering the amount of the judgment won by the injured plaintiff.
125
Q
  1. A 20-year-old student at a state university enrolled in the university’s R.O.T.C (Reserve Officers’ Training Corps.) program. As part of her R.O.T.C. training, the student was required to spend three weeks at a camp for Marine Corps. summer field exercises. While at the camp, the student was given a new synthetic helmet to wear during her basic training.
    An agency purchased the new high-technology helmets for the Army and Marine Corps. to replace the old “steel pot” headgear worn for decades by U.S. soldiers. These new synthetic helmets were manufactured by a corporation and were made of Kevlar, an extremely tough fiber material with high energy-absorbing qualities that made it stronger in some ways than steel.
    When the student received her helmet from the Marine Corps., it was packaged in the original carton supplied by the corporation. On the box, there was a picture that depicted an off-duty marine wearing the helmet while riding a bicycle. One afternoon after training, the student decided to take a ride on her bicycle. Believing that it could be used as a bicycle helmet, the student decided to use it for that purpose.
    During the ride, the student hit a deep pothole that caused her to be thrown headfirst from the bicycle. As she was flying through the air, the strap from her helmet came loose, and the helmet fell off her head. The student landed on a lawn and was temporarily knocked unconscious. She suffered serious head injuries.
    If the student asserts a claim against the corporation to recover damages for her injuries, she will most likely
    (A) prevail, because under the doctrine of res ipsa loquitur negligence can be inferred from the fact that the helmet came off the student’s head.
    (B) prevail, because the original package depicted a picture of a cyclist wearing the helmet.
    (C) not prevail, because the student was not using the helmet for its intended purpose.
    (D) not prevail, because the helmet had been properly designed by the corporation for its intended use by the Army and Marine Corps.
A
  1. (B) The student’s product liability cause of action will be predicated upon the theory of express warranty due to the fact that the box depicted an off-duty marine wearing the military headgear while riding a bicycle. The student relied upon this pictorial representation as an assertion that the helmet was suitable for this use. As a result, the corporation will be held liable for the student’s misuse of the product under this products liability theory. Choice (A) is incorrect because there is no inference of negligence on the part of the corporation. The mere fact that the helmet came off the student’s head in a bicycle accident does not necessarily establish an inference of negligence. Choice (C) is incorrect because in the area of products liability, a manufacturer must be held to foresee a certain amount of misuse or carelessness (e.g., standing on a chair) on the part of the user. Choice (D) is not the best answer because, even though the helmet may have been properly designed for its intended use, the fact remains that some misuse is foreseeable, especially here, when the box itself has this depiction on it. Moreover, the student is not necessarily arguing that the helmet was improperly designed; rather, she would be suing under an express warranty theory, so the fact that the helmet was properly designed is a fact that is irrelevant and will not help the corporation.
126
Q
  1. A doctor parked her car in a public parking lot in a downtown area. A construction company was using a large crane to build a new office building next to the parking lot. When the crane was manufactured, some of the welding of its parts was done improperly. No one had ever noticed this manufacturing defect in the crane. The defect caused the crane to break and topple over. Part of the crane fell on the doctor’s car.
    The doctor’s car was completely destroyed. The car was valued at $35,000. The doctor had auto insurance, and the insurance company promptly paid the doctor $35,000 under the insurance policy.
    If the doctor brings suit against the manufacturer of the crane, will the doctor prevail?
    (A) No, because the doctor had auto insurance, so she suffered no loss.
    (B) No, because the manufacturer was unaware of the defect in the crane.
    (C) Yes, because the manufacturer sold a product with a dangerous defect.
    (D) Yes, because the manufacturer failed to inspect the crane adequately before selling it.
A
  1. (C) The fact that an injured plaintiff has insurance covering a loss does not prevent liability from being imposed on a tortfeasor who caused the plaintiff’s injury. The insurance company may wind up receiving the money that the plaintiff wins in the lawsuit, such asthrough subrogation principles underwhich the insurance company “stands in the shoes” of the plaintiff and recovers for the loss for which it provided coverage. That is a matter between the insurance company and the insured, however, and it does not affect the tortfeasor’s liability. Choice (C) is correct because the manufacturer will be strictly liable if it sold a defective and unreasonably dangerous product. Choice (A) is incorrect because a tortfeasor is liable for damages even though the plaintiff had insurance covering the property damaged. Choice (B) is incorrect because strict liability for selling a defective product applies even if the manufacturer was not aware of the defect. Choice (D) is incorrect because the manufacturer will be strictly liable regardless of whether it failed to inspect the crane adequately or did anything else that was negligent.
127
Q
  1. An owner ran a sporting goods store that specialized in hunting and camping supplies. His friend had often cautioned him that he should not leave so much of his merchandise out in the open. One day, as the friend entered the store to pay a visit, he decided he’d put his unheeded warnings into action by playing a joke on the owner. The friend took a hatchet that was lying on a display case and swung it over his head as if he were going to strike the owner, who was standing at the register with his back turned. The friend then said, “All right buddy, one false move and it’s over.” The owner was very frightened at almost being hit with the hatchet but suffered no actual physical or other harm.
    If the owner asserts a claim against the friend, the most likely result is that he will
    (A) recover, because the friend was negligent.
    (B) recover, because he feared the friend would hit him.
    (C) not recover, because he suffered no physical harm.
    (D) not recover, because the friend was only joking.
A
  1. (B) In this situation, the friend would be liable forthe intentionaltort of assault because the owner feared that he would hit him. Simply defined, assault is an act by the defendant that creates a reasonable apprehension in the plaintiff of immediate harmful or offensive contact to the plaintiff’s person. Choice (A) is incorrect because the friend is liable forthe intentional tort of assault, not for negligence. A negligence claim would require proof of some actual harm. Choice (C) is incorrect because it is not necessary to prove actual damages to sustain an action for assault. Choice (D) is incorrect because a defendant can be liable for assault even if he was merely trying to play a joke.
128
Q
  1. A security guard had a license to carry a concealed handgun. One day, he was walking past a barber shop when he saw his friend inside getting a haircut. The security guard knew that the barber, an elderly man, had a history of significant heart problems. The security guard decided to play a joke on his friend. The security guard took out his gun, burst through the door of the barber shop, pointed the gun at the friend, and shouted, “Don’t move or I’ll shoot!” The barber, thinking that an armed intruder was about to shoot the friend, suffered a heart attack and collapsed.
    If the barber brings suit against the security guard for infliction of emotional distress and seeks recovery for damages arising from the heart attack, he should
    (A) prevail, because the security guard knew the barber was present.
    (B) prevail, because the security guard assaulted the friend.
    (C) not prevail, because the barber was not a member of the friend’s immediate family.
    (D) not prevail, because the resulting harm was unforeseeable.
A
  1. (A) In a basic “intentional infliction of emotional distress” scenario, the defendant does something extreme and outrageous, and the plaintiff is the target of that act. A trickier situation arises where the plaintiff was not the target of the defendant’s extreme and outrageous act, and the plaintiff, instead, suffered severe emotional distress because of something the defendant did to a third person. The plaintiff can recover for intentional infliction of emotional distress in such a situation if either of the following two requirements is present: (1) the defendant knew the plaintiff was present, and the plaintiff was a close family member of the person who was the target of the defendant’s extreme and outrageous conduct, or (2) the defendant knew the plaintiff was present, and what the plaintiff witnessed was so shocking that it caused the plaintiff to suffer actual physical harm, such as a heart attack or stroke. Because the security guard knew the barber was present, and the barber suffered a heart attack from seeing the security guard pretend that he was about to shoot the friend, the barber will be able to hold the security guard liable for intentional infliction of emotional distress. Choice (B) is incorrect because the fact that the security guard could be liable to the friend for assault does not determine whether the security guard could be liable to the barber for intentional infliction of emotional distress. Choice (C) is incorrect because even if the barber is not a member of the friend’s family, he can recover for intentional infliction of emotional distress because he suffered a bodily injury when he had a heart attack. Choice (D) is incorrect because foreseeability of the harm is not an element of an intentional infliction of emotional distress claim. Furthermore, in any event, the security guard knew the barber had a bad heart, so the resulting injury here was very foreseeable.
129
Q
  1. A city resident commuted to work each day by subway. After purchasing her subway ticket, the resident entered the subway for her ride downtown to where she was employed. Looking for a seat, the resident was walking toward the rear of the subway car when it came to an abrupt halt. As a result of the sudden stop, the resident fell onto a commuter, injuring him.
    If the commuter asserts a claim against the subway company to recover for his personal injuries, the commuter will probably
    (A) prevail, because a special duty of care was owed him.
    (B) prevail, because the sudden stop caused the resident to fall onto him.
    (C) not prevail, if the operator of the subway was not negligent in making the stop.
    (D) not prevail, if the resident was negligent in not holding onto the hand rail.
A
  1. (C) In order that a negligent actor shall be liable for another’s harm, it is necessary not only that the actor’s conduct be negligent toward the other, but also that the negligence of the actor be a legal cause of the other’s harm. In the present case, if the subway was not negligent, then it cannot be held liable. There are situations in which the actor is under a special responsibility to protect the plaintiff from the intentional, or even criminal, misconduct of others. Among such relations are those of (1) carrier and passenger, (2) innkeeper and guest, (3) employer and employee, (4) invitor and business visitor, (5) school district and pupil, and (6) bailee and bailor, among others. However, in such situations, the defendant is under a duty to exercise reasonable care. Choice (A) is incorrect because if the subway was not negligent, then it did not fail to exercise reasonable care. Choice (B) is incorrect because while it is factually correct that the sudden stop caused the resident to fall onto the commuter, the subway company will not be liable unless there was negligence. Choice (D) is incorrect because the commuter could prevail against the subway company if the resident was negligent. That is because of the special relationship that exists between a common carrier and its passengers and its duty to exercise reasonable care in protecting those passengers.
130
Q
  1. A hunter was on his property one day looking for rabbits and other small game, which he shot occasionally for sport. As he rounded a clump of bushes, he spotted a hiker, who, he thought, was a man wanted by the police. The hiker, who had his back to the hunter, was carrying a rifle on his shoulder. The hunter called out to the hiker to stop. The hiker was startled and, as he turned around, his rifle fell forward so that it pointed directly at the hunter. The hunter, thinking the hiker was about to shoot him, fired his rifle at the hiker. The bullet missed the hiker and hit a trespasser on the property. The hunter was aware that people often walked onto his land because there was a pond adjoining the property, which provided boating and fishing activities.
    If the trespasser asserts a claim against the hunter for battery, the trespasser will
    (A) recover, because the hunter intended to hit the hiker.
    (B) recover, because the trespasser suffered a harmful and offensive contact.
    (C) not recover, because the hunter accidentally shot the trespasser.
    (D) not recover, because the hunter reasonably acted in self-defense.
A
  1. (D) When a person has reasonable grounds to believe that he is being or is about to be attacked, he may use such force as is reasonably necessary for protection against the potential injury. The actor need only have a reasonable belief as to the other party’s actions. In other words, apparent necessity, not actual necessity, is sufficient. As a result, reasonable mistake as to the existence of the danger does not vitiate the defense. Choice (D) is, therefore, correct because the hunter reasonably acted in self-defense when he shot at the hiker. Note that if, in the course of reasonably defending himself, one accidentally injures a bystander, he is nevertheless protected by the defense. Choice (A) is incorrect because it refers to the doctrine of transferred intent, which is not applicable here because the hunter acted reasonably in self-defense. Choice (B) is incorrect because even though the trespasser suffered a harmful and offensive contact, the hunter reasonably acted in self-defense, which would preclude the trespasser from recovering under a battery theory. Choice (C) is incorrect because although the hunter accidentally shot the trespasser, the hunter acted reasonably in self-defense when he shot at the hiker.
131
Q
  1. A person was taking a long hike on a trail through a forest in a state park. As the hiker came around a bend in the trail, he suddenly saw a hunter standing next to the trail, looking in the other direction. The hunter was holding a rifle. The hunter turned, causing the rifle to be pointed at the hiker. The hiker felt sure that he was going to get shot, but the gun did not go off.
    The hiker asserts a claim for assault against the hunter. In his action, the most likely result is that the hiker will
    (A) recover, because the hunter’s rifle was pointed directly at him.
    (B) recover, because the hunter’s decision to carry the gun was voluntary.
    (C) not recover, unless the hunter intended to scare the hiker.
    (D) not recover, if the hunter did not intend to shoot the hiker.
A
  1. (C) To be liable for assault, the defendant must have intended to put the plaintiff in apprehension of imminent harmful or offensive contact. It is not essential that the defendant intended for the contact actually to occur; it is enough that the defendant intended for the plaintiff to think that the contact would occur. Therefore, the hunter will not be liable for assault unless he intended to frighten the hiker. Choice (A) is incorrect because the fact that the rifle was pointed at the hiker is not enough to create liability without the required intent. Choice (B) is incorrect because even though the hunter decided to carry the gun, he will not be liable for assault unless he intended to put the hiker in apprehension of imminent harmful or offensive contact. Choice (D) is incorrect because the hunter would be liable for assault if he intended to frighten the hiker, regardless of whether he actually intended to shoot him.
132
Q
  1. A certified airline pilot owned a single-engine Cessna plane. One afternoon, the pilot invited his girlfriend, to go flying with him. The girlfriend, who was scared of flying, reluctantly agreed. During the flight, the pilot decided to play a practical joke and shut off the plane’s engine. When the plane went into a nosedive, the pilot said, “Oh my God, we’re going to crash.” The girlfriend became panic- stricken and started screaming hysterically. A few seconds later, the pilot re-started the engine and said, “Don’t worry—I was only joking.” The plane landed safely and neither person was injured.
    If the girlfriend brings suit against the pilot for intentional infliction of emotional distress, she will probably
    (A) recover, because a reasonable person would have been distressed by the pilot’s actions.
    (B) recover, because the pilot’s conduct was extreme and outrageous.
    (C) not recover, because the pilot intended his actions as a practical joke and, therefore, lacked sufficient intent for an intentional tort.
    (D) not recover, because the girlfriend did not suffer any physical injury.
A
  1. (B) One who intentionally or recklessly causes severe emotional distress to another by extreme and outrageous conduct is subject to liability for such emotional distress and, if bodily harm results, for such bodily harm as well. In the present case, the pilot’s conduct was extreme and outrageous. Furthermore, the girlfriend suffered sufficient emotional distress because the facts indicate that she was panic stricken. Exam Tip: For bar exam purposes, emotional distress includes any of the following types of mental suffering: fright, horror, grief, shame, humiliation, embarrassment, anger, chagrin, disappointment, worry, and nausea. Choice (A) is incorrect because the tort of intentional infliction of emotional distress requires severe and outrageous conduct by the defendant, not just something a reasonable person would find distressing. Choice (C) is incorrect because the pilot should have been substantially certain that the girlfriend would suffer emotional distress, and substantial certainty can establish the intent element for an intentional tort. Choice (D) is incorrect because physical injury is not a required element for intentional infliction of emotional distress.
133
Q
  1. A professor was employed by a bar review course as a lecturer. After lecturing for the bar review course for 10 years, the professor was suddenly fired.
    Because the professor was such a popular and well-respected individual in the bar review industry, there was much speculation and interest concerning his termination. A reporter for the local law journal telephoned the bar review course’s administrative director and inquired why the professor had been fired. The director told the reporter that the professor was fired because the president of the bar review course believed that the professor had embezzled money. The next day, the reporter wrote an article that was published by the law journal, which quoted the director and stated that the professor “was fired because he had embezzled money.” It was later learned that the professor was not responsible for any unlawful conduct and had not embezzled funds.
    If the professor asserts a defamation action against the law journal, he will most likely
    (A) prevail, if the professor proves malice.
    (B) prevail, if the newspaper was negligent in not
    ascertaining whether the professor had actually embezzled any funds.
    (C) not prevail, because the director was merely providing an opinion about the reason for the professor’s termination.
    (D) not prevail, because the law journal was merely repeating what it had been told by a source.
A
  1. (B) For many defamation issues, determining whether the plaintiff is a public or private figure is cruciaL A public figure is an individual who has (1) achieved pervasive fame or notoriety (e.g., a movie star or celebrity sports figure), or (2) has voluntarily injected himself/herself into a public controversy. A plaintiff who is a public figure will be required to prove malice in order to recover for defamation. Malice means that the defendant either knew the allegedly defamatory statements were false or acted with reckless disregard for whether they were true or false. Here, the professor is properly viewed as a private figure, even though he may be well known within the bar review industry. Because he is a private figure, the plaintiff merely needs to prove that the newspaper acted negligently, rather than that it acted with malice. Choice (A) is incorrect because it suggests the plaintiff must prove malice here, which is not true because the plaintiff is a private figure. Choices (C) and (D) are incorrect because they suggest the newspaper cannot be held liable, but the plaintiff here can recover if he proves the newspaper was negligent.
134
Q
  1. A college student was walking home from a party. He had been drinking alcohol at the party and was very intoxicated. Local police officers stopped the student and questioned him. When he refused to answer any of their questions, the police put the student in the back seat of a police car. The police left the student in the back seat of the police car for approximately one hour, and then released him.
    If the student asserts a claim against the police for false imprisonment, he will most likely
    (A) prevail, unless the police made a valid arrest.
    (B) prevail, if he consented to the confinement because he was intoxicated.
    (C) not prevail, because police cannot be held liable for false imprisonment.
    (D) not prevail, because the student suffered no harm.
A
  1. (A) As a general rule, a person detained by the police may recover for false imprisonment unless there was a lawful arrest. Choice (B) is incorrect because consent bars liability for an intentional tort only if the plaintiff was capable of giving valid consent. If the student was highly intoxicated, and the police were aware of that fact, the student’s consent would be invalid. Choice (C) is incorrect because the police can be held liable for false imprisonment under some circumstances, such as if they detain someone without a valid reason and without following proper arrest procedures. Choice (D) is incorrect because a defendant may be held liable for false imprisonment even if the plaintiff suffered no specific harm other than being aware of being confined.
135
Q
  1. At 1:00 a.m. one night, two police officers were driving past a tavern when they noticed a man vomiting outside. They stopped their patrol car and walked over to the man, whom they recognized as a troublemaker from the neighboring town. Realizing that the man was extremely intoxicated, they grabbed him and shoved him into their patrol car.
    They drove the man around in the police car for a few minutes and told him he had two choices. They would either take him to jail for the night or drive him to the city limits and drop him off there. The man indicated that he didn’t want to go to jail. One of the officers then said, “Does that mean you want us to drop you off at the city limits?” The man replied, “Yes.” They then drove him to the city limits and left him along a busy highway. Moments later, the man staggered onto the street and was struck by a car driven by a motorist. He suffered multiple injuries.
    If the man asserts a claim against the police department based on negligence, he will most likely
    (A) recover, because the police should have realized it was dangerous to drop him off where they did.
    (B) recover, because the police knew that the man was intoxicated when they dropped him off.
    (C) not recover, because the man chose to be driven to the city limits.
    (D) not recover, because the motorist’s act was a supervening cause of his injury.
A
  1. (A) Negligence is failure to exercise the care of a reasonable person under. the circumstances. The police were negligent because they should have realized it was dangerous to drop off the man where they did. Choice (B) is incorrect because knowledge of the plaintiffs intoxication is only a factor in determining whether the defendant’s actions were negligent. Choice (A) is a better, more complete answer as to why the police were negligent. Choice (C) is incorrect because the plaintiff’s choice does not bar liability from being imposed for the negligent actions taken by the police. Choice (D) is incorrect because the motorist’s act was a very foreseeable consequence of the police officers’ actions, rather than a superseding cause that would bar the officers’ liability.
136
Q
  1. A 35-year-old man with some experience as a truck driver owned a lumber truck. One day, the man set out driving his truck, heavily loaded with lumber, down a mountain road. Sitting next to the man in the passenger seat was a 19-year-old helper. During the course of the trip, when the truck was going down a long hill, the brakes failed. The man shouted to the helper to jump, but the teenager refused to do so and shouted back to the man that he should try to steer the truck down the hill. The man then opened the door on the passenger’s side of the truck and negligently pushed the helper out. The helper, who suffered a broken leg, was rushed to the hospital where he was treated for his injury. As the helper was recuperating, a nurse inadvertently mixed up his chart with that of the female patient in the next room. The nurse gave the helper a fertility pill that made him sterile.
    If the helper brings suit against the man to recover damages for his sterility, the man will
    (A) recover, because it is foreseeable that a hospital can be negligent in its care of patients.
    (B) recover, because the man was negligent in pushing the helper out of the truck.
    (C) not recover, because the nurse’s act was a supervening superseding cause.
    (D) not recover, because it is not foreseeable that a patient would be injured in such a manner.
A
  1. (B) This is a tricky question dealing with intervening causation. Where a defendant causes injury to a plaintiff, and then subsequent medical malpractice causes the plaintiff’s injury to be worsened, courts will treat the subsequent medical malpractice as a foreseeable result of the defendant’s original negligence. Even if the particular act of medical malpractice in a specific situation may not really be foreseeable, it is generally foreseeable that sometimes medical malpractice will occur, and so courts will treat the subsequent medical malpractice as foreseeable and hold the original tortfeasor responsible for the plaintiff’s entire injury. So here, since the man’s conduct in pushing the helper out of the truck was negligent, the man will be liable forthe helper’s injuries, includingthe sterility resulting from the nurse’s error. Choice (A) is a weaker answer because although it contains a correct statement about the foreseeability issue, it does not address whether the man was negligent. If the man was not negligent in pushing the helper, then the man would not be liable for the sterility. Choice (C) is incorrect because if the man was negligent, he can be held liable for the sterility, and the nurse’s error will not be a supervening cause that breaks the link between the man’s original negligence and the sterility. Choice (D) is incorrect because courts will treat subsequent medicaL malpractice as being foreseeable.
137
Q
  1. An elevator company installed an elevator in an apartment building. Under the terms of its contract with the owner of the apartment building, the company agreed to carry out regular monthly inspections of the elevator and to make all necessary repairs. The elevator was installed in the apartment complex in January. For the next 10 months, the company made regular inspections. However, starting in November, the company began to have some difficulties with its employees and failed to send inspectors out for three successive months.
    On February 20, a woman went to visit her friend, who was a tenant on the 20th floor of the building. The woman used the intercom to call up to the friend, who buzzed to unlock the front door so that the woman could enter the building. The woman headed to the elevator. When the doors opened, several people were already inside the elevator, apparently having entered the elevator from the parking garage below the building. One of the people in the elevator was a man who acted shifty and nervous. By the 12th floor, only four persons were left, including the woman and the man. At that floor, the other two people left, leaving the woman and the man alone on the elevator. Apprehensive at finding herself alone with such an unkempt stranger, the woman pressed the 13th floor button, intending to step out and hoping to find other persons on that floor.
    Unfortunately, the elevator stalled, and came to a stop between the 12th and 13th floors. The woman’s fears were justified, as the man suddenly grabbed at her purse. When the woman resisted, the man struck her, and she fell unconscious. When the woman recovered consciousness, she found herself in a hospital with a concussion and multiple contusions on her body. She later learned that the elevator had remained stalled for 50 minutes. Thereafter, it moved to the 20th floor, where she was found by her friend, who took her to the hospital. Two weeks later, the police arrested the man and found the woman’s purse in his possession. This was the first assault that had occurred in the apartment building. The elevator’s stalling was caused by a defective cable that a routine inspection would have discovered.
    If the woman brings suit against the owner of the apartment building for negligence, she will most likely
    (A) recover, because she was an invitee of a tenant in the building.
    (B) recover, because the owner would be vicariously liable for the company’s failure to inspect the elevator.
    (C) not recover, because the woman was negligent in failing to exit the elevator on the 12th floor when the other occupants exited the elevator.
    (D) not recover, because there had been no previous assaults in the apartment building.
A
  1. (D) In this question, your natural inclination is to find the owner liable for failing to have the elevator inspected. However, in any negligence action, it is necessary that the defendant’s negligent conduct be the legal cause of the plaintiff’s harm. Here, the woman was injured when she was assaulted by the man. She was not injured by the stalling of the elevator. Therefore, the man’s action would be viewed as a superseding cause (or unforeseeable intervening force). The attack did occur when the elevator was stalled, but the connection between the stalling and the attack was basically happenstance. In other words, the attack could just as easily have occurred even if the elevator had not stalled. The owner, therefore, should not be liable on the basis of his failure to ensure that the elevator was properly inspected and maintained. The woman could try to assert a negligence claim based on a theory that the building had inadequate security, but such a claim would be successful only if the woman could prove that the owner was aware of similar crimes occurring in the building in the past. Otherwise, the foreseeability required for liability to be imposed on the owner would be absent. Choice (D) is, therefore, the best answer because it indicates that the owner will not be liable because there had been no previous assaults in the building. Choice (A) is incorrect because, regardless of the woman’s status in the building, the ownerwould not be liable to the woman because of the inadequate connection between the owner’s negligence (in not ensuring that the elevator was well inspected and maintained) and the woman’s injuries. Choice (B) is incorrect because vicarious liability does not exist here. The contract between the owner and the company does not create an employee-employer reLationship, which is necessary to find vicarious liability. Moreover, neither the owner nor the company would be liable for the woman’s injuries because of the inadequate connection between the negligence and the injuries. Choice (C) is incorrect because even if the woman’s conduct was viewed as negligent, this would only reduce her recovery through comparative negligence, not eliminate any potentiaL recovery.
138
Q
  1. A young lawyer worked late at her office one night. When she finally decided to go home, she headed to the elevator, got in, and pushed the button for the ground floor. The elevator went all the way to the ground floor, but then just stopped there. The doors did not open. The lawyer tried pushing each of the buttons in the elevator and tried to use the intercom in the elevator to call for help, but nothing on the elevator’s control panel seemed to be functioning. The lawyer was afraid that she would be stuck in the elevator all night. She began pounding on the doors and yelling. She started to feel panicked, hyperventilated, lost consciousness, and fell to the floor of the elevator. As she fell, she struck her head on the elevator wall, causing a large bump and bruise on her head. She was unconscious in the elevator for about 30 minutes before a security guard tried to use the elevator and realized it wasn’t working. The guard managed to force the doors open and found the lawyer. At that moment, the lawyer regained consciousness. An investigation revealed that the elevator stopped working because it had not been properly maintained. The owner of the building had canceled all service and maintenance work on the elevator in order to save money.
    If the lawyer sues the building owner for false imprisonment, she will most likely
    (A) prevail, because she was confined in the elevator for an unreasonably long period of time.
    (B) prevail, because she was injured during her confinement in the elevator.
    (C) not prevail, because the building owner did not intentionally cause the lawyer to be confined.
    (D) not prevail, because the lawyer was unconscious and unaware of what was happening for most of the confinement period.
A
  1. (C) Liability for false imprisonment can be imposed where the defendant did an act with the intent of causing the plaintiff to be confined within a limited area, with the result that the plaintiff is confined within a limited area for an appreciable amount of time and is either aware of or harmed by the confinement. As for any intentional tort, intent is a crucial requirement. The required intent can exist where the defendant had the purpose or desire of causing the required result or where the defendant acted with knowledge that the result was substantially certain to occur. Here, the owner of the building might be liable for negligence, but would not be liable for false imprisonment because he or she had no intent for the lawyer (or anyone else) to be confined. Choice (A) is incorrect because the confinement lasted long enough to qualify as false imprisonment, but the intent element is not fulfilled here. Choice (B) is incorrect because the lack of intent bars recovery for false imprisonment here, even though the plaintiff was injured. Choice (D) is incorrect because the lawyer was aware of her confinement for the first few minutes of the confinement, and also because she injured her head as a result of the confinement.
139
Q
  1. A 12-year-old boy decided to play a practical joke. Knowing that his parents were giving a dinner party that evening, the boy filled a bucket with ice-cold water and balanced it on the partially open door of the guest bedroom, thinking that his father would take the guests’ coats there to put them on the bed. The boy then went off to play at a friend’s house. The boy’s father decided to keep all the guests’ coats in the hall closet instead of putting them in the guest bedroom. Later that evening, one of the guests mistakenly wandered into the guest bedroom while searching for a bathroom. As the guest opened the door, the bucket fell and hit his head, causing a severe cut that required a dozen stitches.
    If the guest sues the boy for negligence, the most likely result is
    (A) the boy wins, because a child’s conduct cannot be negligent.
    (B) the boy wins, because the guest unreasonably assumed the risk of entering an unfamiliar room rather than asking for directions to the bathroom.
    (C) the boy loses, because this “practical joke” posed a risk of severe harm and, therefore, the boy had a duty to exercise as much care as a reasonable adult.
    (D) the boy loses, because he did not act with the amount of care that one would expect from a 12-year-old child.
A
  1. (D) Liability for negligence generally depends on whether the defendant acted with the care of an ordinary reasonable person under the circumstances. A child, however, is not expected to be able to act as carefully and reasonably as an adult. Instead, a child is expected to act with as much care as a reasonable child of the same age, intelligence, and experience. This rule enables children to “act their age” and not be held liable for conduct that is below the standard of care that would apply to adults. While that is the general rule for children, an exception applies where a child engages in an activity that is adult in nature, such as operating a car, airplane, or other motor vehicle. Most courts generally seem to agree that a very young child, such as one under the age of 4 or 5 years, is not capable of negligence. A minority of states use a more specific rule known as the “rule of sevens,” under which a child under the age of 7 years is treated as being incapable of negligence, and a child from the age of 7 to 14 years is presumptively incapable of negligence. Most states have rejected the rule of sevens and, instead, simply say that very young children are incapable of negligence, without being as specific as the rule of sevens would require. Here, the boy was well above the age of 4 or 5 years, at which age children become capable of negligence; he was not engaged in an adult activity like driving a car, so the general standard of care for children will apply. In other words, the issue will be whether the boy exercised as much care as a reasonable child of the same age, intelligence, and experience would use under the same circumstances. Choice (A) is incorrect because a 12-year-old child’s condu€t can be negligent. Choice (B) is incorrect because there is nothing unreasonable about trying to find a bathroom to use. The guest could not have expected and did not assume the risk of a bucket falling on his head. Moreover, even if the guest had acted unreasonably, that would merely reduce his damages to some extent under comparative fault principles, rather than barring his claim completely. Choice (C) is incorrect because even though the practical joke was a very dangerous thing to do, it was not an “adult” type of activity, like driving a car and, therefore, the adult standard of care would not apply.
140
Q
  1. A department store had suffered a succession of thefts of merchandise over a period of months. From reports by employees and customers, the department store’s manager concluded that the losses were due, wholly or in large part, to the depredations of a female shoplifter, aged 30 to 40 years, about 5 feet 5 inches in height, with red hair and green eyes, who usually wore a suit. This information was passed on to all of the employees of the department store.
    One day, a woman entered the store to purchase accessories. The woman was 5 feet 5 inches tall, 37 years old, red-haired and green-eyed, and dressed in a smartly tailored suit. She carried a large shopping bag. The manager noticed her as she picked up, examined, and put down a number of gloves and scarves. After a while, she looked about tentatively, and then started to walk out, swinging her bag. The manager intercepted her and, standing in front of her, politely asked the woman if she would accompany her to the store manager’s office. When the woman asked for an explanation, the manager told her of the store’s recent experience and the suspicion that the woman might be concealing pilfered merchandise in her bag. Flushing angrily, the woman replied, “Very well,” and followed her to the office.
    Once there, the manager began to question the woman quite intensively. After the questioning, the manager then asked the woman’s permission to inspect the contents of her shopping bag. At first, the woman curtly refused and announced that she had had enough of this nonsense. When she rose to go, the manager told her, “Listen, unless you let me look inside that bag, I’m going to call the police.” The woman replied, “Very well,” and handed her the bag. The manager inspected the contents of the bag thoroughly but did not find any pilfered merchandise. She then gave the woman permission to leave. The total length of the woman’s detention was 30 minutes.
    If the woman asserts a claim for false imprisonment against the department store, she will most likely
    (A) prevail, because she was detained for an unreasonably long period of time.
    (B) prevail, because the manager did not have reasonable grounds to suspect that the woman had committed a theft.
    (C) not prevail, because under the circumstances, the manager had reasonable grounds to suspect that the woman had committed a theft.
    (D) not prevail, because the manager gave the woman permission to leave after conducting her investigation.
A
  1. (A) Traditionally, a merchant could stop a person suspected of shoplifting; but the merchant would be liable for false imprisonment if it turned out that the person was not actualLy guilty of shoplifting, even if the merchant acted very reasonably and in good faith. In other words, a merchant’s privilege to detain shoplifters was very limited and turned on whether the detained person actually had stolen something, not on whether there was a reasonable basis for the merchant’s suspicions. Most states have now modified that traditional rule, either by statutes or court decisions, so that the shopkeeper’s privilege is broader and allows the merchant to avoid liability if the merchant acts reasonably. The merchant’s actions must be reasonable both in the sense that there was a reasonable basis for thinking the person had stolen something and in the sense that the merchant handled the situation reasonably and did not detain the person longer than reasonably necessarily to realize that a mistake had been made and the person was innocent. A 30-minute detention may be an unreasonably long period of time to question a shopper and to discover that a mistake has been made and the shopper is not guilty of any crime. In fact, there are a number of decisions holding department stores liable for false imprisonment involving detentions of 30 minutes or less. Choice (B) is incorrect because the store’s employee had reasonable grounds for thinking the woman might be the shoplifter, so the store would not have been liable if it had detained the woman briefly to determine that she was not the thief. It was the length of the detention that was unreasonable here. Choice (C) is incorrect because although the store’s employee had reasonable grounds for detaining the woman, the length of the detention was unreasonable. Choice (D) is incorrect because the fact that the store employee eventually permitted the woman to leave does not prevent the store from being Liable for false imprisonment if the detention was unreasonable in length.
141
Q
  1. A vacationer, on a winter ski holiday, visited a ski lift in a private park. The ski lift company had installed and operated the lift pursuant to a concession agreement with the owner of the private park. Visitors gained entry to the park on payment of a $5 fee, which entitled them to go ice skating, tobogganing, or sledding. A ski lift ticket cost an additional $7 per day. At the top of the ski lift, there was a platform for embarking and disembarking passengers. The ski lift company paid the owner a stipulated rental plus 15 percent of the net proceeds from the lift.
    Two employees of the company operated the lift, one from a station at the bottom and the other from a station at the top of the hill. When the vacationer boarded the ski lift, it was late afternoon and most of the skiers had left. He was the sole passenger on the lift. Meanwhile, the employee at the top had left his post to go to the bathroom, asking his friend to keep watch on the lift and to stop it to allow any passengers to disembark. The friend consented, and the employee showed him how to use the control switch.
    When the vacationer approached the top, the employee was still away. Instead of stopping the lift to permit the vacationer to get off, the friend allowed the lift to keep moving. The vacationer was carried past the platform, and he was swung violently from side to side as the ski lift started downward. When the employee returned and sized up the situation, he threw the switch, stopping the lift. The vacationer, severely bruised and badly frightened, jumped off the ski lift and completed his descent by foot.
    In a personal injury action by the vacationer against the owner of the private park, the vacationer will rely on the concept of
    respondeat superior.
    vicarious liability. joint venture. imputed negligence.
    (A)
    (B)
    (C)
    (D)
A
  1. (C) In the vacationer’s personal injury action against the owner of the park, he will rely principally upon the doctrine of joint venture. The four basic elements of a joint enterprise are as follows: (1) an agreement, express or implied, among the members of the group; (2) a common purpose to be carried out by the group; (3) a community of pecuniary interest in that purpose, amongthe members; and (4) an equal right to a voice in the direction of the enterprise, which gives an equal right of control. The law then treats each member of the joint enterprise as the agent or servant of the others, and so the act of any one within the scope of the enterprise can be charged vicariousLy to the rest. Choices (A) and (B) are incorrect because this question involves a joint venture, rather than the type of master-servant or employer- employee relationship that creates vicarious liability through respondeat superior. Choice (D) is incorrect because it is necessary to show liability based upon a joint venture before inuting negligence to the defendant.
142
Q
  1. A man worked as a clerk in a gourmet coffee shop. His friend often stopped by the shop to chat with the man while the man was working. One afternoon, while the friend was visiting the shop, the man needed to use the bathroom. There were no customers in the shop at that moment, so the man asked the friend to watch the store for him while he went in the back to use the bathroom. The friend agreed to do so.
    While the man was in the bathroom, a customer came into the store. She wanted to purchase some freshly ground coffee. The shop had a grinding machine that customers could use. The customer selected a bag of coffee beans and then took it to the grinding machine. She was unsure how to start the machine, so she asked the friend for help. The friend tried to show the customer how to use the machine, but he was not familiar with how it worked, and he gave the customer erroneous instructions. The customer tried to follow the friend’s instructions, but this led to her getting her finger pinched and cut by the machine.
    If the customer asserts a claim against the coffee shop company for her finger injury, the customer will most likely
    (A) prevail, because the company would be vicariously liable for the man’s unauthorized actions.
    (B) prevail, because the friend’s negligence would be imputed to the company.
    (C) not prevail, because the man acted outside the scope of employment by entrusting the supervision of the shop to his friend.
    (D) not prevail, because the company is not liable for the friend’s negligence.
A
  1. (A) Under the doctrine of respondeat superior, an employer is liable for the torts of employees committed within the scope of their employment. Difficult questions often arise about whether an employee’s conduct was a total departure from the empLoyer’s business or just a roundabout way of doing it. When an employee acts outside the scope of employment, it also can be difficult to determine when the departure from the employer’s work is over and the employee has returned to being within the scope of employment. In general, as long as the event occurs during work hours and there is an intent on the part of the employee to serve the employer’s purpose, the employer will be liable. Here, the man did not have authorization from his employer to entrust the supervision of the shop to his friend while he was in the bathroom. However, this did occur during the man’s work hours, and it was done with the purpose of serving the employer’s purposes of operating the shop. The company, therefore, should be vicariously liable for the man’s actions here, even though they were unauthorized. Choice (B) is incorrect because the company is vicariously liable for the man’s actions, but not for the friend’s actions. Choice (C) is incorrect because entrusting the shop to the friend was done within the scope of the man’s employment, even if it was not specificaLly authorized. Choice (D) is incorrect because the company can be held vicariously liable for the man’s negligence in entrusting the shop to a friend who did not know how to work the grinding machine.
143
Q
  1. A ski resort had a lift that carried customers from the bottom to the top of the ski slope. One afternoon, the lift suddenly broke down because of a power failure in the area. A customer was suspended in his lift chair, one-third of the way up the hill and 50 feet above the ground. The customer remained on the lift for five hours until power was restored. He was then returned uninjured to the bottom of the hill.
    In a suit against the ski resort, the customer is likely to have action for
    (A) false imprisonment.
    (B) negligence.
    (C) assault.
    (D) no cause of action.
A
  1. (D) The customer would not have any cause of action against the ski resort. The intentional torts of false imprisonment and assault require proof that the defendant acted intentionaLly, while negligence requires proof that the defendant failed to exercise reasonable care. Here, there is no indication that the ski resort did anything with the intention of trapping the customer on the ski lift, and no indication that the ski lift failed to act with reasonable care. Choices (A), (B), and (C) are incorrect because the customer would not have any cause of action against the ski resort because there was no intentional tort or negLigence on the part of the ski resort or its employees.
144
Q
  1. A neighborhood homeowners’ association hired a security company to provide an armed guard to patrol the neighborhood. One evening, the guard saw a young man stealing a rake that a homeowner in the neighborhood had left outside in his yard. The guard ran toward the young man. Seeing the guard, the young man dropped the rake and began to flee. Believing that the young man was about to escape, the guard pulled out his gun and fired a shot at the young man. The bullet struck the young man in the arm.
    If the young man asserts a claim against the security company for damages for his injuries, the young man will
    (A) prevail, because the guard used unreasonable force to protect the homeowner’s property.
    (B) prevail, because the guard did not give a warning that he had a gun and was about to shoot.
    (C) not prevail, because the young man was trespassing on the homeowner’s property.
    (D) not prevail, because the young man was engaged in theft when he was shot.
A
  1. (A) The Law has always placed a higher value upon human safety than upon mere rights in property, so the majority rule is that there is no privilege to use any force calculated to cause death or serious bodily injury merely to repel a threat to land or chattels. A privilege to use such force will exist only if there is a sufficient threat to a person’s safety as well as to property interests. The plaintiff here, therefore, will prevail because the guard used unreasonable force. Choice (B) is incorrect because even if the guard had given a warning that he had a gun and was about to shoot and the young man kept running, the guard would not have been entitled to shoot the young man. Choices (C) and (D) are incorrect because the law has always placed a higher value upon human safety than upon mere rights in property. Therefore, the fact that the young man was guilty of trespass and theft would not justify the guard’s use of force likely to cause death or serious bodily injury.
145
Q
  1. A home security protection company provided protection services to property owners in the immediate area. The owner of a summer cottage in the area hired the company to provide 24- hour protection during the winter months when his home was unoccupied. According to the security arrangement, the company’s uniformed guards would periodically patrol the property and, if necessary, provide an “armed response” to any unauthorized individuals who were found trespassing on the property.
    The company provided security protection to the owner’s property for two years. The owner then notified the company that he was planning to sell the cottage. As a result, he requested that the company discontinue its home-protection service, effective immediately. Two weeks later, a burglar broke into the cottage and was ransacking it. As the burglar was exiting from the owner’s house carrying a portable television set, he was seen by a security guard working for the company. The security guard, who was driving past the home on security patrol, had not been informed that the owner had discontinued protection services. The security guard suddenly stopped his patrol vehicle and ran toward the burglar shouting, “Stop! Don’t move or I’ll shoot!” Startled, the burglar dropped the television set and began to flee. Believing that the suspected burglar was about to escape, the security guard pulled out his service revolver and fired a bullet at the burglar. The shot struck the burglar in the leg, seriously wounding him.
    If the burglar asserts a claim against the owner for negligently failing to supervise those providing security services for the owner’s property, the burglar will probably
    (A) prevail, because the burglar was unarmed when he was shot.
    (B) prevail, because the owner knew or had reason to know that the company provided an “armed response” to suspected trespassers.
    (C) not prevail, because the owner was not present when the shooting occurred.
    (D) not prevail, because the owner had discontinued protection services from the company when the shooting occurred.
A
  1. (D) A defendant has no duty of care as to a trespasser whose presence is unknown to him. Further, the owner was under no duty to warn the burglar, a trespasser, of the company’s use of deadLy force because the owner had duly ordered the company to discontinue its protection service. Consequently, the owner will not bear responsibility for the actions of the protection service. Choice (A) is incorrect because the fact that the burglar was unarmed does not make the owner liabLe for something that the company’s security guard did after the owner terminated the company’s services. Choice (B) is incorrect because the owner will not be liable regardless of whether he knew or should have known that the company provided an “armed response.” Choice (C) is incorrect because the owner cannot avoid liability on the grounds that he was not present. He will prevail and not be liable because he had already canceled the company’s services, not because he was not present at the scene of the shooting.
146
Q
  1. At 10:00p.m. onNovember 14, a driver was operating his automobile along Main Street. As the driver was approaching the intersection of Main Street and First Avenue, a motorist, who was driving straight through a red light, suddenly appeared before him. Trying to avoid the motorist, the driver veered his car onto the sidewalk. The car landed in a deep hole in the sidewalk. This hole had been dug by a construction company, which had been repairing a water main break earlier in the day. The construction company had been hired by the local municipal water department. Although the’ construction company had erected a warning sign advising pedestrians about the hole, there was no fence or barrier surrounding it.
    When the driver’s car fell into the hole, it ruptured the water main, engulfing the car with water. Within a short time, the driver, unable to escape, drowned in his car, which rapidly filled with water.
    In a wrongful death action by the driver’s estate against the municipal water department, the estate will most probably
    (A) prevail, because sovereign immunity would not attach to non-delegable duties, which are proprietary in nature.
    (B) prevail, because the city government would be strictly liable for failing to ensure the water main repair work was done properly.
    (C) not prevail, because the municipal water department would not be liable for the negligence of its independent contractor.
    (D) not prevail, because sovereign immunity attaches to functions that are governmental in nature.
A
  1. (A) Sovereign immunity does not attach to non-delegable duties that are “proprietary” in nature. When the city performs a service that might as well be provided by a private corporation, and particularly when it collects revenue from it, the function is considered a “proprietary” one, as to which there may be liability for the torts of municipal agents within the scope of their employment. On the other hand, immunity from tort liability does attach to functions performed in a “governmental,”“political,” or “public” capacity. The following are examples of proprietary functions: where a city supplies water, gas, electricity, or where it operates a ferry, wharves, docks, an airport, or public market. Here, the municipal water department should not be protected from liability by sovereign immunity because the water department’s agent was negligent with respect to services of a proprietary nature. Choice (B) is incorrect because strict liability would not apply. Choice (C) is incorrect because the municipal water department could be held liable for negligence with respect to a non-delegable duty. Choice (D) is incorrect because sovereign immunity would not apply to this work relating to a proprietary function performed by the city.
147
Q
  1. A carpenter was driving home one night after a long day of work. He was driving his car north toward an intersection. There were stoplights at the intersection. The light for the carpenter was green, so the carpenter proceeded to drive through the intersection. A bus entered the intersection from the west and slammed into the carpenter’s car. The carpenter was seriously injured in the crash and missed several weeks of work while recovering from the injuries.
    Which of the following facts or inferences, if true, would be most helpful in an action by the carpenter against the driver of the bus?
    (A) The bus driver had received three speeding tickets in the past.
    (B) The carpenter’s car was in good condition, and it had passed a safety inspection just a week before the accident.
    (C) The traffic signal at the intersection had been inspected the day before the accident and found to be functioning properly.
    (D) The bus driver was operating the bus without a driver’s license in violation of the State Motor Vehicle Code.
A
  1. (C) In a negligence case, the plaintiff generally has the burden of proving that the defendant was negligent and that this negligence was a cause of the accident. Proof that the traffic signal was functioning properly would be very helpful proof for the carpenter’s case because it would strongly suggest that the bus driver ran through a red light. That would establish that the bus driver was negligent perse and that the bus driver’s bad driving caused the accident. Choice (A) is incorrect because the fact that the bus driver had some speeding tickets in the past would provide only minimal support for the contention that the bus driver was negligent on this occasion. Choice (B) is incorrect because the fact that the carpenter’s car was in good shape and had been recently inspected would shed little light on whether the bus driver was negligent and caused this accident. Choice (D) is incorrect because the fact that the bus driver did not have a driver’s license would do little to establish negligence or causation here. While driving without a license is a statutory violation, it probably would not constitute negligence perse because it is a technical or administrative requirement, rather than a “rule of the road” that directly relates to how safely or dangerously the vehicle was being operated.
148
Q
  1. A construction company was doing repairs and replacing portions of a sidewalk and railing next to a lake. The construction crew started tearing out the old sidewalk and railing, but stopped work when it started to get dark. The construction crew left without putting up a warning sign or barrier around the work area. A few hours later, a jogger came along the sidewalk. Not realizing the construction work was in progress there, the jogger stumbled and fell at the spot where the construction crew had torn up the sidewalk and railing. The jogger fell into the lake. As the jogger was attempting to stay afloat, he began screaming, “Help! Help! I can’t swim. I’m drowning.” His screams attracted the attention of a person who was passing on his bicycle. The cyclist immediately hurried to assist the jogger. As the cyclist was leaning over the edge of the lake, trying to help the jogger get out of the water, he lost his balance and fell into the lake. Both the jogger and cyclist suffered serious bodily injuries before they were pulled out of the water by police.
    In a negligence action by the cyclist to recover for his personal injuries, the construction company will most probably
    (A) be held liable, because the cyclist’s attempt to rescue the jogger was foreseeable.
    (B) be held liable, because the construction company would be strictly liable to anyone injured by the failure to put adequate warnings or barriers around the site of the sidewalk repairs.
    (C) not be held liable, because the cyclist assumed the risk by leaning over the edge of the lake.
    (D) not be held liable, because the construction company could not foresee that anyone would be hurt while trying to rescue someone from the lake.
A
  1. (A) The “rescue doctrine” recognizes that if a defendant’s negligence puts someone in danger, it is foreseeable that another person will come to the rescue of the person who is in danger. If the rescuer is injured, the rescuer can hold liable the defendant whose negligence created the need for the rescue attempt. As Judge Cardozo stated in Wagner v. International Railway Co., 133 N.E. 437 (N.Y. 1921), “Danger invites rescue.” Choice (B) is incorrect because strict liability would not apply, and the construction company, instead, would be liable for negligence. Choice (C) is incorrect because someone who attempts to rescue another person from danger will not be treated as having assumed the risk where the alternative is to do nothing and allow the threatened harm to occur. Choice (D) is incorrect because the rescue doctrine provides that the cyclist’s rescue attempt will be treated as foreseeable.
149
Q
  1. An avid baseball fan wanted to purchase tickets for an upcoming baseball game being held at the local stadium. He contacted the stadium’s ticket sales office, which advised him that all tickets were to be placed on sale at the stadium at 9:00 a.m. the following day. The fan, who was employed as a bricklayer, realized that he could not leave work to purchase the tickets. Consequently, the fan telephoned his next-door neighbor and asked him if he would be interested in going to the stadium to purchase the baseball tickets. The neighbor told the fan that he would be happy to purchase the tickets for him and that he (the neighbor) also wanted to buy tickets for himself. However, the neighbor said that his car had broken down and that he did not have available transportation to get to the stadium. The fan suggested that he could lend his car to the neighbor. The neighbor agreed, and the following morning the two men met in front of the fan’s home.
    The fan gave the neighbor the keys to his car and also money with which to buy his baseball tickets. Thereafter, the neighbor drove the fan’s car to the stadium where he purchased the tickets for the fan and (using his own money) also bought a set of tickets for himself. The fan had instructed the neighbor to return the car to the fan’s home after his trip to the stadium. After the neighbor left the stadium, however, he decided to visit a friend in a town located 120 miles from his home. As the neighbor was driving toward the town, an automobile crashed into him, causing significant damage to the fan’s car. The cost of repairing the fan’s car was determined to be more than the retail value of the auto.
    In an action by the fan against the neighbor to recover damages resulting from the accident, the fan will most likely
    (A) recover nothing, because the neighbor was on a joint venture when the accident occurred.
    (B) recover the cost of repairing the car.
    (C) recover the retail value of the car.
    (D) recover, because the neighbor was negligently operating the auto at the time of the accident.
A
  1. (C) In general, any major and serious departure will be held to be a conversion, while minor ones that do no harm will not. Choice (C) is, therefore, correct because the neighbor committed a conversion by his unauthorized use of the motorvehicle (i.e., driving an additional 120 miles to visit a friend). As a result, the neighbor will be liable for the full value of the auto because the accident occurred during the course of this deviation. Choice (A) is incorrect because a joint venture involves parties working together to complete an undertaking in which they have a community of interest and in which they.will be operating with equal control over the instrumentalities being used. Here, there was no equal control of the car. Moreover, there was a major departure from the parties’ original community of interest when the neighbor drove the car in an unauthorized manner. Choice (B) is incorrect because the proper remedy for conversion is damages equal to the full value of the chattel, not the amount representing costs of repair. Choice (D) is incorrect because the plaintiff can recover for conversion here, without the need to prove negligence.
150
Q
  1. The local high school football team won the state championship. After the game the high school held a victory rally at the high school gym. As the team was being honored, the star quarterback of the football team approached one of the cheerleaders and asked her out for a date. The cheerleader, who had a crush on the quarterback, said she’d love to go out with him. The quarterback told her, “Great, I’ll pick you up at eight.”
    Later that evening, the quarterback was getting ready to drive to the cheerleader’s home when his car wouldn’t start. The quarterback then called his next-door neighbor and asked him if he could borrow his car. The neighbor agreed, but told him, “Okay, but make sure you return it by 2:00 a.m.” The quarterback assented and then drove to the cheerleader’s house.
    After picking her up, they drove to an all-night coffee shop where they spent the night talking and catching up over coffee and waffles. Losing track of time, the quarterback and the cheerleader did not leave the coffee shop until 3:00 a.m. After returning home around 4:00 a.m., the quarterback decided that it was too late to return the car, so he parked it in his driveway. He intended to return the car to the neighbor in the morning.
    A short while later, a thief stole the neighbor’s car from outside the quarterback’s home. The police found the car three months later, undamaged. The neighbor, however, refused to accept the car and brought a claim against the quarterback for conversion.
    In his claim, the neighbor will
    (A) succeed, because the quarterback could have returned the car and failed to do so.
    (B) succeed, because the quarterback left the car in his driveway, and it was stolen.
    (C) not succeed, because the quarterback intended to return the identical property he borrowed in an undamaged condition.
    (D) not succeed, because the criminal act of the thief was unforeseeable.
A
  1. (B) Where a person is authorized to use a chattel, but uses it in a manner exceeding that authorization, the person can be held liable for conversion if the unauthorized use of the chattel amounts to a serious violation of another person’s rights. To constitute conversion, the unpermitted use must be serious enough to justify requiring the defendant to pay the full value of the chattel. In the present case, the quarterbackwill be liable for conversion because his failure to return the car on time resulted in its being stolen. Moreover, the fact that the neighborwas deprived of the use of the vehicle for three months was sufficiently serious to constitute a material violation of his ownership rights. Choice (A) is incorrect because the quarterback’s failure to return the car right away would not be a sufficiently serious violation of the neighbor’s rights to generate liability for conversion. Instead, it was the fact that the quarterback left the car in his driveway and it was stolen that amounted to a serious deprivation of the neighbor’s right of control. Choice (C) is incorrect because the quarterback’s intent is not the issue in a conversion action. He exceeded the consent that he received from the neighbor, which is the key fact here. Choice (D) is incorrect because conversion is not measured in terms of negligence concepts like foreseeability.
151
Q
  1. A pedestrian was walking in front of a hotel in the downtown area when a chair was thrown from an unidentified window. The chair struck the pedestrian on the head, knocking her unconscious. When the pedestrian recovered consciousness, she found herself in a nearby hospital with a doctor in attendance. An examination revealed that the pedestrian had suffered a concussion and severe head lacerations. A subsequent investigation revealed that the chair had, in fact, been thrown from a window at the hotel. None of the hotel’s employees or guests, however, admitted culpability for the incident.
    If the pedestrian asserts a claim against the hotel for negligence, will the doctrine of res ipsa loquitur enable her to recover?
    (A) Yes, because the chair was within the control of the hotel.
    (B) Yes, because a chair is not usually thrown from a window in the absence of someone’s negligence.
    (C) No, because the chair was not within the control of the hotel at the time the pedestrian was injured.
    (D) No, because the hotel is not vicariously liable for the tortious conduct of its employees.
A
  1. (C) The doctrine of res ipsa loquitur allows a plaintiff to recover when there is strong circumstantial evidence that the defendant was negligent, even though the plaintiff cannot identify exactly what the defendant did that was negligent. The doctrine originated in Byrne v. Boadle, 159 Eng. Rep. 299 (1863), a British case involving a person who was walking down the street when he was suddenly struck on the head by a barrel of flour. Figuring that the barrel must have come from the warehouse located next to the spot where he was hit by the barrel, the injured person sued the owner of the warehouse. Although the plaintiff could not show exactly what anyone did that was negligent, the court said that the situation spoke for itself, and that negligence could be inferred because it seemed that a barrel would not fall unless one of the defendant’s employees had been negligent in some way. A different situation, with a different result, arose in Larson v. St. Francis Hotel, 188 R2d 513 (Cal. Ct. App. 1948), where a plaintiff was struck by a chair thrown by an unidentified person from an unidentified window of the defendant’s hotel. The plaintiff was unable to recover because the building in question was a hotel. It was not clear that the chair was thrown by an employee of the hotel, rather than a guest. The chair, thus, was not under the exclusive control of the hotel and its employees, and there was no sound basis for inferring that the hotel or its employees must have been negligent. This question is like the Larson case, rather than the Byrne case, so res ipsa loquitur does not apply, and the plaintiff will lose. Choice (A) is incorrect because the chair may have been thrown by a hotel guest, so the chair was not under the hotel’s exclusive controL. Choice (B) is incorrect because even though it may be clear that someone was negligent, that negligent person could have been a hotel guest, rather than a hotel employee. Choice CD) is incorrect because a hotel can be vicariously liable for the tortious conduct of its employees, but it is not clear that it was one of the employees (rather than a hotel guest) who was negligent.
152
Q
  1. A statute in the state makes it a misdemeanor for any motor vehicle to travel to the left of the center line of any two-way highway, road, or street.
    Late for a business appointment, a businesswoman was driving north on a highway when she decided to pass the car in front of her. As she swung across the center line into the southbound lane, her vehicle collided with a fire engine. As a result of the accident, the fire engine was delayed in reaching an owner’s house, which was entirely destroyed by fire. The owner’s home was located approximately one mile from the accident scene.
    If the owner asserts a claim against the
    businesswoman, he will most likely
    (A) recover the fair market value of his house before the fire.
    (B) recover that part of his loss that would have been prevented if the businesswoman had not hit the fire truck.
    (C) recover nothing, because the businesswoman was not responsible for causing the fire.
    (D) recover nothing, because the traffic statute was not designed to protect against the type of harm that the owner suffered.
A
  1. (B) Once it is determined that the defendant’s conduct has been a cause of some damage suffered by the plaintiff, a further question may arise as to the portion of the total damage sustained that may properly be assigned to the defendant, as distinguished from other causes. Where a logical basis can be found for some rough practi. cal apportionment, which limits a defendant’s liability to that part of the harm that he has, in fact, caused, it may be expected that the division will be made. Upon this basis, the businesswoman will be liable for that part of the damage to the owner’s home caused by her negligence in colliding with the fire engine. As a result, choice (B) is the best answer. Choice (A) is incorrect because the owner should not be able to hold the businesswoman responsible for all the damage to his house because it was going to suffer some damage from the fire regardless of what the businesswoman did. Choice (C) is incorrect because the businesswoman can be held liable for the harm that she caused by hitting and delaying the fire truck, even though she did not start the fire at the owner’s house. Choice (D) is incorrect because even if the owner cannot prevail on a negligence perse theory, he can hold the businesswoman liable for ordinary negligence. Negligence.per se would not apply here because the fire damage to the owner’s house was not the type of harm the legislature meant to prevent when it prohibited people from crossing the center line of roadways. However, even when negligence perse is unavailable, a plaintiff can still simply bring an ordinary negligence claim. Here, the owner can argue that the businesswoman failed to exercise reasonable care in her driving and that fire damage to his house was a reasonably foreseeable result of the businesswoman’s unsafe driving and collision with a fire truckthat was hurrying to the scene of the fire at the owner’s house.
153
Q
  1. On June 1, a businessman opened a health and massage spa, which was located on First Street in the city. The spa also provided health club facilities, such as saunas, steam rooms, and whirlpool baths.
    A 75-year-old spinster resided across the street from the spa. The spinster opposed the opening of the spa because she believed that it was a “cover” for an illegal operation. During the day, the spinster sat in her rocking chair on her front porch and observed a constant stream of businessmen entering the spa.
    On the evening of June 29, the spinster, disguising her voice, called the spa and told the businessman, “You pimp, why don’t you take your dirty trade elsewhere?” Without paying any attention to the call, the businessman hung up.
    The spinster then began making repeated crank telephone calls to the businessman’s establishment. Every hour on the hour for the next three weeks, the spinster made her crank calls, harassing the businessman and his employees. As a result of the hourly phone calls, spa business was constantly disrupted, causing the businessman to suffer a decline in the volume of customers. After contacting the police, the businessman discovered that the spinster was the person making the harassing calls.
    If the businessman asserts a claim against the spinster, the theory on which he will most likely prevail is
    (A) public nuisance.
    (B) private nuisance.
    (C) intentional infliction of emotional distress.
    (D) negligence.
A
  1. (B) The essence of a private nuisance is a substantial interference with the use and enjoyment of land. A private nuisance may consist of an interference with the physical condition of the land itseLf, such as by vibration or blasting that damages a house, the destruction of crops, flooding, raising the water table, or the pollution of a stream. Also, it may consist of a disturbance of the comfort or convenience of the occupant of the land, such as by repeated telephone calls, by unpleasant odors, by smoke, dust, or gas, and by loud noises, excessive light, or high temperatures. Choice (A) is incorrect because a public nuisance involves substantial interference with rights of the entire community or general public. The defendant’s conduct here was a problem only for the businessman and his business, not the entire community. Choice (C) is incorrect because the conduct here was not sufficiently extreme and outrageous to generate a claim for intentional infliction of emotional distress. It was, however, sufficiently annoying to produce a claim for private nuisance. Choice (D) is incorrect because negligence is not the best theory for the businessman to rely upon when a disturbance amounting to a private nuisance is clearLy shown to exist. The spinster’s conduct was not careless; it was done deliberately and with the intent of harassing the businessman and his business, so private nuisance is a more fitting cause of action.
154
Q
  1. A man and a woman had a romantic relationship for one year, but the man ended it. The woman was unable to accept the breakup and continued trying to contact the man. The woman would call the man’s home and cellphone many times each day, and also tried to visit the man’s home and workplace frequently. The man tried to discourage the woman’s behavior, but the woman persisted.
    If the man asserts a claim against the woman based on invasion of privacy, the most likely outcome is that the man will
    (A) prevail, because the telephone calls intruded upon his seclusion and solitude.
    (B) prevail, but only if the man is able to prove malice on the defendant’s part.
    (C) not prevail, because the telephone calls did not cause the man to suffer any economic loss or hardship.
    (D) not prevail, unless the woman’s conduct was a crime in some respect.
A
  1. (A) Invasion of privacy is not a single tort but, rather, consists of the four distinct torts of (1) appropriation, for the defendant’s benefit or advantage, of the plaintiff’s name or likeness; (2) intrusion upon the plaintiff’s physical solitude or seclusion; (3) public disclosure of private facts about the plaintiff; or (4) placing the plaintiff in a false light in the public eye. Here, the man could succeed in an invasion of privacy action based on intrusion of his physical solitude and seclusion. The tort has been extended to include persistent and unwanted telephone calls. See Carey v. Statewide Finance Co., 223 A.2d 405 (Conn. Cir. Ct. 1966). Also, the tort has been applied to peering into windows of a home and eavesdropping upon private conversations by means of wiretapping and microphones. Choice (B) is incorrect because malice is not a requirement for an invasion of privacy claim. Choice (C) is incorrect because a plaintiff can recover damages for invasion of privacy even if the plaintiff suffered no economic harm. Choice (D) is incorrect because a defendant’s conduct can be tortious even if it is not a crime.
155
Q
  1. A resident lived in a house across the street from a small office building. One of the offices in that building was that of a criminal defense lawyer. The resident hated the idea of the lawyer’s office being close to her home. She felt that it meant dangerous criminals were coming and going in her neighborhood. The resident spent a lot of time looking out her window, trying to figure out which people going in and out of the office building were criminals, and imagining what sorts of crimes they had committed. The resident eventually decided that she needed to drive the lawyer out of the neighborhood. She made a telephone call to the lawyer and said, “You are scum, and so are all of your clients. I know that you’re conspiring to commit crimes with all those bad people you represent. The police should arrest you and put you in jail with your clients.” The lawyer was very upset by what he heard.
    If the lawyer asserts a claim for defamation against the resident based on the telephone call, he will most likely
    (A) succeed, because the resident’s remarks constituted slander per Se.
    (B) succeed, because the lawyer found the remarks to be upsetting.
    (C) not succeed, because the resident’s remarks were a matter of personal opinion rather than statements of fact.
    (D) not succeed, because the resident’s remarks were not published or communicated to anyone but the plaintiff.
A
  1. (D) For a libel or slander claim, it is essential that the defamatory statements were communicated to someone other than the person defamed. Where there is no communication to anyone but the plaintiff, a defamation claim simply cannot succeed, although there might be liability on some other theory, such as intentional infliction of emotional distress. Thus, the resident’s telephone call to the lawyer would not generate liability for defamation. Choices (A), (B), and (C) are incorrect because liability for defamation can arise only where the allegedly defamatory statement was communicated to someone other than the plaintiff.
156
Q
  1. A resident in an exclusive residential area is a marine biologist. To aid in his study of sharks, he had a large tank built in his backyard in which he placed a great white shark.
    Aside from a smell that emanated from the tank, some neighbors were concerned and afraid to let their children outside for fear that they would wander onto the resident’s property. In order to convince his neighbors that they had nothing to fear, the resident invited them over to view the shark tank. While a neighbor was standing near the tank, the shark splashed its tail, and the neighbor got very wet. The neighbor, who had a cold, developed bronchitis.
    If the neighbor sues the resident for damages incurred from being drenched by the shark and bases her suit on strict liability, she will most likely
    (A) recover, because the possessor of wild animals is strictly liable for all injuries caused to others by the animals.
    (B) recover, because the smell from the shark tank was a nuisance.
    (C) not recover, because she suffered injury only because she had a cold and, therefore, was unusually vulnerable to harm from being splashed with water.
    (D) not recover, because she did not suffer the type of harm normally inflicted by a shark.
A
  1. (D) A possessor of wild animals is strictly liable for injuries to others that result from the animals’ normally dangerous propensities. For example, if you keep a lion on your property, you will be liable if it attacks someone, even if you exercise as much care as possible in keeping and handling the lion. Likewise, if the shark had bitten someone, the person keeping the shark would have been strictly liable. However, sharks do not normally harm people by splashing them, so the resident would not be strictly liable forthe neighbor’s illness. Choice (A) is incorrect because although keepers of wild animals are generally strictly liable, that rule does not apply where the harm caused by the animal is not related to the animal’s dangerous propensities. Choice (B) is incorrect because although the smell may have been a nuisance, the question asks about the neighbor’s ability to recover for the bronchitis she suffered because of being splashed. Choice (C) is incorrect because the fact that the neighbor had a special vulnerability to getting sick, because of her cold, would not bar her recovery. Under the “eggshell skull” or “thin skull” rule, a defendant can be liable even if the plaintiff’s harm is unexpectedly severe.
157
Q
  1. An environmentalist was very interested in environmental issues, particularly protection of wetland areas. He decided to dig out the lawn in his back yard and turn the space into a swampy marsh. Eventually, his back yard was filled with tall grasses, reeds, and other marsh plants. A wide variety of frogs, turtles, snakes, birds, and other animals inhabited the yard. The ground was usually covered by several inches of standing water.
    The environmentalist’s neighbors were not pleased with the condition of the environmentalist’s yard. They complained that it produced foul odors, and they claimed that the standing water was a breeding ground for mosquitoes and other insects.
    Several months after the environmentalist converted his yard into a marsh, a real estate investor purchased the house closest to the environmentalist’s back yard swamp. The investor lived in a large city several hundred miles away, and he purchased the house next to the environmentalist’s for investment purposes. The investor rented the house to a family under a long-term lease. The tenant family complained frequently to the investor about being annoyed by the environmentalist’s yard.
    If the investor asserts a nuisance claim against the environmentalist, the environmentalist’s best defense would be
    (A) that he had sound environmental reasons for maintaining the swampy condition of his yard.
    (B) that turning his yard into a swampy marsh did not violate any zoning ordinance.
    (C) that the investor owns the property but has rented it out, so the investor does not have actual possession or the right to immediate possession of the land.
    (D) that when the investor purchased the house, he knew or should have known about the swampy condition of the environmentalist’s property.
A
  1. (C) To recover for the tort of nuisance, the conduct of the defendant must constitute an invasion of one’s possessory interest in land. The environmentalist’s strongest defense would be that the property was occupied by another family (under a long- term lease) and, therefore, the real estate investor who owned the property was not entitled to actual or immediate possession. Thus, there was not an invasion of the investor’s present possessory interest in the land. Choice (A) is incorrect because the environmentalist’s property could be a nuisance, even if his motives and environmental objectives were good. Choice (B) is incorrect because the environmentalist’s property could be a nuisance, even if it did not violate any zoning ordinances. Choice (D) is incorrect because the environmentalist’s property could be a nuisance, even though he turned it into a swamp before the investor purchased the neighboring property. The fact that the investor “came to the nuisance” (i.e., purchased neighboring property after the nuisance was already in existence) would not necessarily bar the investor’s claim and, instead, would be just a factor to be considered in a court’s weighing of the equities and its decision about what relief (if any) to grant.
158
Q
  1. The local jurisdiction has a modified comparative negligence statutein effect that provides:
    “Contributory negligence shall not bar recovery in any action by any person or his legal representative to recover damages for negligence resulting in death or injury to person or property, if such negligence was not as great as the negligence of the person against whom recovery is sought, but any damages allowed shall be diminished in proportion to the amount of negligence attributable to the person for whose injury, damage, or death recovery is made.”
    In addition, this jurisdiction follows joint and several liability for joint tortfeasors.
    A buyer recently purchased an undeveloped beachfront lot in the jurisdiction. He hired a building contractor to construct a new house on the property. Thereafter, the contractor employed a subcontractor to provide on-site maintenance. The contractor contracted with a cement company to supply concrete for the foundation of the structure.
    After excavation started, an employee of the cement company delivered a load of concrete to the construction site. To facilitate delivery, the contractor and the subcontractor had built an access ramp extending from the street to the lot. As the employee was driving over the ramp, it suddenly collapsed and caused the truck to overturn, seriously injuring the employee.
    A subsequent investigation determined that the contractor and the subcontractor negligently used substandard materials in constructing the ramp. In addition, the employee was partially at fault for driving while intoxicated. The employee brought suit against the contractor and the subcontractor to recover damages for his injuries. A special trial was held on the issue of negligence, and it was adjudged that each party was contributorily negligent as follows:

PARTY DEGREE OF
NEGLIGENCE
Plaintiff: (the EE) 40%
Defendant: (the contractor) 35%
Defendant: (the subcontractor) 25%

The employee, who suffered damages in the amount of $100,000, brings a personal injury action against the contractor individually. He will most likely
(A) recover nothing, because he was more at fault than the contractor.
(B) recover $100,000.
(C) recover $55,000.
(D) recover $35,000.

A
  1. (A) Under some modified comparative fault statutes, the plaintiff’s negligence is compared with that of each individual defendant. For example, if P is 40 percent at fault and is suing Dl who is 10 percent at fault and D2 who is 50 percent at fault, P would be able to recover from D2, but not from D1.On the other hand, the modifled comparative fault statutes in other states require the plaintiffs negligence to be compared to the sum of the fault shares of all the defendants. Using the same example, P could recover from either Dl or D2 because her percentage of fault, 40 percent, is less than the aggregate fault (i.e., 60 percent) of Dl and D2.The comparative fault statute in this question refers to the plaintiff’s fault being compared to that of “the person” against whom recovery is sought. It does not say that the plaintiff’s fault should be compared to the sum or aggregate of all the defendants’ shares. The plaintiff employee’s share of the fault was 40 percent, which is more than the defendant contractor’s 35 percent share. As a result, the plaintiff would be unable to recover. Choices (B), (C), and (D) are incorrect because the comparative fault statute here bars the plaintiff from recovering anything because the plaintiffs share of the fault is greaterthan the defendant’s share.
159
Q
  1. A traffic accident occurred at a road intersection. A motorcycle, a car, and a truck were involved. The motorcyclist was injured and brought suit against the driver of the car and the driver of the truck. The jury returned a verdict finding that the motorcyclist’s injuries were caused by negligence on the part of all three of the parties. The jury assigned 55 percent of the fault for the motorcyclist’s injuries to the motorcyclist, 25 percent to the driver of the car, and 20 percent to the driver of the truck. The jury found that the amount of the motorcyclist’s injuries was
    $100,000.
    The motorcyclist enforces the judgment against the driver of the car and collects $45,000 from the driver of the car. If the driver of the car then brings an action against the driver of the truck for contribution, the driver of the car should
    (A) recover nothing, because he was more at fault than the driver of the truck.
    (B) recover $27,000.
    (C) recover $20,000.
    (D) recover $15,000.
A
  1. (C) On the Multistate Bar Exam, you must assume that pure comparative fault and joint and several liability apply unless the question states otherwise. Under pure comparative fault, the plaintiff can recover damages representing the shares of fault assigned to the defendants, even if the plaintiffs fault exceeded the fault of the defendants. Under joint and several liability, each defendant can be held liable for the full amount of damages owed to the plaintiff, and then the defendant who pays the judgment can use contribution claims to obtain reimbursement from the other defendants for their shares of the judgment. Here, under comparative fault, the plaintiff is able to recover 45 percent of the damages, or $45,000. If the driver of the car is forced to pay the $45,000 under joint and several liability, the driver of the car then can seek reimbursement from the other defendant for its share of the judgment. Choice (A) is incorrect because a contribution claim would be allowed to distribute the liability among the defendants according to their shares of the fault, even if the defendant seeking contribution had the higher share of the fault. Choices (B) and (D) are incorrect because the amount that could be recovered through a contribution claim would be $20,000, which is the truck driver’s share of the judgment.
160
Q
  1. A company owns and operates a chemical plant that manufactures paraquat, an herbicide used primarily as a weed killer. The plant is located in a rural area in the southwestern part of the state. In its production of paraquat, the company emits putrid smelling fumes throughout the surrounding countryside. Although the fumes are harmless to health, they are rank and highly offensive. Despite extensive research by the company to alleviate the problem, there is no known way to manufacture paraquat without discharging malodorous fumes into the air. An owner of a large dairy farm that is adjacent to the plant finds the foul-smelling herbicide fumes to be extremely unpleasant and totally objectionable.
    The owner brings an action based on private nuisance against the company and seeks to enjoin the manufacture of paraquat at the chemical plant. Which of the following facts, if proven, will be most helpful to the company’s defense?
    (A) Federal, state, and local agencies approved the design of the plant and equipment used to produce paraquat.
    (B) The company has a contract with the federal government whereby it supplies 80 percent of its paraquat production to the Drug Enforcement Agency, which uses the herbicide as a marijuana eradicator.
    (C) The company commenced the manufacture of paraquat at the plant before the owner acquired the land and built his dairy farm.
    (D) Paraquat is the only herbicide that can safely and effectively kill marijuana, which, if not controlled, poses a serious drug-enforcement problem.
A
  1. (D) A private nuisance is defined as a substantial and unreasonable interference with one’s use and enjoyment of land. When the plaintiff wishes to have an activity enjoined on the theory of a nuisance, it is necessary to show that the defendant’s conduct in carrying on the activity at the place and at the time the injunction is sought is unreasonable. Such conduct is unreasonable only if the gravity of the harm caused outweighs the utility of the conduct. This would be so if a reasonable person would conclude that there was a feasible way, economically and scientifically, to avoid a substantial amount of the harm without material impairment of the benefits. In this question, the company’s best defense in a private nuisance action would be choice CD). If paraquat was the only herbicide that could safely and effectively kill marijuana, which, if not controlled, would pose a serious drug enforcement problem, then there would be no feasible scientific means to avoid the harm without impairing the benefits. In other words, the utility of the conduct would outweigh the harm caused. Choices (A) and (B) are incorrect because neither government approvals nor existence of a government contract address the judicial standard of balancing the utility versus the harm. Choice (C) is incorrect because “coming to the nuisance” is only one factor the courts employ in “balancing the equities.”
161
Q
  1. A person owned property next to a highway. After raking leaves on his property into a large pile, the landowner loaded the leaves into several large metal barrels so that he could burn the leaves. Before starting the fire, the landowner telephoned the local weather bureau to determine which direction the wind would be blowing. Because a highway was located on the southern edge of his property, he was aware that it would be unsafe to burn the leaves if the wind was blowing in that direction. The weather bureau, however, indicated that the wind would be gusting in a northerly direction. Shortly after the landowner set the leaves on fire, the wind current unexpectedly shifted and started gusting in a southerly direction. As a consequence, the smoke and ashes blew over the highway, resulting in poor visibility. Moments later, a motorist was driving his automobile on the highway in a westerly direction. The posted speed limit was 45 m.p.h., although the driver was traveling about 55 m.p.h. The driver could not see the highway clearly and crashed his vehicle into a median strip.
    If the driver asserts a claim against the landowner, the most likely result is
    (A) the landowner will prevail, because the driver was driving in excess of the speed limit.
    (B) the landowner will prevail, if his decision to burn the leaves was reasonable under the circumstances.
    (C) the driver will prevail, if the smoke from the burning leaves prevented him from clearly seeing the roadway.
    (D) the driver will prevail, because the landowner will be strictly liable for causing the accident.
A
  1. (B) Liability for negligence can be imposed if the defendant failed to exercise the care of a reasonable person under the circumstances. One of the most common defenses to a negligence claim is that the plaintiffs own negligence was also a cause of the plaintiffs injuries. Under the traditional rule of “contributory negligence,” a plaintiffs claim would be barred completely if the plaintiffs negligence was a cause of the plaintiff’s injuries. However, in most states, contributory negligence rules have been replaced with comparative fault rules under which the plaintiff’s negligence merely reduces the plaintiffs recovery by some percentage set by the jury, rather than barring the plaintiff’s claim completely. On the Multistate Bar Exam, you must assume that comparative fault applies unless the question states otherwise. Here, the plaintiff driver will prevail if Kis conduct was not negligent. Choice (A) is incorrect because even if the driver was negligent by driving in excess of the speed limit, that would merely result in reduction of his recovery to some extent under comparative fault, rather than barring the claim completely. Choice (C) is incorrect because even if the smoke from the burning leaves obscured the driver’s vision and caused the accident, that would not make the landowner liable unless the landowner was negligent. Choice (D) is incorrect because there is no reason for strict liability to be imposed on the landowner. Instead, the landownerwould be liable only if negligent.
162
Q
  1. A farmer raked up some leaves on his property, put them into a metal barrel, and set the leaves on fire. The farmer then went off to his barn to do some other work. A few minutes later, a wind gust blew some burning leaves onto a neighbor’s property, causing a small fire amid some brush.
    If the neighbor asserts a claim against the farmer, the neighbor will most likely
    (A) recover, because the farmer is strictly liable for the spread of the fire.
    (B) recover, because the farmer was negligent in leaving the fire unattended.
    (C) recover, because the farmer created a public nuisance in failing to contro’ the fire.
    (D) not recover, because the farmer is not liable for an unforeseeable act of God.
A
  1. (B) At early common law, something approaching strict liability for fire was imposed upon landholders. Under the modern view, however, in the absence of Legislation, there is no liability for the escape of fire where the defendant was not negligent. There may, as in the present case, be liability for negligence in failing to control the fire or to take adequate precautions with respect to the fire. Choice (A) is incorrect because most courts today do not impose strict liability for handling fire. Choice (C) is incorrect because a public nuisance exists only where there is a significant and unreasonable interference with a right of the entire community or general public. This fire merely caused harm to one person, so it is not a public nuisance. Choice (D) is incorrect because a wind gust is not unforeseeable, and a person can be Liable for negligence that causes harm through operation of a natural force, such as wind.
163
Q
  1. An auto mechanic was repairing a car in his auto. shop when a spark ignited gasoline and oil residue on the floor. The mechanic managed to douse the flames, but not before the fire created giant billows of smoke that floated over onto a neighbor’s property and caused the neighbor’s home to become discolored.
If the neighbor asserts a claim against the mechanic, the neighbor would most likely be able to recover for
(A) strict liability.
(B) negligence.
(C) nuisance.
(D) trespass.
A
  1. (B) Liability for negligence can be imposed if the defendant causes harm by failing to exercise the care of a reasonable person under the circumstances. Here, the mechanic could be Liable for negligence if he failed to exercise reasonable care. Choice (A) is incorrect because there is no basis for strict liability to be imposed on the mechanic. Although handling flammable substances like gasoline may be an ultrahazardous activity in many circumstances, operating an auto repair shop is not such a dangerous activity that it would be treated as uLtrahazardous and subject to strict liability. Choice (C) is incorrect because a nuisance requires a disturbance of a continuing or chronic nature. For exampLe, if a factory emits smoke or dust on a daily basis, then an action for nuisance may arise. The smoke here, however, was a one-time occurrence and, therefore, not a nuisance. Choice (D) is incorrect because trespass requires that the defendant intentionally caused someone or something to enterthe plaintiff’s land. Here, the mechanic did not intentionally create the smoke. In addition, smoke is treated as an intangible item (Like noises, odors, lights, etc.) that is incapable of constituting a trespass to another person’s land.
164
Q
  1. A woman was employed as a legal secretary for a local attorney. After the attorney terminated the woman’s employment, she sent a job application and résumé to another attorney. In the résumé she sent to the second attorney, the woman listed her former employment with the first attorney. After receiving the woman’s résumé, the second attorney telephoned the first attorney for his opinion of her qualifications. The first attorney replied that he dismissed the woman “because she was unprofessional and incompetent.” The first attorney’s assessment was based on one malpractice incident for which he blamed the woman, but which, in fact, was attributable to another secretary in his law firm. Although the first attorney reasonably believed that his low rating of the woman was a fair reflection of her performance, he wrongfully held her responsible for the malpractice incident. Based on the first attorney’s poor recommendation, the second attorney did not hire the woman.
    In a defamation action by the woman against the first attorney, the plaintiff will most likely
    (A) prevail, because the first attorney’s statement reflected adversely on the woman’s professional competence.
    (B) prevail, because the first attorney was mistaken in the facts upon which he based his opinion of the woman’s performance.
    (C) not prevail, because the first attorney had reasonable grounds for his belief that the woman was incompetent.
    (D) not prevail, because the woman listed her former employment with the first attorney in her résumé to the second attorney.
A
  1. (C) When an employer asks questions and checks references of a job applicant, those who are asked about the applicant have a qualified privilege to answer and give their opinion about the applicant. The statements made about the applicant will be priviLeged, even if they are false, if they were made in good faith. This qualified privilege serves the purpose of enabling people to speak freely and share information with the employer, who is trying to decide whether to hire the applicant. Choice (C) is, therefore, correct because the statements will be privileged, since the facts state that the first attorney “reasonably believed that his low rating of the woman was a fair reflection of her performance.” Choices (A) and (B) are incorrect because they ignore the qualified privilege that will bar the plaintiff’s claim here. Choice (D) is incorrect because the fact that the plaintiff listed the defendant as a former employer on her résumé will not bar the plaintiff’s claim. It is the fact that the defendant had reasonable grounds for his adverse opinion and statements about the plaintiff that will bar liability from being imposed.
165
Q
  1. A husband was about to leave his home for work one morning when his wife ran in from the kitchen explaining that their 4-year-old son had just swallowed some medicine that had been prescribed for use as a skin lotion. Dashing to the car, the husband then drove his wife and ailing son to the nearest hospital. On the way to the hospital, the son had a seizure and stopped breathing. Seconds later, the car went out of control despite the husband’s reasonable efforts, swinging across the center line into oncoming traffic, where it collided wit.I1 a car driven by a driver who was driving in the opposite direction.
    In this jurisdiction, a statute makes it a misdemeanor for any motor vehicle to travel to the left of the center line of any two-way highway, road, or street.
    If the driver asserts a claim against the husband, the most likely result is that the plaintiff will
    (A) prevail, because the husband is strictly liable for violating the statute.
    (B) prevail, because the statute was designed to protect motorists such as the driver.
    (C) not prevail, because the driver had the last clear chance to avoid the accident.
    (D) not prevail, because the husband was acting reasonably in an emergency.
A
  1. (D) Under the prevailing view, one who is confronted with an emergency is not to be held to the standard of conduct normally applied to one who is not in such a situation. An emergency has been defined as a sudden or unexpected event or combination of circumstances that calls for immediate action. Under such conditions, the actor cannot reasonably be held to the same conduct as one who has had full opportunity to reflect, even though it later appears that he made the wrong decision. This same concept applies in negligence perse situations, because a person who has a valid excuse for violating a statute will not be found negligent per Se. Valid excuses would include being unable to comply with a statute because of a sudden emergency, like the child’s seizure. Choice (D) is, therefore, correct because the plaintiff would not be able to recover because the defendant was acting reasonably in an emergency. Choice (A) is incorrect because strict liability would not apply to violation of a traffic law in an emergency. Choice (B) is incorrect because negligence perse would not apply here, even if the statute was designed to protect motorists, like the driver, if the husband has a valid excuse for violating the statute. Choice (C) is incorrect because the “last clear chance” doctrine is obsolete. It was an exception to the doctrine of contributory negLigence, allowing a plaintiff to recover, despite being negligent, if the defendant had the last clear chance to avoid the accident. In jurisdictions that have replaced contributory negligence with comparative fault, the doctrine of last clear chance no longer exists. On the Multistate, you must assume pure comparative fault applies unless the question states otherwise. Moreover, even in a cQntributory negligence jurisdiction, the doctrine of last clear chance would be relevant only where it is the defendant that has the last clear chance to avoid the accident. Here, choice (C) talks about the plaintiff having the last clear chance, so the doctrine would not apply.
166
Q
  1. A car owner noticed a peculiar “shimmy” in the steering wheel of his automobile, which appeared to him to have been getting worse in the course of the preceding week. A few days after discovering the shimmy in the steering wheel, the car owner lent his automobile to his next-door neighbor. When the neighbor picked up the car, the car owner forgot to tell him about the shimmy. The neighbor was driving the car at a reasonable rate of speed within the posted speed limit when the car began to swerve across the road. The neighbor turned the steering wheel in an attempt to stay on the road. The steering failed, however, and the car veered off the road and onto the sidewalk. The car struck a pedestrian who was injured in the collision.
    If the pedestrian initiates suit against the owner of the car, the pedestrian will most likely
    (A) prevail, because the car owner knew the steering was faulty and failed to tell his neighbor who borrowed the car.
    (B) prevail, because the car owner is strictly liable under the circumstances.
    (C) not prevail, because the faulty steering was the cause-in-fact of the pedestrian’s harm.
    (D) not prevail, because the car owner was a gratuitous lender.
A
  1. (A) In the case of gratuitous bailments, the majority of courts have treated the guest as a licensee on personal property in essentially the same position as one entering by permission upon the land of another. Application of that analogy has led to decisions in several states holding that while the driver or owner is under a duty to exercise reasonable care for the protection of the guest in his active operation of the car and is required to disclose to him any defects in the vehic’e of which he has knowledge, he is not required to inspect the automobile to make sure it is safe. Choice (A), therefore, is correct because the car owner will be liable because he knew the steering was faulty and did not tell the neighbor who borrowed the car. Choice (B) is incorrect because the duty upon the car owner to his guest is that of reasonable care, not strict liability. Strict liability for the steering defect would apply only to the manufacturer and other sellers of the car. Choice (C) is incorrect because the pedestrian plaintiff will be able to hold the car owner liable for failing to warn about the known flaw in the car’s steering. Choice (0) is incorrect because a gratuitous lender is required to disclose any defects in the vehicle of which he has knowledge.
167
Q
  1. On May 19, a telephone operator for the local telephone company received a call in which a male voice said: “I want to report that the downtown sporting arena is going to be blown up tonight.” The caller then hung up. The line on which the call was made was a line owned by a woman.
    Immediately after receiving the call, the telephone operator reported the threatening conversation to the police. About half an hour later, during which time she had handled a number of other calls, the telephone operator received a call from a police officer who was at the woman’s home. He asked her to listen to a voice. After she did, the officer asked the telephone operator if she could identify it. The telephone operator responded that she was positive that it was the voice of the person who had made the threat. As a result of the telephone operator’s identification, the woman’s boyfriend was arrested and charged with the crime of terrorist threatening.
    As a consequence of the arrest, the boyfriend lost his job and suffered embarrassment and ridicule in the community. At trial, however, the telephone operator’s identification proved to be erroneous, and the boyfriend was exonerated.
    In a defamation action for slander by the boyfriend against the telephone operator and the telephone company, he will most likely
    (A) succeed, because the telephone operator’s erroneous identification constituted slander per Se.
    (B) succeed, because the telephone operator’s erroneous identification resulted in the loss of his good reputation in the community.
    (C) not succeed, because the telephone operator’s erroneous identification was made without actual malice.
    (D) not succeed, because the telephone operator’s erroneous identification was protected by a qualified privilege for statements made in the public interest.
A
  1. (D) A qualified privilege exists for statements made in the “public interest.” The privilege is a rather narrow one, protecting communication to those public officers or others who may reasonably be expected to take some effective action on a matter of public, rather than purely private, importance. Thus, anyone has a qualified privilege to give information to proper authorities for the prevention or detection of crime. Choices (A) and (B) are incorrect because the qualified privilege would protect the defendants from liability. Choice (C) is incorrect because the qualified privilege for reporting information of a crime is the key reason the defendants can avoid liability, and the lackof malice alone would not be enough to bar liability from being imposed.
168
Q
  1. A homeowner was planning to construct a new pooi in her back yard. She hired a well-known pool contractor to design and construct the pool. The contractor, in turn, hired a subcontractor to dig the hole and plaster the pooi area. After the subcontractor completed his job, the contractor then hired an electrician to install the heater and wiring for the pool. While the electrician was performing his work, the wiring became disjointed and had to be replaced at a substantial expense to the homeowner.
    If the homeowner sues the electrician in tort to recover the damages she suffered because of this occurrence, will the homeowner prevail?
    (A) Yes, if the wiring became disjointed because the electrician’s plans departed from established standards in the electrical industry.
    (B) No, unless the electrical wiring that became disjointed was defective.
    (C) No, if the electrician used his best professional judgment in performing the electrical work.
    (D) No, unless the homeowner knew that the contractor had hired the electrician to perform the electrical work.
A
  1. (A) Skilled workers, such as electricians and carpenters, are obligated to exercise the care of a reasonable person in their profession. Choice (A) is the best answer here because it shows that the electrician’s conduct fell below the reasonable standard required by members of his profession. Choice (B) is incorrect because it states that the only basis of recovery would be for strict liability. Since the homeowner may recover under a negligence theory, choice (B) is not the best answer. Choices (C) and (D) are incorrect because they state that the homeowner will not prevail. Here, the homeowner will prevail because the electrician, regardless of his using his best professional judgment, can still be liable if his conduct fell below that of a reasonable member of his profession.
169
Q
  1. A buyer purchased a new convertible from an automobile dealership. A few weeks later, the buyer began smelling a pungent gasoline odor in the vehicle. The buyer immediately took the car to the dealership and told the service manager what was wrong. The service manager indicated that he would take care of the problem and contact the buyer when the car was ready. The next day, the buyer was informed that the problem had been corrected and the car was available for pickup. The buyer then went to the dealership, took possession of his car, and drove off. After traveling about five miles, the buyer again smelled gasoline fumes. Irritated that the problem had not been corrected, the buyer decided to drive back to the dealership. As he was doing so, the car suddenly exploded, and the buyer suffered third-degree bums over 90 percent of his body.
    A subsequent investigation revealed that the explosion was caused by a defective gas tank that had ruptured. This produced a gasoline leak that was ignited by sparks from the car’s underbody. A reasonable inspection would have disclosed the defective gas tank. The trouble the buyer had described to the service manager was indicative of such a problem.
    If the buyer asserts a claim against the manufacturer of the convertible for damages for his injuries, will the buyer prevail?
    (A) Yes, if the dealership should have replaced the gas tank.
    (B) Yes, because the buyer’s injury was caused by the defective gas tank.
    (C) No, if the buyer should have realized the gasoline smell presented a hazardous condition and stopped the car before the explosion occurred.
    (D) No, unless the gas tank was defective when the car left the manufacturer’s plant.
A
  1. (D) A person, including a manufacturer, who is engaged in the business of selling a product, can be held strictly liable for injuries caused by the product’s being defective and unreasonably dangerous. The defect must have existed at the time the product left the defendant’s hands. Clearly, if the buyer’s car left the manufacturer’s plant in a defective condition, then strict liability would be imposed on the manufacturer for all harm proximately caused by the defect. Choice (0)15 correct. Choice (A) is incorrect because the manufacturer’s liability for selling a defective product does not depend on whether the car dealer was negligent. Choice (B) is incorrect because it addresses only the present defective condition of the product. In other words, the manufacturer will be strictly liable only if the defect existed when the car left the manufacturer’s hands, and not if the defect arose only later, after the product had been shipped by the manufacturer. Choice (C) is incorrect because even if the plaintiff was negligent for driving the car back to the dealership rather than stopping immediately, that negligence would merely reduce the plaintiff’s.recovery under comparative fault principles, rather than barring it completely.
170
Q
  1. A person purchased a new car from a local auto dealership. Over the next month, the purchaser drove the car a little more than 1,000 miles. The purchaser noticed that the car made a squeaking noise when the brakes were applied, so the purchaser took the car back to the dealership to have the mechanic there take a look at it. The mechanic worked on the car and then told the purchaser that the problem had been fixed. A few days later, the brakes failed, causing the purchaser to crash into a tree. The purchaser suffered a serious head injury in the crash.
    If the purchaser asserts a claim against the car dealership for damages for his injuries, will the purchaser prevail?
    (A) Yes, if the brakes were defective when the dealership sold the car to the purchaser.
    (B) Yes, because the dealership is strictly liable for defective repairs on cars it has sold.
    (C) No, because the car had been driven more than 1,000 miles.
    (D) No, unless the dealership’s employee was negligent in repairing the brakes.
A
  1. (A) A person who is engaged in the business of selling a product can be he’d strictLy liable for injuries caused by the product’s being defective and unreasonably dangerous. The defect must have existed at the time the defendant sold the product. The auto dealer will be strictly liable if the car’s brakes were defective at the time the dealer sold the car to the plaintiff. Choice (B) is incorrect because strict liability extends to defects existing at the time of sale, not to defective repairs. Choice (C) is incorrect because no matter how few or how many miles the car was driven, the dealer’s Liability is based on defects existing at the time of the sale. Choice (D) is incorrect because a defendant’s negligence is irrelevant if he is strictly liable. Exam Tip: When the question states that the plaintiff “asserts a claim” against the defendant, look at the status of the defendant to determine if he has potential strict liability for selling a defective product.
171
Q
  1. A woman was employed as a bank teller. One morning, a customer entered the bank to make a deposit. As the customer handed the deposit to the woman, she saw that he had a misprinted $5 bill in his possession. The woman knew that the $5 bill, which had President Lincoln’s picture upside down, was worth $500 to bill collectors. The woman then asked the customer if he would like to exchange “that old $5 bill for a new bill.” The customer accepted the woman’s offer and handed her the misprinted bill for a new one. One week later, the customer learned that the $5 bill that he gave the woman was valued at $500.
    If the customer asserts a claim against the woman for deceit, will he prevail?
    A) Yes, because the customer was the true owner of the misprinted bill and, therefore, he was entitled to the benefit of the bargain.
    (B) Yes, because the woman did not disclose the true value of the misprinted bill.
    (C) No, because the woman made no false representation of fact.
    (D) No, because the customer was not justified in relying on the woman’s offer.
A
  1. (B) The elements of deceit consist of the following: (1) a false representation made by the defendant; (2) knowledge or belief on the part of the defendant that the representation is false (i.e., “scienter”); (3) an intention to induce the plaintiff to act orto refrain from action in reliance upon the misrepresentation; (4) justifiable reliance upon the representation on the part of the plaintiff; and (5) damage to the plaintiff, resulting from such reliance. As a general rule, an action for deceit will not lie for such tacit nondisclosure. To this general rule, however, the courts have developed a number of exceptions. For instance, where the parties stand in some confidential or fiduciary relationship to one another, such as that of principal and agent, executor and beneficiary of an estate, bank and investing depositor, or majority and minority stockholders, there is a duty of full and fair disclosure of all material facts. Therefore, choice (B) is cortect because the woman, as a fiduciary, owed the customer a duty of disclosure and was aware that he was acting under a misapprehension as to the true value of the misprinted $5 bill. Choice (A) is incorrect because “benefit of the bargain” is not a tort remedy but, rather, a contract Law concept, which means that a part to a contract should be made “whole” if a breach occurs. Choice (C) is incorrect because there was a fiduciary duty between the parties and, therefore, a duty of full and fair disclosure of all material facts. Choice (D) is incorrect because the woman, as a fiduciary, breached the duty she owed to the customer, whether he was justified in relying upon her offer or not.
172
Q
  1. A property owner held title in fee simple to a tract of 20 acres located outside the boundaries of the city. Thereafter, the property owner constructed a shopping center on the property and leased commercial buildings and parking facilities to various tenants. The shopping center, which was located near a public high school, attracted many teenagers who often loitered in the parking lot. The youths frequently harassed shoppers and damaged autos by breaking off windshield wipers and radio antennas.
    Customarily, the local police department patrolled the shopping center and drove by three or four times each day. This, however, did not prevent the teenagers from hanging out at the shopping center. One afternoon, a shopper was shopping at the center when an unidentified youth damaged her car by throwing a rock through the back window.
    The shopper brings an action against the property owner to recover for the damage to her auto. She will most likely
    (A) prevail, unless the person who was responsible for damaging her car can be identified.
    (B) prevail, if the damage to her car could have been prevented had the property owner taken reasonable security measures.
    (C) not prevail, because the car was damaged by the malicious acts of an independent third person.
    (D) not prevail, because the local police had the primary duty to provide security protection at the shopping area.
A
  1. (B) This Multistate question is based on section 359 of the Restatement (Second) of Torts, which provides: “A lessor who leases land for a purpose which involves the admission of the public is subject to liability for physical harm caused to persons who enter the land for that purpose by a condition of the land existing when the lessee takes possession, if the lessor (a) knows or by the exercise of reasonable care could discover that the condition involves an unreasonable risk of harm to such persons, and (b) has reason to expect that the lessee will admit them before the land is put in safe condition for their reception, and (c) fails to exercise reasonable care to discover or to remedy the condition, or otherwise to protect such persons against it.” The property owner, as lessor, had a duty to exercise reasonable care to remedy the unsafe condition of the parking lot, so choice (B) is correct. Choice (A) is incorrect because the property owner could be held liable even if the youth who threw the rock could be identified. Choice (C) is incorrect because the property owner, as the lessor, had a duty to exercise reasonable care to remedy the unsafe condition and can be held liable for harm resulting from the youth’s malicious conduct. Choice (D) is incorrect because the police department’s performance of its functions does not relieve the property owner of his duty or liability.
173
Q
  1. A legislative assistant to a state senator approached various wealthy lobbyists without the senator’s knowledge to solicit illegal campaign contributions for the senator’s upcoming re-election campaign. The assistant dictated several letters requesting such contributions, which the senator signed without reading, along with a pile of other correspondence. Before the letters were mailed, however, the senator discovered what had happened and then immediately terminated the assistant’s employment.
    Later that same evening, after being notified that he was fired, the assistant returned to the senator’s office and used his keys, which had not yet been returned, to enter. The assistant made copies of the letters in question. The following day, the assistant turned over the copies of the letters to an investigative reporter from the local newspaper in the area. The reporter had heard about the assistant’s dismissal from another staff member and was curious about all the underlying circumstances. After the assistant provided the reporter with all of the pertinent facts, the reporter wrote a news story regarding the senator’s solicitation of illegal campaign contributions. Although the reporter’s story was printed in the newspaper, he did not reveal the source of his information. As soon as the publication was made, the FBI initiated an investigation of the senator’s campaign finances.
    If the senator asserts a claim based on invasion of privacy against the newspaper for the publication of the article concerning the solicitation of illegal campaign contributions, the most likely result is that the senator will
    (A) prevail, because the newspaper story placed him in a “false light in the public eye.”
    (B) prevail, because the newspaper disclosed private facts about the plaintiff.
    (C) not prevail, unless he is able to prove malice on the defendant’s part.
    (D) not prevail, because the newspaper was acting in the public interest by printing the news story.
A
  1. (C) The senator would be entitled to recover for invasion of privacy only if he were able to prove malice on the part of the newspaper. In Time, Inc. v. Hill, 385 u.s. 374 (1967), the u.s. Supreme Court applied the New York Times v. Sullivan rule to the tort of invasion of privacy. In other words, the Court extended the constitutional privilege (the guarantee of freedom of speech and of the press) to invasion of privacy cases where the published matter in question was in the public interest unless the plaintiff established that the defendant acted with “malice.” Thus, the misstatements of fact by the newspaper defendant are privileged unless the senator can prove that the misstatements were made with (1) knowledge of falsity, or (2) in reckless disregard of the truth. Students shouLd note that the two branches of invasion of privacy that turn on publicity (placing the plaintiff in a false light and public disclosure of private facts about the plaintiff), are both encompassed within the constitutional privilege. Choice (A) is incorrect because liability for placing the defendant in a false light could not be imposed without proof of malice. Choice (B) is incorrect for the same reason; the newspaper could not be held liable for public disclosure of private facts unless the plaintiff proves malice. Choice (D) is incorrect because if the plaintiff can prove malice, the newspaperwill be liable even ifitwas acting in the public interest by publishing the story.
174
Q
  1. A newspaper published a story about a U.S. senator having close ties to people suspected of being involved in organized crime. The senator claimed that the allegations in the newspaper story were false, and he brought a defamation action against the newspaper.
    Which of the following is the most accurate statement with regard to the newspaper’s liability?
    (A) A qualified privilege of fair comment existed because the newspaper was reporting on a matter of public interest.
    (B) Because the senator is a public figure, he has the burden of proof to show malice on the part of the newspaper.
    (C) The newspaper would not be liable because, under the First Amendment’s freedom of the press, the newspaper was privileged to publish the story.
    (D) The newspaper would be relieved of any liability for defamation because an absolute privilege existed.
A
  1. (B) In New York Times v. Sullivan, 376 u.s. 254 (1946), the u.s. Supreme Court held that the First Amendment’s guarantees of freedom of speech and freedom of the press confer a privilege on news media defendants that protects them from liability for false statements about public officials or figures if the statements were made without malice. Malice was further defined by the Court as (1) knowledge of falsity, or (2) reckless disregard of the truth. Therefore, a public official or public figure may not recover for defamatory remarks relating to his official conduct in the absence of proof that the defendant made the statement with malice. Choice (A) is incorrect because the qualified privilege is Lost if the newspaper published the statement with malice. Choice (C) is incorrect because the statements must be made without malice in order to have First Amendment protection. Choice (D) is incorrect because an absolute privilege did not exist in the facts presented. An absoLute privilege exists only for statements made in the following situations: (1) judicial proceedings; (2) legislative proceedings (i.e., matters covered by the Speech and Debate Clause); (3) executive proceedings; and (4) compelled broadcast situations, where the news media is compelled to carry a particular speaker’s message (such as a television station required to carry a speaker’s message or a newspaper required to print a notice or statement).
175
Q
  1. A U.S. senator fired one of her staff members for being insubordinate and refusing to follow the senator’s instructions about handling several important matters. To get revenge for being fired, the staff member forged some documents that falsely suggested the senator was taking bribes from lobbyists. The staff member anonymously mailed the forged documents to an investigative reporter from a major mitional newspaper. Based on the forged documents, the newspaper published stories accusing the senator of corruption. The staff member eventually admitted that he had forged and mailed the documents to the reporter.
    If the senator brings a defamation suit against the former staff member for forging and mailing the documents to the reporter, the senator will most likely
    (A) recover, because the staff member was aware that the documents were false.
    (B) recover, because the senator can prove that she suffered actual economic or other harm as a result of the newspaper stories.
    (C) not recover, because the senator is a public figure.
    (D) not recover, because the First Amendment provides a privilege for speech relating to a legislator and her staff.
A
  1. (A) Where a written statement is defamatory on its face (referred to as libel per se), the majority of American courts take the position that the injury to the plaintiff’s reputation is presumed by law. Libel per Se, thus, is actionable without pleading or proving special damages. In the present hypothetical situation, the information that the former staff member sent to the reporter would constitute LibeL per se because the documents imputed criminal conduct to the senator. Choice (A) is, therefore, correct because the senator can recover for defamation here without specific proof of harm to her reputation; that harm will be presumed. The fact that the former staff member knew the documents were false satisfies the malice requirement for defamation of a public official or figure. Choice (B) is incorrect because where an action involves libel per Se, proof of actual economic or other harm to the plaintiff’s reputation is not required. Choice (C) is incorrect because the former staff member knew the documents were false and defamatory, so the proof of malice required for a defamation suit by a public official or public figure is present. Choice CD) is incorrect because although legislators have a special privilege under the Constitution’s Speech and Debate Clause, that privilege extends only to statements that are part of the legislative process. Statements made by a former staff member about the senator would not be protected by the Speech and Debate Clause.